You are on page 1of 135

CATION (Secondary)

MULTIPLE CHOICE
1. Which does Naom Chomsky assert about language learning for children?
I. Young children learn and apply grammatical rules and vocabulary as they are exposed to them.
II. Begin formal teaching of grammatical rules to children as early as possible.
III. Do not require initial formal language teaching for children.
A. I and III C. I only
B. II only D. I and II
2. Which is/are the sources of man’s intellectual drives, according to Freud
A. Id C. Id -ego
B. Super ego D. Ego
3. Theft of school equipment like TV, computer, etc. by teenagers in the community itself is becoming a common phenomenon. What does
this incident signify?
A. Prevalence of poverty in the community.
B. Inability of school to hire security guards.
C. Deprivation of Filipino schools.
D. Community’s lack of sense of co-ownership.
4. As a teacher, you are reconstructionist, Which among these will be your guiding principle?
A. I must teach the
child every knowledge, skills and value that he needs for a better future.
B. I must teach the child to develop his mental powers to the full.
C. I must teach the child so he is assured of heaven.
D. I must teach the child that we can never have real knowledge of anything.
5. The concepts of Trust vs. mistrust, autonomy vs. shame & self-doubt, and initiative vs. guilt are most closely related with works of _____.
A. Erickson C. Freud
B. Piaget D. Jung

6. Student Z does not study at all but when the Licensure Examination for Teachers comes, before he takes the LET, he spends one hour or
more praying for a miracle, i.e., to pass the examination. Which attitude towards religion or God is displayed?
A. Religion as fake C. Religion as authentic
B. Religion as magic D. Religion as real
7. Teacher B engages her students with information for thorough understanding, for meaning and for competent application. Which principle
governs Teacher B’s practice?
A Constructivist C. Behaviorist
B. Gestalt D. Cognitivist
8. Student B claims: “I cannot see perfection but I long for it. So it must be real. “under which group can he be classified?
A. Idealist C. Realist
B. Empiricist D. Pragmatist

9. What does extreme authoritarianism in the home reinforce in learners


A. Doing things on their own initiative
B. Ability to direct themselves
C. Dependence on others for directions.
D. Creativity in work.
10. You arrive at knowledge by re-thinking of latent ideas. From whom does this thought come?
A. Experimentalist C. Idealist
B. Realist D. Existentialist
11. Behavior followed by pleasant consequences will be strengthened and will be more likely to occur in the future. Behavior followed by
unpleasant consequences will be weakened and will be more likely to be repeated in the future. Which one is explained
A. Freud’s psychoanalytic theory
B. Thorndike’s law effect
C. B.F. Skinner’s Operant conditioning theory
D. D. Bandura’s social learning theory
12. Principal B tells her teachers that training in the humanities is most important. To which educational philosophy does he adhere?
A. Existentialism C. Progressivis
B. Perennialism D. Essentialism
13. All subject in Philippine elementary and secondary schools are expected to be taught using the integrated approach. This came about as a
result of implementation of ___________.
A. Program for Decentralized Education
B. School-Based Management
C. Basic Education Curriculum
D. D. Schools First Initiative
14. Principal C shares this thought with the teachers: Subject matter should help students understand and appreciate themselves as unique
individual who accept complete responsibility for their thoughts, feelings and actions. “ From which philosophy is this thought based?
A. Perennialism C. Existentialism
B. Essentialism D. Progressivism
15. Based on Piaget’s theory, what should a teacher provide for children in the concrete operational stage?
A. Activities for hypothesis formulation.
B. Learning activities that involve problems of classification and ordering.
C. Games and other physical activities to develop motor skills.
D. Stimulating environment with ample objectives to play with.
16. To come closer to the truth we need to “go back to the things themselves.” This is the advice of the ______.
A. Behavioris C. idealist
B. phenomenologists D. pragmatists
17. Researches conducted show that teacher’s expectations of students become self-fulfilling prophecies. What is this phenomenon called?
A. Halo effect C. Ripple effect
B. Pygmalion effect D. Hawthorne effect

18. Teacher F is convinced that whatever a student performs a desired behavior, provided reinforcement and soon the student will learn to
perform the behavior on his own. On which principle is Teacher F’s conviction based?
A. Cognitivism C. Behaviorism
B. Environmentalism D. Constructivism
19. Teacher U teaches to his pupils that pleasure is not the highest good. Teacher’s teaching is against what philosophy?
A. Realism C. Epicureanism
B. Hedonism D. Empiricism
20. Studies in the areas of neurosciences discloses that the human brain has limitless capacity. What does this imply
A. Some pupils are admittedly not capable of learning.
B. Every pupil has his own native ability and his learning is limited to this native ability.
C. Every child is a potential genius.
D. Pupil can possibly reach a point where they have learned everything.
21. Availment of the Philippine Education Placement Test for adults and out-of-school youths is in support of the government’s educational
program towards _____.
A. equitable access C. quality and relevance
B. quality D. relevance
22. A mother gives his boy his favorite snack every time the boy cleans up his room. Afterwards, the boy cleaned his room everyday in
anticipation of the snack. Which theory is illustrated?
A. Associative learning
B. Classical conditioning
C. Operant conditioning
D. Pavlonian conditioning
23. Which group of philosophers maintain the “truth exists in an objective order that is independent of the knower”?
A. Idealists C. Existentialists
B. Pragmatists D. Realists
24. Under which program were students who were not accommodated in public elementary and secondary schools because of lack of
classroom, teachers, and instructional materials, were enrolled in private schools in their respective communities at the government’s expense?
A. Government Assistance Program
B. Study Now-Pay later
C. Educational Service Contract System
D. National Scholarship Program
25. Which of the following prepositions is attributed to Plato?
A. Truth is relative to a particular time and place.
B. Human beings create their own truths.
C. Learning is the discovery of truth as latent ideas are brought to consciousness
D. Sense perception is the most accurate guide to knowledge.
26. In a treatment for alcoholism, Ramil was made to drink an alcoholic beverage and then made to ingest a drug that produces nausea.
Eventually, h nauseated at he sight and smell of alcohol and stopped drinking alcohol. Which theory explains this?
A.Operant conditioning
C. Associative learning
B. Social learning theory
D. Attribution theory
27. In a social studies class, Teacher I present a morally ambiguous situation and asks his students what they would do. On whose theory is
Teacher I’s technique based
A. Kohlberg C. Piaget
B. Bandura D. Bruner
28. Quiz to formative test while periodic is to _________.
A. criterion-reference test
C. norm-reference test
B. summative test
D. diagnostic test
29. Your teacher is of the opinion that the world and everything in it are ever changing and so you the skill to cope with change. What is his
governing philosophy?
A. Idealism C. Experimentalism
B. Existentialism D. Realism
30. The search for related literature by accessing several data bases by the use of a telephone line to connect a computer that have database
is termed ______.
A. compact disc search C. on-line search
B. manual search D. computer search
31. In which competency do my students find the greater difficulty? In item with a difficulty index of
A. 0. 1 C. 0.5
B. 0. 9 D. 1.0
32. We encounter people whose prayer goes like this: O God, if there is a God; save my soul if I have a soul,” from whom is this prayer?
A. Stoic C. Agnostic
B. Empiricist D. Skeptic
33. If teacher wants to test student’s ability to organize ideas, which type of test should she formulate?
A. Short answer C. Essay
B. Technical problem type
D. Multiple-choice type
34. Who among the following needs less verbal counseling but needs more concrete operational forms of assistance? The child who ________.
A. has mental retardation
B. has attention-deficit disorder
C. has learning disability
D. has conduct disorder
35. Which applies when skewness is zero?
A. Mean is greater than median.
B. Median is greater than the mean.
C. Scores have three modes.
D. Scores are normally distributed 36. Which does NOT belong to the group of alternative learning systems
A. Multi-grade grouping
C. Graded education
B. Multi-age grouping
D. Non-graded grouping
37. The first thing to do in constructing a periodic test is for a teacher to ______.
A. decide on the number of items for the test
B. go back to her instructional objectives
C. study the content
D. decide on the type of test to construct
38. Which one can enhance the comparability of grades?
A. Using common conversation table for translating test scores in to ratings.
B. Formulating tests that vary from one teacher to another.
C. Allowing individual teachers to determine factors for rating
D. Individual teachers giving weights to factors considered for rating.
39. The cultivation of reflective and meditative skills in teaching is an influence of _____.
A. Shintoism C. Confucianism
B. Zen Buddhism D. Taoism
40. Teacher Y does norm-referenced interpretation of scores. Which of the following does she do?
A. She describes group performance in relation to a level of mastery set.
B. She uses a specified content as its frame of reference.
C. She compares every individual student’s scores with others’ scores.
D. She describes what should be their performance.
41. The best way for a guidance counselor to begin to develop study skills and habits in underachieving student would be to ______.
A. have these underachieving students observe the study habits of excelling students.
B. encourage students to talk study habits from their own experiences
C. have them view filmstrips about various study approaches
D. give out list of effective study approaches
42. Teacher F wanted to teach the pupils the skills to do cross stitching. He check up quiz was a written test on the steps of cross stitching. What
characteristic of a good test does it lack?
A. Scorability C. Objectivity
B. Reliability D. Validity
43. For which may you use the direct instruction method?
A. Become aware of the pollutants around us.
B. Appreciate Milton’s Paradise Lost.
C. Use a microscope properly.
D. Distinguish war from aggression.
44. In the context on the theory on multiple intelligences, what is one weakness of the paper-pencil test?
A. It is not easy to administer.
B. It puts the non-linguistically intelligent at a disadvantage
C. It utilizes so much time.
D. It lacks reliability.

45. NSAT and NEAT results are interpreted against set mastery level. This means that NSAT and NEAT fall under _________.
A. intelligence test C. criterion-referenced test
B. aptitude test D. norm-referenced test
46. Teacher B uses the direct instruction strategy. Which sequence of steps will she follow?
I. Independent practice
II. Feedback and correctiveness
III. Guided student practice
IV. Presenting and structuring
V. Reviewing the previous day’s work
A. V-IV-III-II-I C. V-II-IV-III-I
B. III-II-IV-I-V D. I-V-II-III-IV
47. Which guideline must be observed in the use of prompting to shape the correct performance of your students?
A. Use the list intrusive prompt first.
B. Use all prompts available.
C. Use the most intrusive prompt first.
D. Refrain form using prompts.
48. What measure/s of central tendency do/es the number 16 represent in the following data: 14,15,17,16,19,20,16,14,16?
A. Mode C. Median
B. Mode and median D. Mean
49. Which holds true to standardized tests?
A. They are used for comparative purposes.
B. They are administered differently.
C. They are scored according to different standards.
D. They are used for assigned grades.
50. Study this group of tests which was administered with the following results, then answer the question.
Subject Mean SD Ronnel’s scrore
Math 56 10 43
Physics 41 9 31
English 80 16 109
In which subject(s) were the scores most homogenous?
A. Math C. Physics
B. English D. Physics and Math
51. In the parlance of test construction what does TOS mean?
A. Table of Specifics C. Table of Specific Test Items
B. Term of Specifications D. Table of Specifications
52. Which is a major advantage of a curriculum-based assessment?
A. It is informal nature.
B. It connects testing with teaching.
C. It tends to focus on anecdotal information on student progress.
D. It is based on a norm-referenced measurement model.
53. Teacher H gave first-grade class a page with a story in which picture take the place of some words. Which method did she use?
A. The whole language approach
B. The Spaulding method
C. The rebus method
D. The language experience approach
54. Out of 3 distracters in a multiple choice test items, namely B, C, and D, no pupil choice D as answer. This implies that D is ________.
A. an ineffective distracter
B. a vague distracter
C. an effective distracter
D. a plausible distracter
55. The burnout malady gets worse if a teacher doesn’t intervene to change whatever areas he or she in control. Which one can renew a teacher’s
enthusiasm?
A. Stick to job C. Judge someone else as wrong
B. Initiate changes in jobs D. Engage in self-pity
56. If teacher has to ask more higher-order questions, he has to ask more ____ questions.
A. closed C. concept
B. Fact D. convergent
57. What can be said of Peter who obtained a score of P75 in a Grammar objective test?
A. His rating is 75 items in the test correctly.
B. He answered 75 % of the test items correctly.
C. He answered 75.
D. He performed better that 5% of his classmate.
58. I drew learners into several content areas and encouraged them to solve a complex question for interdisciplinary teaching. Which strategy did
I use?
A. Problem-centered learning
B. Unit method
C. Reading-writing activity
D. D. Thematic instruction
59. Which guidance in test construction is NOT observed in this test item “ Jose Rizal wrote ____”.
A. The central problem should be packed in the stem.
B. There must be only one correct answer.
C. Alternatives must grammatical parallelism.
D. The alternative must be plausible.
60. To elicit more students’ response, Teacher G. made use of covert responses. Which one did she NOT do?
A. She had the students write their response privately.
B. She showed the correct answers on the overhead after the students have written their responses.
C. She had the students write their responses privately then called each of them.
D. She refrained from judging on the student’s responses.
61. Direct instruction id a facts, rules, and actions as indirect instruction is for _____ ,____, _____.
A. hypotheses, verified data and conclusions
B. concepts, patterns and abstractions
C. concepts, processes and generalizations
D. guesses, data and conclusions
62. Which test has broad sampling of topics as strength?
A. Objective test C. Essay test
B. Short answer test D. Problem test .
63. Teacher T taught a lesson denoting ownership by means of possessives. He first introduced the rule, then gave examples, followed by
class exercises, then back to the rule before he moved into second rule. Which presenting technique did he use
A. Combinatorial C. Part-whole
B. Comparative D. Sequential
64. In his second item analysis, Teacher H found out that more from the lower group got the test item # 6 correctly. This means that the test item
_____.
A. has a negative discriminating power.
B. has a lower validity.
C. has a positive discriminating power.
D. has a high reability.
65. Teacher E discussed how electricity flows through wires and what generates the electric charge. Then she gave the students wires, bulbs,
switches, and dry cells and told the class to create a circuit that will increase the brightness of each bulb. Which one best describes the approach
used?
A. It used a taxonomy of basic thinking skills.
B. It was constructivist.
C. It helped students understand scientific methodology.
D. None of the above
66. Teacher W wants to review and check on the lesson of the previous day. Which one will be most reliable?
A. Having students identify difficult homework problems.
B. Having students correct each other’s work.
C. Sampling the understanding of a few students.
D. Explicitly reviewing the task-relevant information necessary for the day’s lesson.
67. Shown a picture of children in sweaters inside the classrooms, the students were asked this question: In what kind of climate do these
children live? This is a thought questions on ______.
A. inferring C. applying
B. creating D. predicting
68. Study this group of tests which was administered with the following results, then answer the question.
Subject Mean SD Ronnel’s scrore
Math 56 10 43
Physics 41 9 31
English 80 16 109
In which subject(s) did Ronnel perform best in relation to the group’s performance?
A.Physics and Math C. Physics
B.English D. Math
69. Which criterion should guide a teacher in the choice of instructional devices?
A. Attractiveness C. Novelty
B. Cost D. Appropriateness
70. Study this group of tests which was administered with the following results, then answer the question.
Subject Mean SD Ronnel’s scrore
Math 56 10 43
Physics 41 9 31
English 80 16 109
In which subject(s) did Ronnel perform poorly in relation to the group’s performance?
A. English C. Math
B. English and Math D. Physics
71. “What is most likely to happen to our economy when export continuously surpasses import” is a thought question on ______.
A. creating C. relating cause-and-effect
B. synthesizingD. predicting
72. Which method has been proven to be effective in courses that stress acquisition of knowledge?
A. Socratic method
C. Indirect instruction
B. Cooperative learning
D. Mastery learning
73. Which is the first step in planning an achievement test?
A. Define the instructional objective.
B. Decide on the length of the test.
C. Select the type of test items to use.
D. Build a table of specification.
74. Which activity should a teacher have more for his students if he wants to develop logical-mathematical thinking?
A. Drama C. Problem solving
B. Choral reading D. Storytelling
75. Why should a teacher NOT use direct instruction all the time
A. It requires much time.
B. It requires use of many supplemental materials.
C. It is generally effective only in the teaching of concepts and abstractions.
D. It reduces student’s engagement in learning.
76. Which are direct measures of competence?
A. Personality tests C. Paper-and-pencil test
B. Performance tests D. Standardized tests
77. Which is one role play in the pre-school and early childhood years?
A. Develops competitive spirit.
B. Separates reality from fantasy.
C. Increase imagination due to expanding knowledge and emotional range.
D. Develops the upper and lower limbs.
78. Teacher A discovered that his pupils are very good in dramatizing. Which tool must have helped him discover his pupils’ strength?
A. Portfolio assessment C. Journal entry
B. Performance test D. Paper-and-pencil test
79. Teacher M’s pupils are quite weak academically and his lesson is already far behind time table. How should Teacher M proceed with his
lesson?
A. Experientially C. Logically
B. Inductively D. Deductively
81. I want to teach concepts, patterns and abstractions. Which method is most appropriate?
A. Indirect instruction C. Direct instruction
B. Discovery D. Problem solving
82. If your Licensure Examination for Teacher (LET) items sample adequately the competencies listed in the syllabi, it can be said that LET
possesses ______ validity.
A. concurrent C. content
B. construct D. predictive
83. Read the following then answer the questions:
TEACHER: IN WHAT WAYS OTHER THAN THE PERIODIC TABLE MIGHT WE PREDICT THE UNDISCOVERED ELEMENTS?
BOBBY: WE COULD GO TO THE MOON AND SEE IF THERE ARE SOME ELEMENTS THERE WE DON’T HAVE.
BETTY: WE COULD DIG DOWN TO THE CENTER OF THE EARTH AND SEE IF WE FIND ANY OF THE MISSING ELEMENTS.
RICKY: WE COULD STUDY DEBRIS FROM THE METEORITES-IF WE CAN FIND ANY.
TEACHER: THOSE ARE ALL GOOD ANSWERS. BUT WHAT IF THOSE EXCURSIONS TO THE MOON, TO THE CENTER OF THE EARTH OR TO FIND
METEORITES WERE TOO COSTLY AND TIME CONSUMING? HOW MIGHT WE USE THE ELEMENTS WE ALREADY HAVE HERE ON EARTH TO FIND
SOME NEW ONES?
Question: The Teacher questions in the above exchange are examples of ____ questions.
A. Fact C. Direct
B. Concept D. Closed
84. In Krathwoh’s taxonomy of objectives in the affective, which is most authentic?
A.Characterization C. Responding
B. Valuing D. Organization
85. “In the light of the facts presented, what is most likely to happen when…?” is a simple thought question on _____.
A. inferring C. synthesizing
B. generalizing D. justifying
86. The teacher’s first task in the selection of media in teaching is to determine the
A. choice of the students
B. availability of the media
C. objectives of the lesson
D. technique to be used
87. In self-directed learning, to what extent should a teacher’s “scaffolding” be?
A. To a degree the student needs it.
B. None, to force the student to learn by himself
C. To the maximum, in order to extend to the student all the help he needs.
D. To the minimum, to spend up development of student’s sense of independence.
88. Which is a form of direct instruction?
A. Discovery process
C. Programmed instruction
B. Problem solving
D. Inductive reasoning
89. With synthesizing skills in mind, which has the highest diagnostic value?
A. Essay test C. Completion tes
B. Performance test D. Multiple choice test
90. How can you exhibit legitimate power on the first day of school?
A. By making your students feel they are accepted for who they are.
B. By informing them you are allowed to act in loco parents.
C. By making them realize the importance of good grades.
D. By making them feel you have mastery of subject matter.
91. Based on Edgar Dale’s Cone of Experience, which activity is farthest from the real thing?
A. View images C. Watch a demo
B. Attend exhibit D. Hear
92. Which can effectively measure student’s awareness of values?
A. Projective techniques C. Likert scales
B. Moral dilemma D. Anecdotal record
93. I combined several subject areas in order to focus on a single concept for interdisciplinary teaching. Which strategy/method did I use?
A. Problem-entered learning
B. Thematic instruction
C. Reading–writing activity
D. Unit method
94. The test item ”Group the following items according to shape” is a thought test item on _______.
A. creating C. classifying
B. generating D. comparing
95. For maximum interaction, a teacher ought to avoid _____ questions.
A. informational C. leading
B. rhetorical D. divergent
96. By what name is Socratic method also known ?
A. Mastery learning C. Morrison method
B. Indirect instruction D. Questioning Method
97. Which is an appropriate way to make manage off-task behavior?
A. Make eye contact.
B. Stop your class activity to correct a child who is no longer on task.
C. Move closer to the child.
D. Redirect a child’s attention to task and check his progress to make sure he is continuing to work
98. Which one can best evaluate student’s attitudinal development?
A. Essay test C. Observation
B. Portfolio D. Short answer test
99. What should a teacher do for students in his class who are on grade level
A. Give them materials on their level and let them work at a pace that is reasonable for them, trying to bring them up to a grade level.
B. Give them the same work as the other students, because they will absorb as much as they are capable of.
C. Give them the same work as the other students, not much, so that they won’t feel embarrassed.
D. Give them work on the level of the other students and work a little above the classmates level to challenge them.
100. With-it-ness, according to Kourin, is one of the characteristics of an effective classroom manager. Which phrase goes with it?
A. Have hands that write fast.
B. Have eyes on the back of your hands.
C. Have a mouth ready to speak.
D. Have minds packed with knowledge.
101. On whose philosophy was A.S.Neil Summerhill, one of the most experimental schools based?
A. A. Rousseau C. Montessori
B. B. Pestalozzi D. John Locke
102. Which Filipino trait works against the shift in teacher’s role from teacher as a fountain of information to teacher as facilitator?
A. Authoritativeness C. Hiya
B.Authoritarianism D. Pakikisama
103. Teacher A is a teacher of English as Second Language: she uses vocabulary cards, fill-in-the blank sentences, dictation and writing exercises in
teaching a lesson about grocery shopping. Based on this information, which of the following is a valid conclusion?
A. The teacher is applying Bloom’s hierarchy of cognitive learning.
B. The teacher is teaching variety of ways because not all students learn in the same manner.
C. The teacher wants to make her teaching easier by having less talk.
D. The teacher is emphasizing reading and writing skills.
104. With specific details in mind, which one has (have) a stronger diagnostic value?
A. Multiple choice test
B. Non-restricted essay test
C. Restricted essay test
D. Restricted and non-restricted essay tests
105. Teacher B is a teacher of English as Second Language: she uses vocabulary cards, fill-in-the blank sentences, dictation and writing exercises in
teaching a lesson about grocery shopping. Based on this information, which of the following is a valid conclusion?
A. The teacher is reinforcing learning by giving the same information in a variety of methods.
B. The teacher applying Bloom’s hierarchy of cognitive learning.
C. The teacher is wants to do less talk.
D. The teacher is emphasizing listening and speaking skills.
106. In a criterion-referenced testing, what must you do to ensure that your test is fair
A. Make all of the questions true or false.
B. Ask each student to contribute one question.
C. Make twenty questions but ask the students to answer only ten of their choice.
D. Use objectives for the unit as guide in your test construction.
107. Read this question: “How will you present the layers of the earth to your class?” This is a question that _________.
A. directs
B. leads the students to evaluate
C. assesses cognition
D. probes creative thinking
108. In Krathwoh’s affective domain of objectives, which of the following is the lowest level of effective behavior?
A. Valuing C. Responding
B. Characterization D. Organization
109. Which is NOT a sound purpose for asking questions?
A. To probe deeper after an answer is given
B. To discipline a bully in class
C. To remind students of a procedure
D. To encourage self-reflection
110. It is not wise to laugh at a two-year old child when he utters bad word because in his stage he is learning to __________.
A. considered other’s views
B. distinguish right from wrong
C. socialize
D. distinguish sex differences
111. Research tells that teachers ask mostly content questions. Which of the following terms does NOT refer to content questions?
A. Closed C. Concept
B. Direct D. Convergent
112. In mastery learning, the definition of an acceptable standard of performance is called a
A. behavior C. SMART
B. condition D. criterion measure
113. Based on Edgar Dale’s Cone of Experience, which activity is farthest from the real thing?
A. Read C. View images
B. Hear D. Attend exhibit
114. After giving an input on a good paragraph, Teacher W asks her students to rate a given paragraph along the elements of a good paragraph.
The students’ task is in level of _________.
A. application C. evaluation
B. analysis D. synthesis
115. Which is one characteristic of an effective classroom management?
A. It quickly and unobtrusively redirects misbehavior once it occurs.
B. It teaches dependence on other for self-control.
C. It respects cultural norms of a limited group students.
D. Strategies are simple enough to be used consistently
116. Read the following then answer the questions:
TEACHER: IN WHAT WAYS OTHER THAN THE PERIODIC TABLE MIGHT WE PREDICT THE UNDISCOVERED ELEMENTS?
BOBBY: WE COULD GO TO THE MOON AND SEE IF THERE ARE SOME ELEMENTS THERE WE DON’T HAVE.
BETTY: WE COULD DIG DOWN TO THE CENTER OF THE EARTH AND SEE IF WE FIND ANY OF THE MISSING ELEMENTS.
RICKY: WE COULD STUDY DEBRIS FROM THE METEORITES- IF WE CAN FIND ANY.
TEACHER: THOSE ARE ALL GOOD ANSWERS BUT WHAT IF THOSE EXCURSIONS TO THE MOON, TO THE CENTER OF THE EARTH OR TO FIND
METEORITES WERE TOO COSTLY AND TIME CONSUMING? HOW MIGHT WE USE THE ELEMENTS WE ALREADY HAVE HERE ON EARTH TO FIND
SOME NEW ONES?
Question: which questioning strategy/ies does/do the exchange of thoughts above illustrate?
A. Funneling
B. Sowing and reaping
C. Nose-dive
D. Extending and lifting
117. How can you exhibit referent power on the first day of school?
A. By making them feel you know what you are talking about.
B. By telling them the importance of good grades.
C. By reminding your students your authority over them again and again.
D. By giving your students a sense of belonging and acceptance.
118. A sixth grade twelve-year old boy comes from a dysfunctional family and has been abused and neglected. He has been to two orphanages
and three different elementary schools. The student can decode on the second grade level, but he can comprehend orally material at the fourth
or fifth grade level. The most probable cause/s of this student’s reading problem is/are ______.
A. emotional factors
B. poor teaching
C. neurological factors
D. immaturity
119. Which questioning practice promotes more class interaction?
A. Asking the question before calling a student
B. Focusing on divergent questions.
C. Focusing on convergent questions.
D. Asking rhetorical questions
120. The following are sound specific purpose of questions EXCEPTS
A. To call the attention of an inattentive student
B. To teach via student answers
C. To stimulate learners to ask questions
D. To arouse interest and curiosity
121. Which technique should a teacher use to encourage response if his students do not respond to his question?
A. Ask a specific student to respond, state the question, and wait a response.
B. Tell the class that it will have detention unless answers are forthcoming.
C. Ask another question, an easier one.
D. Wait for a response.
122. Which types of play is most characteristic of a four to six-year old child?
A. Solidarity plays and onlookers plays
B. Associative and cooperative plays
C. Associative and onlookers plays
D. Cooperative and solidarity plays
123. The principle of the individual difference requires teachers to ___________.
A. give greater attention to gifted learners
B. provide for a variety of learning activities
C. treat all learners alike while in the classroom
D. prepare modules for slow learners in class
124. Referring to Teacher S, Nicolle describes her teacher as “fair, caring and someone you can talk to”. Which power or leadership does Teacher S
have?
A. Referent power C. Reward power
B. Legitimate power D. Expert power
125. During the Spanish period, what was/were the medium/media of instruction in schools?
A.The Vernacular C. Spanish
B.English D. Spanish and the Vernacular
126. Rodel is very aloof and cold in his relationship with classmates. Which basic goals must have not been attained by Rodel during his
developmental years, according to Erickson’s theory of psychological development?
A. Autonomy C. Initiative
B. Trust D. Generativity
127. How can you exhibit expert power on the first day of school?
A. By making them feel you know what you are talking about.
B. By telling them the importance of good grades.
C. By reminding then your students your authority over them again and again.
D. By giving your students a sense of belonging and acceptance.
128. Teacher P wants to develop the skill of synthesizing in her pupil. Which one will she do?
A. Ask her students to formulate a generalization from the data shown in graphs.
B. Ask her students to answer questions beginning with “What if …”
C. Tell her pupils to state data presented in graphs.
D. Directs her students to ask questions on the parts of the lesson not understood.
129. John Watson said “Men are built not born. ” What does this statement point to?
A. The ineffectiveness of training on a person’s development.
B. The effect of environmental stimulation on a person’s development.
C. The absence of genetic influence on a person’s development.
D. The effect of heredity.
130. Teacher H strives to draw participation of every student into classroom discussion. Which student’s need is she trying to address? The need
A. to show their oral abilities to the rest of the class
B. to be creative
C. to feel significant and be part of a group
D. to get everything out in the open
131. Teacher G’s lessons objective has something to do with the skill of synthesizing? Which behavioral term is most appropriate?
A. Test C. Appraisal
B. Assess D. Theorize
132. Which is a sound classroom management practice?
A. Avoid establishing routines; routines make your student robots.
B. Establish routines for all daily needs and tasks.
C. Apply rules and policies on a case to case basis.
D. Apply reactive approach to discipline.
133. A child who gets punished for stealing candy may not steal again immediately. But this does not mean that the child may not steal again.
Based on Thorndike’s theory on punishment and learning, this shows that ______________.
A. punishment strengthen e a response
B. punishment remove a response
C. punishment does not remove a response
D. punishment weakens a response
134. Which assumption underlines the teacher’s use of performance objectives?
A. Not every form of learning id observable.
B. Performance objectives assure the learner of learning.
C. Learning is defined as a change in the learner’s observable performance.
D. The success of the learner is based on teacher’s performance.
135. As a teacher, what do you do when you engage yourself in major task analysis?
A. Test if learning reached higher level thinking skills
B. Breakdown a complex task into sub-skills
C. Determine the level of thinking involved
D. Revise lesson objectives
136. The following are used in writing performance objectives EXCEPT
A. delineate C. integrate
B. diagram D. comprehend
137. Teacher B clears his throat to communicate disapproval of a student’s behavior. Which specific influence technique is this?
A. Signal interference C. Interest boosting
B. Direct appeal D. Proximity control
138. An effective classroom manager uses low-profile classroom control. What is a low-profile classroom technique?
A. Note to parents
B. After-school detention
C. Withdrawal of privileges
D. Raising the pitch of the voice
139. The primary objective of my lesson is: “To add similar fractions correctly.” Before I can do this I must first aim at this specific objective: “to
distinguish a numerator from a nominator. “What kind of objective is the latter?
A. Major C. Enabling
B. Terminal D. Primary
140. With which goals of educational institutions as provided for by the Constitution is the development of work skills aligned?
A. To develop moral character
B. To teach the duties of citizenship
C. To inculcate love of country
D. To develop vocational efficiency
141. In instructional planning it is necessary that parts of the plan from the first to the last have ___________.
A. clarity C. coherence
B. symmetry D. conciseness
142. All of the following describe the development of children aged eleven to thirteen EXCEPT
A. they shift from impulsivity to adaptive ability
B. sex differences in IQ becomes more evident
C. they exhibit increased objectivity in thinking
D. they show abstract thinking and judgement
143. If a teacher plans a constructivist lesson, what will he most likely do? Plan how he can _______.
A. do evaluate his student’s work
B. do reciprocal teaching
C. do lecture to his students
D. engage his students in convergent thinking
144. In mastery learning, the definition of an acceptable standard of performance is called a
A. SMART C. behavior
B. criterion measure D. condition
145. “A stitch on time saves nine”, so goes the adage. Applied to classroom management, this means that we _______.
A. may not occupy ourselves with disruptions which are worth ignoring because they are minor
B. must be reactive in our approach to discipline
C. have to resolve minor disruptions before they are out of control
D. may apply 9 rules out of 10 consistently
146. Ruben is very attached to his mother and Ruth to her father. In what developmental stage are they according to Freudian psychological
theory?
A. Oedipal stage C. Anal Stage
B. Latent stage D. Pre-genital stage
147. What was the prominent educational issues of the mid 1980’s?
A. Bilingual Education C. Accountability
B. Value Education D. Mainstreaming
148. Which behavior is exhibited by a student who is strong in interpersonal intelligence?
A. Works on his/her own.
B. Keeps interest to himself/herself
C. Seeks out a classmate for help when problem occurs.
D. Spends time meditating.
149. Which is behavioral term describes a lesson outcome in the highest level of Bloom’s cognitive domain?
A. Create C. Analyze
B. Evaluate D. Design
150. The main purpose of compulsory study of the Constitution is to ___.
A. develop students into responsible, thinking citizens
B. acquaint student with the historical development of the Phil Constitution
C. make constitutional experts of the students
D. prepare students for law-making
151. A goal-oriented instruction culminates in _______.
A. planning activities
B. evaluation
C. identification of topics
D. formulation of objectives
152. Based on Freud’s psychoanalytic theory which component (s) of personality is (are) concerned with a sense of right and wrong?
A. Super ego C. Id
B. Super-ego and Ego D. Ego
153. Who among the following puts more emphasis on core requirements, longer school day, longer academic year and more challenging
textbooks?
A. Perennialist C. Progressivist
B. Essentialist D. Existentialist
154. A student passes a research report poorly written but ornately presented in a folder to make up for the poor quality of the book report
content. Which Filipino trait does this practice prove?
A. art of academics
B. substance over “porma”
C. art over science
D. “porma” over substance
155. Which one may support equitable access but may sacrifice quality?
A.Open admission
C. Deregulated tuition fee hike
B. School accreditation
D. Selective retention
156. Based on Piaget’s theory, what should a teacher provide for children in the sensimotor stage?
A. Games and other physical activities to develop motor skill.
B. Learning activities that involve problems of classification and ordering.
C. Activities for hypothesis formulation.
D. Stimulating environment with ample objects to play with.
157. A teacher’s summary of a lesson serves the following functions, EXCEPT
A. it links the parts of the lesson.
B. it brings together the information that has been discussed.
C. it makes provisions for full participation of students.
D. it clinches the basic ideas or concepts of the lesson.
158. As a teacher, you are rationalist, Which among these will be your guiding principle?
A. I must teach the child that we can never have real knowledge of anything.
B. I must teach the child to develop his mental powers to the full.
C. I must teach the child so he is assured of heaven
D. I must teach the child every knowledge, skills and value that he needs for a better future.
159. Bruner’s theory on intellectual development moves from enactive to iconic and symbolic stages. In which stage(s) are diagrams helpful to
accompany verbal information?
A. Enactive and iconic C. Symbolic and enactive
B. Symbolic D. Iconic
160. Student’s scores on a test were: 72,72,73,74,76,78,81,83,85.
The score 76 is the ___.
A. mode C. average
B. mean D. median
161. Standard deviation is to variability as mode to _____.
A. level of difficulty C. correction
B. discrimination D. central tendency
162. A teacher who equates authority with power does NOT usually __________.
A.Shame C. develop self-respect in every pupil
B.retaliate D. intimidate
163. Which is a true foundation of the social order?
A. Obedient citizenry
B. The reciprocation of rights and duties
C. Strong political leadership
D. Equitable distribution of wealth
164. Standard deviation is to variability as mean is to _______.
A. coefficient of correlation C. discrimination index
B. central tendency D. level of difficulty
165. Teacher Q does not want Teacher B to be promoted and so writes an anonymous letter against Teacher B accusing her of fabricated lies.
Teacher Q mails this anonymous letter to School Division Superintendent. What should Teacher Q do if she has to act professionally?
A. Submit a signed justifiable criticism against Teacher B, if there is any.
B. Go straight to the School Division Superintendent and gives criticism verbally.
C. Hire a group to distribute poison letters against Teacher B for the information dissemination.
D. Instigate student activists to read poison letter over the microphone.
166. Each teacher is said to be a trustee of the cultural and educational heritage of the nation and is under obligation to transmit to learners such
heritage. Which practice makes him fulfill such obligation?
A. Use the latest instruction technology
B. Observing continuing professional education.
C. Use interactive teaching strategies
D. Study the life of Filipino heroes
167. Which type of reports to “on-the spot” description of some incident, episode or occurrence that is being observed and recorded as being of
possible significance?
A. Autobiographical report
B. Value and interest report
C. Biographical report
D. Anecdotal report
168. Teacher A is directed to pass an undeserving student with a death threat. Which advice will a hedonist give?
A. Pass the student. Why suffer the threat?
B. Don’t pass him. You surely will not like someone to give you a death threat in order to pass.
C. Pass the student. That will be use to the student, his parents and you.
D. D. Don’t pass him. Live by principle of justice. You will get reward, if not in this life, in the next.
169. If you agree with Rizal on how you can contribute to our nation’s redemption, which should you work for?
A. Opening our doors to foreign influence
B. Upgrading the quality of the Filipino through education
C. Stabilizing the political situation
D. Gaining economic recovery
170. “All men are pretty much alike. It is only by custom that they are set apart, “ said one Oriental philosopher. Where can this thought be most
inspiring?
A. In a multi-cultural group of learners.
B. In multi-cultural and heterogeneous groups of learners and indigenous peoples’ group.
C. In a class composed of indigenous people.
D. In heterogeneous class of learners.
171. In what way can teachers uphold the highest possible standard of quality education?
A. By continually improving themselves personally and professionally
B. By wearing expensive clothes to change people’s poor perception of teachers
C. By working out undeserved promotions
D. By putting down other professions to lift the status of teaching
172. How would you select the most fit in government position? Applying Confucius teaching, which would be the answer?
A. By course accreditation of an accrediting body
B. By merit system and course accreditation
C. By merit system
D. By government examinations
173. The attention to the development of a deep respect and affection for our rich cultural past is an influence of ______
A. Confucius C. Teilhard de Chardin
B. Heqel D. Dewey
174. A teacher / student is held responsible for his actions because s/he _______.
A. has instincts C. has mature
B. has a choice D. is reason
175. The typical autocratic teacher consistently does the following EXCEPT
A. intimidating students
C. shaming students
B. ridiculing students
D. encouraging students
176. Teacher H and teacher I are rivals for promotion. To again the favor of the promotion staff, teacher I offers her beach resort for free for
members of he promotional staff before ranking. As one of the contenders for promotions, is this becoming of her to do
A. Yes. This will be professional growth for the promotional staff.
B. No. This may exert undue influence on the members of the promotional staff and so may fail to promote on the basis of merit.
C. Yes. The rare invitation will certainly be welcomed by an overworked promotional staff.
D. Yes. There’s nothing wrong with sharing one’s blessings.
177. Rights and duties are correlative. This means that.
A. rights and duties regulate the relationship of men in society.
B. rights and duties arise from natural law.
C. each right carries with it one or several corresponding duties.
D. rights and duties ultimately come from God.
178. In the Preamble of Code of Ethics of Professional Teachers, which is NOT said of teachers?
A. LET passer
B. Duly licensed professionals
C. Possess dignity and reputation
D. With high moral values as well as technical and professional competence
179. What should you do if a parent who is concerned about a grade his child received compared to another student’s grade, demands to see
both students’ grades?
A. Refuse to show either record.
B. Show only his child’s record.
C. Refuse to show any record without expressing permission from principal.
D. Show both records to him.
180. Teacher often complain of numerous non-teaching assignments that adversely affect their teaching. Does this mean that teachers must be
preoccupied only with teaching?
A.
Yes, if they are given other assignments, justice demands that they be properly compensated.
B. Yes, because other community leaders, not leaders, not teachers, are asked to lead in community activities.
C. Yes, because every teacher is expected to provide leadership and initiative in activities for betterment of communities.
D. No, because teaching is enough full time job.
181. Which illustrates a development approach in guidance and counseling?
A. Spotting on students in need of guidance
B. Teaching students how to interact in a positive manner
C. Acting as a mentor
D. Making the decision for the confused student
182. Whose influence is the education program that puts emphasis on self-development through the classics, music, and ritual?
A. Buddha C. Confucius
B. Mohammed D. Lao tsu
183. Helping in the development of graduates who are “maka-Diyos” is an influence of ___.
A. naturalistic morality
B. classical Christian morality
C. situational morality
D. dialectical morality
184. What is the mean of this score distribution 4,5,6,7,8,9,10?
A. 7 C. 8.5
B. B. 6 D. 7.5
185. Are percentile ranks the same as percentage correct?
A. It cannot be determined unless scores are given.
B. It cannot be determined unless the number of examinees is given
C. No
D. Yes
186. Teacher F is a newly converted to a religion. Deeply convinced of his new found religion, he starts Monday classes by attacking one religion
and convinces his pupil to attend their religion services on Sundays. Is this in accordance with the Code of Ethics of Professional Teachers?
A. Yes. What he does strengthens value education.
B. No. A teacher should not use his position to proselyte others.
C. Yes. In the name of academic freedom, a teacher can decide what to teach.
D. Yes. What he does is a value education.
187. In a study conducted, the pupils were asked which nationality they preferred. If given a choice. Majority of the pupils wanted to Americans.
In this case, in which obligation, relative to the state, do schools seem to be failing? In their obligation to
A. respect for all duly constituted authorities
B. promote national pride
C. promote obedience to the laws of the state
D. install allegiance to the Constitution
188. A guest in one graduation rites told his audience: “Reminder, you are what you choose to be”. The guest speaker is more of a/an _____.
A. realistic C. pragmatist
B. Idealistic D. existentialist
189. From whom do we owe the theory of deductive interference as illustrated in syllogism?
A. Plato C. Socrates
B. Aristotle D. Pythagoras
190. In what way can teachers uphold the highest possible standard of quality education?
A. By continually improving themselves personally and professionally
B. By putting down other professions to lift the status of teaching
C. By wearing expensive clothes to change people’s poor perception of teachers
D. By working out undeserved promotions
191. Two students are given the WISC III. One has a full scale IQ of 91, while the other has an IQ of 109. Which conclusion can be drawn?
A. Both students are functioning in the average range of intellectual ability
B. Another IQ test should be given to truly assess their intellectual potential.
C. The first student is probably below average, while the second has above average potential.
D. The second student has significantly higher intellectual ability.
192. Which describes norm-referenced grading?
A. What constitutes a perfect score
B. The student’s past performance
C. An absolute standard
D. The performance of the group
193. Teacher A knows of the illegal activities of a neighbor but keeps quiet in order not to be involved in any investigation. Which foundational
principle of morality does Teacher A fail to apply?
A. Always do what is right
B. The principle of double effect
C. The end does not justify the means
D. Between two evils, do the lesser evil
194. Teacher Q does not want Teacher B to be promoted and so writes an anonymous letter against Teacher B accusing her of fabricated lies.
Teacher Q mails this anonymous letter to School Division Superintendent. What should Teacher Q do if she has to act professionally?
A. Hire a group to distribute poison letters against Teacher B for the information dissemination.
B. Submit a signed justifiable criticism against Teacher B, if there is any.
C. Go straight to the School Division Superintendent and gives criticism verbally.
D. Instigate student activists to read poison letter over the microphone
195. A teacher’s summary of a lesson serves the following functions, EXCEPT
A. it makes provisions for full participation of students.
B. it brings together the information that has been discussed.
C. it links the parts of the lesson.
D. it. clinches the basic ideas or concepts of the lesson
196. Soc exhibits fear response to freely roaming dogs but does not show fear when a dog is on a leash or confined to a pen. Which conditioning
process is illustrated?
A. Extinction C. Generation
B. Discrimination D. Acquisitio
197. Ruben is very attached to his mother and Ruth to her father. In what developmental stage are they according to Freudian psychological
theory?
A. Latent stage C. Anal Stage
B. Pre-genital stage D. Oedipal stage
198. The concepts of Trust vs. mistrust, autonomy vs. shame & self-doubt, and initiative vs. guilt are most closely related with works of _____.
A. Jung C. Erickson
B. Freud D. Piaget
199. A goal-oriented instruction culminates in _______.
A. evaluation
B. formulation of objectives
C. identification of topics
D. planning activities
200. Which of the following is considered a peripheral device?
A. Keyboard C. Monitor
B. CPU D. Printer
18. C 67. A 117. A 167. D

ANSWER KEY 19. B 68. C 118. A 168. D

PROFESSIONAL EDUCATION 20. C 69. D 119. B 169. B

(Secondary) 21. A 70. A 120. A 170 B

1. A 50. C 100. D 150. B


22. B 71. C 121. C 171. A
2. B 51. B 101. no answer 151. D
23. A 72. C 122. C 172. D
3. D 52. D 102. B 152. A
24. C 73. A 123. B 173. A
4. A 53. no answer 103. B 153. no answer
25. no answer 74. A 124. A 174. B
5. A 54. A 104. B 154. D
26. C 75. D 125. C 175. A
6. B 55. B 105. A 155. A
27. A 76. B 126. B 176. B
7. D 56. C 106. D 156. A
28. D 77. C 127. D 177. C
8. C 57. A 107. D 157 C
29. C 78. B 128. A 178. A
9. C 58. D 108. C 158. B
30. C 79. B 129. B 179. D
10. C 59. B 109. B 159. no answer
31. A 80. C 130. C 180. C
11. C 60. D 110. C 160. D
32. C 81. C 131. D 181. A
12. D 61. B 111. D 161. D
33. C 82. C 132. C 182. A
13. C 62. C 112. C 162. C
34. A 83. C 133. D 183. B
14. C 63. D 113. C 163. D
35. D 84. D 134. C 184. A
15. B 64. A 114. A 164. B
36. C 85. A 135. B 185. C
16. C 65. D 115. D 165. A
37. B 86. C 136. D 186. B
17. B 66. A 116. A 166. D
38. A 87. C 137. B 187. B

39. B 88. A 138. C 188. D

40 C 89. A 139. C 189. B

41. B 90. B 140. D 190. A

42. D 91. B 141. C 191. B

43. A 92. B 142. D 192. D

44. B 93. B 143. D 193. C

45 C 94. C 144. B 194. B

46. C 95. B 145. C 195. D

47. A 96. D 146. A 196. B

48. A 97. D 147. B 197. D

49. A 98. B 148. C 198. C

99. A 149. B 199. A

200. D
D. “Happy are those who are merciful to 6. Mang Tacio has been unemployed
others; God will be merciful to them!” for quite sometime due to his negative
General Education (secondary)
3. Which is NOT personal integrity? attitude toward work. Which program of
MULTIPLE CHOICE
A. Time C. Order the Department of Social Welfare and
1. Which refers to FUNNEL EFFECT?
B. Place D. Harmony Services will help him?
A. The belief that every criminal gets
4. Carter’s part in relinquishing U.S. A. Income in Kind Program
caught and is punished.
control of the Central Zone to Panama is C. HRD Program
B. The belief that crime is under control
described as a victory for ________. B. Anti-Medicancy Program
in the United States.
A. conservatism D. Social Insurance Program
C. The idea that only a very few suspects
C. isolationism 7. The main message of the Moral
arrested for committing a crime are
B. anti-imperialism Recovery Program launched by Leticia
actually punished.
D. imperialism Ramos Shahani starts with the ________.
D. The idea that all crimes put into the same
5. Of the following changes in the A. world
criminal justice system.
socio-economic, political cultural and C. self
2. One of the most outstanding
physical that have occurred in the Filipino B. family
accomplishments of the cooperative
family, which one remains to be TRUE? D. nation
movement is the encouragement of thrift.
A. The loss of the traditional evening 8. Which are limited only to the sale of
Which maximum of God puts this into
prayer and ritual of blessing (mano) real property and stock transaction?
practice?
B. The unity of the family despite A. Business incomes
A. “God Helps those who help
competing demands C. Employment incomes
themselves”.
C. The continued parental influence B. Capital gains
B. “Look at the birds: they do not plant
over children’s language dress and other D. Passive incomes
seeds, gather a harvest and put it in barns;
behavior 9. If a farmer would want assistance
yet your Father in heaven takes care of
D. The continued support for parents like pricing, guarantee for all agricultural
them!”
and siblings produce or cooperative management
C. “He is near to those who call to Him,
training, where would he go?
who call to Him with sincerity.”
A. Support services of the Department B. The safety of the community over the German culture came from a strong,
of Agrarian Reform sympathy for the criminal healthy and rustic lineage?
B. Special Agrarian Court under the C. The needs of the criminals over the
Regional Trial Court advice of the judge A. Dependency Theory
C. DAR adjudication board D. The punishment of the criminal over B. Culture of Poverty Theory
D. Land Bank the safety of the community C. Social Darwinist Theory
10. The Soviet Union’s attempt to 13. Lucy’s husband has been a drug D. Theory of Capitalism
establish a missile base in Cuba is dependent. She wanted him rehabilitated 17. Why was San Andres Cooperative
interpreted as a direct violation of to be economically productive. Where will Association of Paco, not exempted
A. The Truman Doctrine she commit her husband? from taxation?
B. The Monroe Doctrine A. DARN A. I accumulates reserves and undivided
C. The Strategic Arms Limitation Talks C. DARE net savings of P8,000,000.00
(SALT) B. Bukang Liwayway Ceter B. I accumulates reserves and undivided
D. The Declaration of Independence D. NFPI net savings of P10,000.00
11. As a representative of the Urban 14. Which will solve poverty caused by C. I accumulates reserves and undivided
Poor Commission of the Association of capitalism? net savings of P9,000,000.00
Religious Superiors (ARS), which action will A. Fascism D. I accumulates reserves and undivided
you most likely take to resolve the long- C. Empirism net savings of P11,000.00
term roots of structural inequalities- B. Communism 18. Juliet Villaruel was a landowner
proliferation of child labor and child D. Socialism from Cabio, Nueva Ecija. Under the CARL,
prostitution? 15. Which family obligation is especially she was claiming 8 hectares, 5 hectares of
A. Raise views of human rights abuse. valued? which represented the retention limit and
B. Organize regular programs for A. Supplying groceries to relatives in the 3 hectares for her only child. Why was
information and discussion remote barrios her child denied 3 hectares?
of human rights B. Providing health assistance to
C. Conduct skills training relatives living in the same locale A. Her son was 15 years old who was
D. Raise questions over the C. Sending to college relatives in actually tilting the farm
government’s commitment to rebuild remote barrios B. Her son was 17 years old who was
human rights D. Keeping immediate family managing the farm
12. What values are being given priority members out of trouble C. Her son was 13 years old who has
by juries in criminal cases? 16. Which theory was asserted by the been helping till the farm
A. The rights of the criminal over the Pan-German belief in the superiority of the D. Her son was 19 years old who was
strict interpretation of the law Aryan race and that the strength of the actually tilting or managing the farm
19. Which part of the Allied action has 22. The following are defects present at B. those legitimate.
been detailed in this passage? the time of marriage which is voidable and C. those born during a valid marriage
After Saddam Hussein violated international annullable EXCEPT of parents.
arguments by sending Iraq troops to Kuwait A. impotence D. those born out a valid marriage of
and missiles into their neighboring C. fraud parents.
countries, the Allies responded with military B. deceit 27. Of the following, which is imposed a
action. D. threat final tax of ten percent (10%)
A. The Allied bases in Saudi Arabia A. PSCO and lotto winnings.
B. The ground was in the desert 23. Why does a pendulum in a B. Books literary works and musical
C. The movement of Allied troops in Iraq grandfather clock once set in motion compositions
D. The campaign in the Baghdad area continue to swing, thereby regulating the C. Currency banks deposit
20. Which one is the human right to clock’s movement? This is due to the Law of D. Royalties
life? A. Universal Gravitation 28. Which of the following foreign policy
A. Peace C. Applied Force actions today is a direct result of early
C. Own Property B. Action and Reaction American imperialist policy?
B. Live in national and international order D. Inertia A. Station of American troops in West
D. Fair trial 24. New ponies, perennial plants that Germany
21. Two days after Japan attacked Pearl produce shows flowers can be propagated B. American’s military support of Israel
Harbor, Roosevelt made the following from the parent plant by dividing corns that in the Middle East
statements: grow underground. The reproductive form C. American’s patrolling of waters of
“In the past few years and most violently in resembles a _____________. the Libyan coast
the past few days, we learned a terrible A. bulb C. seed D. American’s establishment of military
lesson. We must begin the great task that is B. runner D. bud bases I nth Philippines
before us by abandoning once and for all 25. The San Pascual Credit Cooperative 29. Which area of the brain controls
the illusion that we can ever again isolate of Quezon City wishes to apply for a loan of feelings on the side of a person’s face?
ourselves from the rest of the humanity.” five pesos from one of the financial A. The left occipital lobe
In the statement, Roosevelt is expressing institution, EXCEPT? C. The right parietal lobe
the ideas of ________. A. Development Bank of the Philippines B. The left parietal lobe
A. an internationalist B. Central Bank of the Philippines D. The right occipital lobe
C. an imperialist C. Philippine National Bank 30. Why does a bullet when discharged
B. an anti-imperialist D. Land Bank of the Philippines into the air eventually fall to the ground?
D. an isolationist 26. The following are legitimate children This is due to the Law of ___________.
EXCEPT? A. Universal Gravitation
A. those born by artificial insemination. C. Inertia
B. Applied Force I. In a vacuum, the rate of accelerator C. Planting grass in gullies to act as a
D. Action and Reaction is the same for all objects regardless of filter
31. The principle under which weight. D. Planting crops in alternate rows
thermostat operates is the same when? II. Outside a vacuum air resistance is (Strip farming)
A. a gas expands to fill the container in what makes different objects fall at 36. You buy a new refrigerator for
which it is held. different rates. P12,800.00 and make a down payment of
B. a pendulum swings when it is set III. Gravity has no effect at all on P2,500.00. If you finance the remainder at
into motion. objects that fall in a vacuum. 8% annually for three years, how much will
C. a chemical reaction occurs when A. I and II you actually pay for the refrigerator?
two substances combine. C. I, II and III A. P12,190.00
D. the level of mercury rises or falls in B. I and III D. P12,772.00
a glass tube. D. II and III B. P10,309.00
32. Which explains the reason why 34. Thousands of street children in large E. P15,272.00
there are continuous and increasing human Brazilian cities were murdered by C. none of these
rights violations? parliamentary death squads which includes 37. 4 1/5 or 4 1/5 + 3 2/7 =
police officers. What could be the reason __________.
A. The United Nation’s General why these operations were not suppressed + 3 2/7
Assembly approved only resolutions on by the government? ________
human rights and the basic freedoms which A. Totalitarian governments do not A. 7 3/12
are not binding give protection C. 7 17/35
B. The solutions used are ineffective. B. The business people even funded B. 7 3/35
C. The United Nations as an these operations to clean up their streets D. 7 1/35
international body is rather slow in the and neighborhoods. E. none of these
exercise of its powers C. The Universal Declaration of Human 38. In which kingdom should MOLD be
D. The United Nations uses as a single Rights was only lip service classified?
solution on all forms of human rights D. To how they treat their people was A. Protista C. Fungi
violations. nobody else’s business. B. Plantae D. Animalia
33. In an experiment, a vacuum is 35. Which of the following procedures 39. Which of the following BEST
created when air is removed from a tube. A used by a farmer is NOT related directly to demonstrates the greenhouse principle?
coin and bits of confetti are released in the preventing erosion? A. A heated aquarium
vacuum at the same time. They fall at the A. Contour plowing around a hill B. A car with rolled-up windows
same rate and reach the bottom at the B. Planting more seeds than are C. A microwave oven
same time. The experiment proves that necessary to yield a bountiful crop. D. A solar battery-powered calculator
40. Which of the following should you B. heat causes the ink to disperse 47. Scientist also find that other stalky
expect to be true about the rate of cellular C. a new compound is formed by the vegetables such as carrots also help lower
respiration for a group of students who are combination of ink and water pressure. This statement is BEST classified
the same age, height, and weight? D. ink molecules have less density than as
A. Athletes would tend to have higher water molecules A. experiment C. finding
rates of cellular respiration than 44. Which Law of force and motion B. nonessential fact D. prediction
nonathletes. explains this occurrence, when a rocket is 48. Each of the following objects is
B. Africans would have higher rates of propelled upward by the powerful designed to employ the buoyancy principle
cellular respiration than Asians. downward discharged of exhaust gases? EXCEPT a
C. Boys would have a higher rates of A. Universal Gravitation A. life preserver C. submarine
cellular respiration than girls. C. Applied Force B. kite D. canoe
D. Nonathletes would have higher B. Action and Reaction 49. What is the difference between the
rates of cellular respiration than Athletes. D. Inertia largest 4-digit number and the smallest 4-
41. Which kingdom should 45. An elderly woman suffered a stroke- digit numbers?
STREPTOCOCCUS be classified? a restriction of blood flow to the brain. if A. 8999 D. 8888
A. Protista C. Fungi the stroke caused to the right side of her B. 8000 E. none of these
B. Plantae D. Monera body to become temporarily paralyzed, she C. 9998
42. What is the function of DIFFUSION in most likely experienced a decreased blood 50. Four mangoes cost P29.00 at that
the human body? flow to price what will 2½ dozen mangoes cost?
A. Regulates blood flow A. the left side of her body A. P217.50 C. P348.50
B. Plays an insignificant role in the C. the left side of her brain B. P188.50 D. P870.00
body’s functioning B. the front of her brain E. none of these
C. Allows an even distribution of D. the right side of her brain 51. Which of the following internal
substances throughout all cells of the body 46.Which of the following methods can all forces interrupt the external forces erosion?
D. Comes into play in times of extreme diabetics control their condition and avoid I. Forces that cause volcanoes
illness heart disease and blindness? II. Forces that cause ocean trenchers
43. In an experiment, a drop of blue ink I. Regulates their intake of glucose III. Forces that cause create mountains
is placed on the surface of a glass of water. II. Increase the levels of insulin in the A. I, II and III C. II and III
In a few minutes, the drop of ink is body by taking insulin injections B. I and III D. I and II
dispersed throughout the water, turning it III. Maintaining a reasonable exercise 52. The How many gallons of water will
light blue. The result of the experiment regimen to keep weight down fill a fish tank that is 18 inches by 12 inches
proves that A. I C. I and II by 48 inches (There are 231 cubic inches per
A. molecules of ink and molecules of B. II D. I and III gallon) Round your answer to the nearest
water are in constant motion gallon.
A. 45 gallons D. 47 gallons B. 60 A. 108 degrees C. none of these
B. 40 gallons E. 37 gallons E. 70 B. 18 degrees D. 123 degrees
C. none of these C. none of these c. 105 degrees
53. What is the sum of all the two digit 57. Employees at Shaira’a Musicmart 61. If a baseball player hits 10 home
numbers which are divisible by 5? get a 20% discount on all purchases. If runs in the first 45 games, at the same rate
A. 945 Teresa buys three tapes at P47.49 each. how many home runs can he expect to hit
D. 1050 How much will she have to pay after her during 162-games season?
B. 950 employee discount? A. 38
E. none of these A. P16.98 C. 36
C. 960 D. P17.98 B. 42
54. How many whole numbers can B. P19.98 D. 40
divide 30 exactly? E. none of these E. none of these
A. Eight C. P18.98 62. Which are the next three terms in
C. Five 58. One package is 100 pounds, and the the progression 1/125, -1/25, 1/5…7 terms?
B. Six other is 150 pounds. The weight of the A. –2,6,-26
D. Four second package is how many times of the C. –4,8,-28
E. none of these first? B. –3,7,-27
55. Which one explains why oxygen, a A. 1 ½ times heavier D. –1,5,-25
gas is the largest component of the Earth’s D. 10 pounds heavier
crust? B. ½ as heavy E. none of these
A. Oxygen gives Earth’s crust its E. 20 pounds heavier
lightness C. none of these 63. How many ways can a committee of
B. Oxygen is the most abundant 59. A carpenter wanted three piece of 4 people be selected from a group of 7
element in the world wood each 1 5/8 feet long. If he planned to people?
C. Oxygen is capable of combining with cut them from a 6-foot piece of wood, how A. 35 D. 210
most of the elements in the Earth’s crust much of the piece would be left? B. 70 E. none of these
D. Oxygen is needed to sustain all life A. 4 3/8 ft C. 140
on Earth C. 4 7/8 ft 64. Which is the length of the
56. To pass the English Test, Lucille must B. 1 1/8 ft hypotenuse of a right triangle with legs 5
get 75% of the items correct. Out of 80 D. 3 ft inches and 12 inches?
questions, how many must she correctly E. none of these A. 17 in. C. 11 in.
answer? 60. How much larger is the supplement B. 13 in. D. 20 in.
A. 55 of a 57 degree angle than the complement E. none of these
D. 65 of a 75 degree angle?
65. Which of the following is the BEST 70. How much topsoil is needed to aside 1/6 of the total land roads. How
example of self-preservation? cover a garden 25 feet by 40 feet to a depth many lots will this tract yield?
A. A mouse runs when it sees a cat. of 6 inches? A. 30 C. 42 B. 35 D. 45
B. A dog barks when it sees its owner A. 480 cuft E. none of these
C. A Man decide to quit smoking D. none of these 75. Find m in the proportion
D. A salmon swims back to the place of B. 440 cuft m/12=30/24.
its birth to lay eggs. E. 460 cuft A. 30 C. 20 B. 15 D. 25
66. A meter was cut at the 35-cm mark. C. 500 cuft E. none of these
What is the ratio of the smaller piece to the 71. A car dealer is offering a rebate of 76. If P75,000 is shared among three
larger piece? P7,500.00 on any new-car purchase. If the children in the ratio of 3:7;15, the size of
A. 7:13 C. 35:100 purchase price of a car is P200,000.00 more the smaller share is?
B. 65:35 D. 65:100 than it was last year. What is the rate of A. P9 C. P25 B. P15 D. P35
E. none of these the discount offered by the rebate? E. none of these
67. The hypotenuse of a triangle is 25 feet. A. 10% C. 7.5% 77. In how many ways can you arrange
If one leg is 24 feet, what is the length of B. not enough information is given three mathematics books (Algebra,
the other leg? D. 13.3% E. 14.2% Geometry, Trigonometry) in order on shell?
A. 6 ft D. none of these 72. In the progression 18, -12,8……which A.6 C. 12 B.8
B. 5 ft E. 7ft C. 20 ft term is 512/729? D. 24 E. none of these
68. Which is the equivalent common A. the 8th C. the 9th 78. Which are the next three terms in
fraction of the repeating decimal B. the 6th D. the 7th the progression 1,4,16…8 terms?
3.242424…? E. none of these A.64,256,1024 C. 66,258,1026
A. 107/33 C. 109/33 73. Which of the following facts support B.67,259,1027 D. 65,257,1025
B. 110/33 D. 108/33 the big bang theory’s explanation of the E. none of these
E. none of these creation of the universe? 79. Which one should be TRUE is
69. Tides, caused by the moon’s gravity, A. The universe does not expand nor Earth’s rotational axis not tilt?
create a fractional force that is gradually contract. I. Days and nights would be the same
slowing down Earth’s rotational speed. One B. The universe seldom expand. length everywhere on Earth.
million years from now, scientist may C. The universe will have background II. There would be no hours of
discover that compared to today, Earth’s radiation. darkness on points along the equator
A. day is longer C. day is D. The universe has no beginning nor III. Earth would have no seasons.
shorter end. IV. Each part of the Earth would have
B. year is shorter D. year is longer 74. Mr. Garcia owns a 10½ hectares the same daily temperature pattern
tract of land. He plans to subdivide this A. II and IV
tract into ¼ hectare lots. He must first set C. IV and I
B. III and IV B. mabisa D. makapangyarihan 88. What is meant by the expression TO
D. I and III 84. The stanza below is taken from “Barter” GET BLOOD OUT OF A STONE in this
80. The carat is a unit of measure used by Sara Teasdale sentence?
to weight precious stones. It equals 3.086 Life has loneliness to sell, Geraldo has owned me fifty
grains. How many grains does 2.8 carat Music like a curve of gold, thousand pesos for over a year now. I‘ve
diamond weigh? Scent of pine trees in the rain, asked him for it on several occasions, but
A. 864.08 C. 8.6408 Eyes that love you, arms that hold, it’s like trying to get blood out of a stone.
B. 86.408 D 5640.8 And for your spirit’s still delight, A. Something is impossible.
E. none of these Holy thoughts that star the night. B. Someone refuses to cooperative.
81. Robert Frost wrote the poem To what does Teasdale compare C. Someone is willing to give what is
“Acquainted with the Night” from which the music? asked.
stanza is taken: A. The scent of pine trees D. Someone wants revenge.
I have been one acquainted with the night. C. Eyes that love 89. What correction should be made to this
I have walked out in rain-back in rain. B. A curve of gold sentence? First born often pattern their
I have outwalked the farthest city light. D. The rain behavior after they’re parents and other
The poet in the stanza talks of 85. Which word ends with [S] pronounced adults.
A. isolation and loneliness. [Z]? A. Replace their to they’re
B. happiness in having been acquainted A. Maps C. Laughs C. Replace they’re to their
with the night. B. Jokes D. Buys B. Change pattern to patterned
C. joy getting out of the house. 86. Which of the following lines is a simile? D. No correction is necessary
D. youthful delight playing in the rain. A. “Holding wonder like a cup” 90. What is meant by SOFT OPINION in this
82. What is meant by AT SIX AND SEVENS in C. “Eyes that love you, arms that hold” sentence?
this sentence? B. “Life has loneliness to sell” Rebecca realized that if she stayed in her
We moved into the house last week, D. “Buy it ang never count the cost” present job it would mean competing with
but I’m afraid everything is still at six and 87. Which is the BEST WAY to write the an envious rival. Leaving the company
sevens. underlined portion of this sentence? A would probably be a soft option
A. The things have not been shipped person should keep in mind some basic A. An action that is difficult to take
C. In an orderly manner safety rules when you are deciding whether C. An action that is easier
B. In a state of confusion or not to use a fire extinguisher. B. An action that is not agreeable
D. The boxes are still intact A. Rules you decide D. An action that is weakly funded
83. Sa “Espiritu ni Bathala ang nangangalaga C. Rules you are deciding 91. What correction should be made to this
ng kanilang kalusugan” ang ipinahihiwatig B. Rules when you decided sentence? Most State tourism departments
na katangian ay _______. D. Rules when deciding and some travel agencies have bed and
A. malinis C. maliksi breakfast listings.
A. Insert a comma after agencies extinguishers can actually help people fight However, their VCR kept them from
C. Change tourism to Tourism fires. missing their favorite prime time shows.
B. Change have to has A. They’re is C. Their are A. Keepes C. Had kept
D. Change State to state B. Their is D. There are B. Keeps D. Keeped
92. What is suggested in the opening line? 96. What do the following lines 101. Which is the BEST way to write the
June 13, 1986-they came from all over CONVEY? underlined portion of this sentence?
America- 200,000 heroes strong, with their Midnight, not a sound from the pavement. Researchers also speculate that some
families. Has the moon lost her memory? teachers might have given boys more
A. The writer holds great admiration She is smiling alone. computer time because parents and
for the veterans In the lamp light the withered leaves teachers expected boys to need computers
B. The writer was a veteran of the war Collect at my feet for future careers.
C. The writer is opposed to the And the wind begins to moan. A. Will expect C. Will have expected
Vietnam War A. Confusion C. Loneliness B. Expected D. Expecting
D. The writer is a flag-waving patriot B. Optimism D. Eagerness 102. Which word contain the [ae] sound?
93. To gain the attention of the audience, 97. What correction should be made to A. Carriage C. Castle
the trick is __________. this sentence? B. Cabin D. Can
A. start low, speak hurriedly Most fire-related death’s result from 103. What correction should be made to
C. start high, speak rapidly households fries, yet many people do not this sentence?
B. start high, speak loudly have fire extinguishers in their homes. One of their theories is that the first child
D. start low, speak slowly A. Remove the comma after fires receives more of the parents’ attention
94. What correction should be made to this C. Change have to has than other children so first-borns tend to be
sentence? Recently, educators exammined B. Change result to results more intellectual.
the effectiveness of computer instruction in D. Replace death’s with deaths A. Change is to are
schools. 98. Which verb in the sentence is C. Change parents to parent’s
A. Replace educators with educator’s pronounced with the ending as [d]? B. Insert a comma after children
B. Change the spelling of exammined to They laughed and joked as they walked and D. Change theories to theory’s
examined played. 104. What is meant by LAST DITCH in this
C. Change schools to Schools A. Joked C. Walked sentence?
D. Replace computer with computer’s B. Laughed D. Played The aged bishop prepared to fight to the
95. Which is BEST WAY to write the 99. Which of the following words DOES last ditch to defend his good name.
underlined portion of this sentence? NOT contains the [voiceless th]? A. One’s last courage C. One’s last hope
There is smoke detectors in many homes to A. Mouth C. Teet B.Breath D. Health B. One’s last strategy D. One’s last defense
warm residents of a fire, but fire 100. Which is the BEST way to write the 105. Which is the BEST way to write the
underlined portion of this sentence? underlined portion of this sentence?
Recently psychologists have been B. These D. Thing
researching birth order, their research 110. Which word is read on a high note A. Choose the goals and means of
suggests that personality and intelligence to describe the kind of day? development
are based partly on where a child ranks in This is a cold day. B. Share in scientific and technological
the family. A. Is C. Day advances of the world
A. Order, Their C. Order, or their B. Cold D. This C. Right to information
B. Order and their D. Order, their 111. What is the mood of these lines? D. Sovereignty over our natural
106. What is meant by TWO PINS in this Daylight, I must wait for the sunrise resources
sentence? I must think of a new life 115. Which antidote would have a similar
For two pins I could have hit him on the And I mustn’t give in. effect if vinegar or citrus juice were not
nose. When the dawn comes available?
A. A second course of action Tonight will be a memory, too A. Milk C. Vegetable oil
C. The second chance And a new day will begin. B. Raw egg white D. Water
B. Without much persuading A. Afraid C. Depressed 116. Which method of reproduction
D. Have a second alternative choice B. Sarcastic D. Hopeful provides for the most variety of offspring?
107. Which is the BEST way to write the 112. Which is the BEST way to write the A. Cloning C. Asexual reproduction
underline portion of this sentence? underlined portion of this sentence? B. Sexual reproduction D. Cellular
The hosts also benefit from running such a However, if a fire extinguisher is handy, a reproduction
business because they can stay at home quick-thinking person often can use them to 117. Spouses Jose S. Luz and Celerina Luz
make money, and meeting a variety of put out small fire. filed a petition to adopt Gregorio Luz Ona,
people. A. They C. Them their nephew. The spouses are childless
A. Get to meet C. And meet B. Him D. It and they reared from his birth 1971 until
B. To meet D. And be meeting 113. What correction should made to the 1975 and they continue to support him.
108. Which is the BEST way to write the sentence? Gregorio had to be left in the Philippines
underlined portion of the sentence Buying fire extinguisher knowing how to use when the spouse went to the United States
The studies revealing that, for various it, and placing it in a location familiar to all where Jose is employed. The MSSD
reasons, girls spent less time working with family members can help protect families recommended the adoption to the court on
computers than boys. against fire. the premise that petitioners are in a better
A. Revealing studies A. Insert a comma after extinguisher position to provide for the minor child than
C. Studies’ revelations B. Change the spelling of families to familys the natural parents who are impoverished.
B. Studies revealed C. Change placing to place What could be the possible decision of the
D. Studies will reveal D. Insert a comma after help court on he petition?
109. Which word contains the voiced Th? 114. Which one is the right to human A. The court denied the petition
A. Thank C. Think dignity? because the spouses are already aliens.
B. The petition for adoption was 121. Carolina Diaz filed a petition for D. The relationship between virtue and
granted because the court finds that it is to habeas corpus against Mr. and Mrs. Ramon conduct
the best interest of the child. Alde to recover custody of Lina Diaz Tan 124. Which of the following ethnocentric
C. The petition for adoption was alias "Gracia Alde,” the natural daughter of behavior?
denied because the spouses are non- Carolina Diaz, who was a hostess. A. A tourist who lectures his foreign hosts
residents of the Philippines. What could be the possible action of the on the “uncivilized” nature of their marriage
D. The court denied the petition on the court on the petition filed by Carolina Diaz? customs
premise that the trial custody required by A. Her petition would be granted B. A student who tutors an immigrant in
PD 603 cannot be effected for spouses are because she now works as a clerk in a English
non-residents. prestigious office. C. A Hispanic community group demands
118. The undeclared war in Korea most B. Her petition would be denied that public aid forms be published in
closely resembled the situation of because she was a former hostess. English and Spanish
A. The Spanish-American War C. Her petition would be denied D. A peace Corps volunteer who helps dig
C. World War I because when Gracia was given to the Aldes wells in Central Africa
B. Vietnam War it was tantamount to abandonment of the 125. Which one is the right to human
D. World War II child, resulting to termination of parental dignity?
119. Which is the MOST important authority. A. Political independence
perceived need and problem of the Filipino D. Her petition would be granted B. Honor and reputation
family? because she is the natural mother. C. Form association
A. Unemployment or financial problem 122. Why did the register of deeds charge D. Social and economic reforms
B. Proveness to vices Lucio Cruz registration fee the instrument 126. Where would you commit a drug
C. Protectiveness of children relative to his loan? dependent for him to achieve a natural,
D. Double standard on the roles of male A. His loan was 30,000.00 tensionless, and anxiety-free state?
and female C. His loan was P50,000.00 A. NFPI C. DARE
120. Which has become a prototype of B. His loan was P60,000.00 B. DARN D. Bukang Liwayway Center
other schemes that defeated the real and D. His loan was P40,000.00 127. Mary Rose, an 18 year old was sexually
true purpose of the CARL? 123. Which one BEST defines personal abuse by 3 teenagers from well-to-do
A. The conversion of farmlands to industrial integrity? families from Makati. Despite pressures,
complexes A. The unity between ignorance and she came out into the open to get justice.
B. The stocks option scheme of Hacienda reality Which need did Mary Rose satisfy?
Luisita B. The unity of man’s deeds, words, A. Need for family unity
C. The conversion from agriculture to thoughts and realities C. Need for universal solidarity
subdivision C. The unity of mans social, political and B. Need for civic responsibility
D. The voluntary offer to sell physical aspects D. Need for personal integrity
128. 10 A. P 171.15 D. P 159.95 D. The presence of certain bacteria in
- 3 2/7 or 10-3 2/17= B. P 139.25 E. P 148.75 the digestive tract causing
__________ C. none of these methemoglobinemia
A. 7 2/17 C. 6 1/17 133. Which location will have most nearly 137.Without the process of meiosis, we can
B. 6 2/17 D. 6 15/17 twelve hours of daylight and twelve hours infer that offspring from sexual
E. none of these of darkness during December? A town that reproduction would
129. At which time during the year does the is located A. have a high degree of genetic variety
ozone level present a particular health A. halfway between the equator and South B. have twice assigned number of
threat in urban areas for people with Pole chromosomes
respiratory problem? B. close to equator C. be identical
A. Spring C. Summer C. close to the North Pole D. have a number of mutations.
B. Fall D. Winter D. close to the South Pole 138. The Jones family has four children, all
130. What day follows the day before 134. A nation in which loess would likely to girls. The fifth child born is a boy. This
yesterday if 2 days from now will be be found today is change is the result of
Sunday? no answer A. conception classes takes by the
A. Tuesday D. Wednesday A. Iceland C. Japan parents
B. none of these E. Thursday B. United States D. Ecuador B. the timing of fertility cycles.
C. Saturday 135. It was also discovered that the C. the father’s contribution of a “Y”
131. Which is the BEST evidence that chemical 3m butylphthalide can lower the chromosomes.
helium gas is lighter than air? blood pressure of rats. This statement is D. the “law of averages” finally catching
A. Helium has the lowest boiling point of all classified as ________. up.
elements. A. prediction C. finding 139. Which of the following will occur if a
B. Helium atoms do not combine with other B. experiment D. nonessential cold bottle of soda is left open on a kitchen
air atoms. fact counter?
C. Helium-filled balloons rise in air. 136. Which is NOT among the hazardous A. The pressure that the soda exerts on
D. By volume, helium makes up only effects of water pollution to health? the bottle will increase.
0.0005% of air. A. The epidemic threat of hepatitis and B. The temperature of the soda will
132. During a recent shopping spree, Tomas dysentery decrease.
and Nena bought some new accessories for B. The increase incidence of liver C. The amount of dissolved carbon
their apartment. Nena choose a crocheted cancer dioxide gas will decrease
throw pillow at P24.95, and Tomas C. The dumping of mercury in the sea D. The amount of dissolved carbon
purchased a rural landscape painting for causing blindness, brain damage or death dioxide gas will remain the same.
P135.00. How much did they actually spend
if they paid 7% sales tax on their purchases?
140. What do you predict will happen when 144. Who is the speaker of this poem? Now the trumpet summons us
you bring two bar of magnets closer A. Tom Merritt C. God again-not as a call to bear arms, though
together? B. Merritt’s wife D. The sheriff arms we need, not as a call to battle,
A. They will repel each other. 145. The way in which the poet present though embattled we are; but a call to bear
B. They will create an alternating current. these words in line 12 implies that Tom the burden of a long twilight struggle, year
C. Noing will happen. A. tried to annoy the other man. in and year out, “rejoicing in hope, patient
D .They will attract each other. B. was shot before he finished the in tribulation,” a struggle against the
141. A tightly coiled spring demonstrate? statement common enemies of many tyranny,
A. Steam energy C. Potential energy C. did not want to hurt the other man. poverty, disease, and war itself.
B. Kinetic energy D. Chemical energy D. begged the man to stop seeing his Can we forge against these enemies
142. What does a stick of unlit dynamite wife. a grand and global alliance, North and
demonstrate? 146. The poet introduces the poem with South, East and West, that can assure a
no answer lines 1-3 to show us that Tom Merritt more fruitful life for all mankind? Will you
A. Chemical energy C. Kinetic Energy A. had suspected that his wife was join me in this historic effort?
B. Nuclear Energy D. Potential Energy seeing another man In the long history of the world, only
143. Which is the main goal of drug abuse B. was sure that his wife was ill a few generations have been granted the
education? C. was a very suspicious person role of defending freedoms in its hour of
A. Arrest C. Control D. was sure that his wife still loved him maximum danger. I do not shrink
B. Prevention D. Rehabilitation 147. Which of the following techniques is responsibility; I welcome it. I do not believe
Situation 1 – Below is the poem written by used in the poem? that any of us would exchange places with
Edgar Lee Masters in 1915: A. Verse C. Free verse any other people with any other people or
At first I suspected something __________ B. Rhyme D. Personification any other generation. The energy, the faith,
She acted so calm and absent-minded. Situation 2 – below is an excerpt from John the devotions which we bring to this
And one day I heard the back door shut, F. Kennedy’s “Inaugural Address.” Read the endeavor will light our country and all who
As I entered the front, and I saw him slink excerpt and answer the questions that serve it, and the glow from that fire can
Back of the smokehouse into the lot,And follows truly light the world.
across the filed. In your hands, my fellow citizens, And so, my fellow Americans, ask
And I meant to kill him on sight. more than mine, will rest the final success not what your country can do for you; ask
But that day, walking near Fourth Bridge, or failure of our course. Since this country what you can do for your country.
All of a sudden I saw him standing, as founded, each generation of Americas My Fellow citizens of the world, ask
Scared to death, holding his rabbits, has been summoned to give testimony to not what America will do for you, but what
And all I could say was, “Don’t, Don’t its national loyalty. The graves of young together we can do for the freedom of man.
Don’t,” Americans into answered the call to service 148. One of the purposes of the speech is to
As he aimed and fired at my heart/ surround the globe. motivate listeners to
A. serve their country night school to become an electrician. That B. increasing and decreasing finances
C. prepare themselves for battle year, at Sun Records, Presley recorded a C. successes and disappointments in his
B. enlist the armed forces personal record for his mother, a song that career
D. preserve the right to bear arms was heard by the company’s president. As a D. use of drugs, “uppers and downers”
149. The speech is characterized by all of result of the president’s recognition 153. The main idea of the sketch is that
the following stylistic devices EXCEPT Presley’s first record “That’s All Right A. singers are more successful if
A. the use of the personal pronouns we Mama,” was out in 1954. they appear in films
and us to build rapport with listeners He toured the South, and in 1955 five of B. there has always been obscenity on
B. catchy turns of phase in which his record were released simultaneously. television
subjects and objects are inverted His first national television appearance was C. opportunity and luck are often as
C. a standard, predictable rhythm and that year of Jackie Gleason’s “The Stage important as hard work
the use of rhyme Show.” But Presley became known for his D. celebrities are usually more
D. the repetition of key words appearance on “The Ed Sullivan Show,” famous after their death
150 The tone of the speech can BEST be where young singer gyrated as he sang 154. The last sentence reveals that the
characterized as “rock n’ roll” music. During the live author’s attitude toward Presley
A. sad C. light-hearted television performance, Presley wad is one of
B. uplifting D. sarcastic photographed only from the waist up A. indifference C. disbelief
151. In the speech, Kennedy paints a picture because his motions were considered B. admiration D. disgust
of the United States as a nation that is obscene. 155. The statement that ”success often
A. longing to return to the past “Elvis the Pelvis” began his film comes to those with humble beginnings”
C. struggling to survive career in 1956 with LOVE ME TENDER and would apply best to which of the following
B. on the brink of world war signed a long-term film contract. The movie figures?
D. the leading defender of freedom critics were not always kind, but teenagers A. Ramon Magsaysay
Situation 3 – Below is a Boigraphical sketch flocked to Presley’s films. Within a few C. Corazon C. Aquino
of an American movie writer: short years, Presley had established a B. Gloria M.
How did Elvis Presley Achieve Recognition career that would span twenty-five years of D. Joseph Estrada
Success often comes to those with ups and down and make him one of the Situation 4- The poem below is entitled
humble beginnings. Elvis Aaron Presley was most popular entertainers in history. Long “Suburban Prophecy” which is
born on January 8, 1935 in Tupelo, after his untimely death at age 42, Presley written by Howard Nemerov
Mississippi. He first sang in a church anf would be remembered as “The King of Rock On Saturday, the power-mowers’
taught himself to play the guitar, but he n Roll.” whine
never learned to read music. By 1953, he 152. The author uses the phrase ups and Begins the morning. Over this
had moved to Memphis, Tennessee, downs to refer to Presley’s neighborhood
graduated from high school. And enrolled in A. gyrations as performed
Rises the keening, petulant voice, A. sight C. smell Rizal, ngunit kung ang ating kalooban
begin B. touch D. hearing nama’y itim na duwende ng kasamaan
Green oily teeth to chatter and 159. How long does the action of poet take anong kapinsalaan para sa kinabukasan.
munch the cud. place? Kinabukasan, kabataan, tayo raw
Monster, crawling the carpets of the A. A week C. An Afternoon ang pag-asa ng inang Bayan. Tayo raw ang
world, B. Twenty-four hours D. A morning maghahatid sa kanya sa lagnit ng kaganaan
Still send from underground against your Situation 5 – Ang sumusunod ay isang at karangalan o hihila sa kanya sa putik ng
blades talumpati na may pamagat na SA kahirapan at kahihiyan. Ang panahon ng
The roots of things battalions greens and KABATAAN na isinulat ni Onofre pagkilos ay ngayon, hindi bukas, hindi sa
curled Pagsanghan isang taon. Araw-araw ay tumatawid
And tender, that will match your Isa sa mga salitang napag-aralan tayong palangit o bumabaluktod tayong
blades with blades natin sa wikang Pilipino ay salitang paputik. Tamang-tama ang sabi ng ating
Till the revolted throats shall NABANSOT. Kapag ang isang bagay daw ay mga ninunong kung ano raw ang
strangle on dapat pang lumaki ngunit ito’y tumigil na sa kinamihasnan ay siyang pagkakatandaan.
The tickle of their dead, till straws paglaki, ang bagay na ito raw ay Huwag nating akalaing makapagpapabaya
shall break NABANSOT. Marami raw uri tayo ng ating pag-aaral ngayon at sa araw
Crankshafts like camels, and the sun go ngpagkabansot ngunit ang pinakamalungkot ng bukas ay bigla tayong maging mga
down na uri raw ay ang ng isipan, ng puso at ng dalubhasang magpapaunlad sa bayan.
diwa. Huwag nating akalaing makapaglulublob
On dinosaurs in swamps. And night Ang panahon ng kabataan ay tayo ngayon sa kalaswaan at kahalayan at
attack panahon ng pagklaki, ngunit ang ating sa mahiwagang araw ng bukas bigla tayong
Follows and by the time the Sabbath dawns paglaki ay kailangang paglaki at pag-unlad magiging ulirang mga magulang.
All armored beasts are eaten by their lawns. ng ating buong katauhan. Hindi lamang ng Kabataan, tunay na pag-ibig sa
156. To what does the phrase “your blades” atinmg sukat at timbang. Kung ga-poste bayan, ang tunay na nasyonalismo, ay wala
in line 8 refer? man ang ating at ga-pison man angating sa tamis ng pagnarap wala rin sa pagpag ng
A. Lawmowers C. Carpets bigat ngunit kung ang pag-iisip namat nati’y dila. Ang tunay na pag-ibig ay nasa pawis
B. Roots D. Monster ga-kulisap lamang kay pangit na ng gawa.
157. The poet’s use of words such as whine, kabansutan. Kung tumangkad man tayong 160. Alin salita ang paulit-ulit na binabangit
voice, teeth, chatter and munch is to tangkad-kawayan at bumilog man tayong ni Onofre Pagsanghan?
suggest that the power-mowers are bilog-tapayan, ngunit kung tayo nama’y A. Nabansot C. Kabataan
A. very powerful C. like cows tulad ni “bondying” ay di pagkatiwalaan B. Bayan D. Kung
B. alive D. green anong laking kakulangan. Kung 161. Sa alin makikita ang tunay na
158. The imagery in the first stanza appeals magkakatawan tayong katawang “Tarza” at NASYONALISMO?
to the reader’s sense of mapatalas ang ating isipang sintalas ng kay A. Diwa C. Sulat
B. Gawa D. Salita C. Kung ano ang ibig natin ay C. Pagpapatuloy ng buhay
162. Bakit di dapat tumangkad tulad ni mangyayari B. Malapit ng mamatay
“bondying”? D. Magtiis kung dumarating ang hirap. D. Mahirap na buhay
A.Di ito mapagkakatiwalaan 166. Sa aling salita magkakaroon ng saglit 171. Kaninong tula hango ang sumusunod?
C. May kakulangan ito na paghinto kung pinapilitang si Rose ang “Ang hindi magmahal sa sariling wika
B.Di totoo ito D. Magulo kasi ito nakabasag ng pinggan? mahigit sa hayop at malansang isda”
163. Alin sa mga sumususnod ang mensahe A. Jose Rizal C. Apolinario Mabini
ng taluimpati? Hindi si Rose ang nakabasag ng pinggan. B. Emilio Jacinto D. Graciano Lopez Jaena
A. Ang mataas na paniniwala at A. Rose C. Hindi 172.Ano ang pinakaangkop na kahulugan
taimtim na pananalig ay B. Pinggan D. Nakabasag nito?
kailangang taglayin upang ang hangarin sa 167. Alin ang naayong pamagat sa tanagang “Nagsasaya tayo ngayon sapagkat ang
buhay ay ating kamtin. sinulat ni Jose Villa Panganiban? inyong namatay na kapatid ay muling
B. Ang panahon ng kabataan ay Ano man sa daigdig, nabuhay; ang nawawala ay muling nakita.”
panahon ng paglaki at Maaring magamit, A. Ang pagbabalik ay dapat ipaghanda
pagbabagong makabuluhan. Ano mang masaisip; nang malaki.
C. Ang gawa ang siyang sukat ng Di sukat maiipit. B. Ang pagbabago ng kapatid ay dapat
kadakilaan. A. Pagkainip C. Pag-asa pahalagahan.
D. Ang kabataan ay siyang pag-asa ng B. Paraya D. Pagbibigay C. Dapat silang magsaya sa muli nilang
bayan. 168. Ano ang ipinahihiwatig ng salitang may pagsasama-sama
164. Anong tayutay ang tinutukoy nito? salungguhit? D. Ang pagsasama nila ay dahil sa
Durog ang katawang bumagsak sa semento Matayog ang lipad ni Pepe kaya’t bata pa muling pagbabalik ng kapatid.
si Miguel siya’y nagsisikap na siya. 173. Alin ang kahulugan ng KAHIRAMANG
A. Pagtutulad C. Pagmamalabis A. May kayabangan si Pepe. SUKLAY?
B. Pagbibigay katauhan D. C. Marunong si Pepe. A. Kakilala C. Karibal
Pagwawangis B. Mataas ang pangarap ni Pepe. B. Kaibigan D. Kalahi
165. Anong aral ang ibinibigay ng D. Ibig ni Pepeng maabot ang langit. 174. Alin sa mga sumusunod ang aral na
sumususnod na salawikain? 169. Alin antas ng tono ng lumilitaw sa ibinibigay ng epikong Muslim na
“Ang taong napapailalim ay naipapaibabaw bahaging may salungguhit ng pangungusap INDARAPATRA AT SULAYMAN?
rin.” na nagdududa? A. Pagmamahal
A. Maaring ngayon ay hirap pagdating Nagpuputol ng puno ang lalaki. C. Katapatan
ng bukas ay may ginhawa rin A. 1 C. 4 B. Katapangan
B. Tiyak ang pag-unlad kapag nauna ang B. 2 D. 3 D. pagtanaw ng utang-na-loob
hirap 170. Alin ang kahulungan ng AGAW-BUHAY? 175. Alin sa mga salita ang kasingkahulugan
A. Masiglang-masigla ng salitang may salungguhit?
Ang ama ni Anita ay kilalang bulanggugo sa 180. Which word in the passage does ”Mag-aral sa bahay ng mga araling ukol sa
kanilang lalawigan NOT require a change in pitch to show halaman.”
A. Laging ibinubulong confidence A. Pawatas C.
C. Laging handang makipag-away “I am the master of my fate, I am the Imperpektibo
B. Laging handang gumasta captain of my soul.” B. Kontemplatibo D. Perpektibo
D. Laging handang makipagtalo A. Am C. Master 186. Kilalanin ang uri ng pariralang may
176. Si Mariano Ponce ay propagandistang B. Captain D. Fate salungguhit.
may sagisag sa panulat na ________. 181. Which pitch is used for the word Ang pangangalaga sa mga likas na
A. Tamaraw C. Kapre STRANGE in this sentence? yaman ay tungkulin nating lahat.
B. Tikbalang D. Kalapate What a strange story! A. Pangngalan C. Pang-ukol
177. Alin ang di karaniwang anyo ng A. 3 C. 4 B. Pangngalang-diwa D. Pawatas
pandiwang HINTAY KA? B. 2 D. 1 187. Si Dr. Jose Rizal ay sumulat ng aklat
A. Tay C.Intay 182. Carl Sundburg wrote “Jazz Fantasia” ng itinampok sa ibat ibang bansa.
B. Tayka D.Teka which has for its first stanza: Ang pangungusap ay nagagamit bilang
178. Ano ang kahulugan ng taludtod na Drum on your dreams, better on your bajos, _______________.
ito sob On the long cool winding A. panuring C. tuwirang layon
“Ang anak mo ay alagaan sa marubdob na saxophones.Go to it, O jazzmen. B. pamuno D. paksa
pagsuyo sikapin mo sa sarili’y huwag siyang Which words illustrate alliteration 188. Lines 11 and 12 are taken from the
maging luko talipandas sa paglaki na sa A. Batter and banjos C. Sob and winding poem
sama marahuyo sa lahi mo’t sa Bayan moy B. Long and cool D. to and it “maggie and milly and molly and may.”
isang tinik sa balaho.” 183. Anong uri ng panghalip ang salitang For whatever lose (like a you or a me)
A. Mahalin ang anak ng walang may salungguhit sa pangungusap? It’s always ourselves we find in the sea
hangganan. May padalang tulong ang pamahalaan para Which of the following ideas is the author
B. Tamang pagpapalaki sa anak ang sa kanila. expressing?
dapat. A. Pambalana C. Paari A. The sea is a source of life and death.
C. Suyuin ang anak at ibigay lahat ng B. Palagyo D. Palayon B. The sea represents all of our moods.
hilig. 184. Alin uri ng parirala ang may C. The sea is the best place for a
D. Paligayahin ang tahanan. salungguhit sa pangungusap? person to reflect about life.
179. Which is the BEST way to write the Utang sa kanyang sipag at sikap sa paggawa D. The sea and its surroundings can
underlined portion of this sentence? ang kanilang maalwang pamumuhay give people a fresh view on life.
Many viewers taped shoes to watch later. A. Pangngalan C. Pawatas 189. Nasa anong kaganapan ng pandiwa
A. Tapped C. Tape B. Pangngalanng-diwa D. Pang-ukol ang pangungusap?
B. had taped D. Had tapped 185. Sabihin ang aspekto ng pandiwa sa Naglaro ng basketball sa Rizal Stadium ang
pangungusap na ito. koponan ng aming pamantasan.
A. Sanhi C. Kagamitan he intended. How many people does he D. P 2.06 per lb
B. Tagaganap D. Ganapan actually reach B. none of these
190. Alin antas ng tono ang lumitaw sa A. 16,000 E. P 2.30 per lb
bahaging may salungguhit ng pangungusap C. 24,000 C. P 2.50 per lb
na nagsasalaysay? Magbabasa ng mga B. 10,000 197. Which is the sum of the infinite
gawain ang guro D. 36,000 progression 3/2. 1, 2/3, 4/9…?
A. 1 C. 2 E. none of these A. 6 1/2
B. 3 D. 4 194. Alin sa mga sumusunod ang C. 4 ½
191. Dadalaw sa mga paaralan si Dr. mensahe ng epiko ng Ilokano na BIAG NI B. 5 ½
Filemon S. Salas, ang tagapamanihala ng LAM-ANG? D. 7 ½
mga paaralang lungsod, sa lungsod ng A. Pinatutunayan ng epiko ang yaman ng E. none of these
Pasay. Ilokano sa lahat ng bagay. 198. What indoor relative humidity range
Ang pangungusap ay nagagamit bilang B. Kailangan paniniwalaan ang ukol sa bisa would probably be comfortable to
_____________. ng mga anting-anting dahil sa mga preventing temperature and humidity levels
A. panuring C. tuwirang layon pangyayaring nagpapatunay dito. are extremely low?
B. paksa D. pamuno C. Dito nagpapatunay na walang A. 90% to 100 %
192. Alin sa mga sumusunod ang aral na kamatayan. C. 20% to 30 %
ibinigay ng ANG ALAMAT NI MARIANG D. Masasalat ang mga katutubong ugali at B. 60% to 70 %
MAKILING na ikinuwento ni Jose Rizal? mga tradisyong dapat pagyamanin at D. 30% to 40 %
A. Pagyamanin at pangalagaan ang ating panatilihin upang pakinabangan ng 199. As a representative of the Urban
bayan at lahi pagkat hiyas at yaman natin kabataan. Poor Commission of the Association of
ito. 195. Alin ang di karaniwang anyo ng Religious Superiors (ARS), which action will
B. Pag-ibig ang makapagbabago sa pandiwang WINIKAKO? you most likely take to resolve the long-
mundong ito. A. Ikako term roots of structural inequalities-
C. Kabanalan ang magpatawad at tulungan C. Kako proliferation of child labor and child
ang isang nagkasala. B. Wikako prostitution?
D. Dahil sa pagmamalabis at D. Wika ko A. Raise views of human rights abuse.
pagsasamantala, maraming biyaya ang sa 196. The Miranda Family purchased a B. Organize regular programs for
kanyay nawawala. 250-pound side of beef and had it information and discussion of human rights.
193. A Politician wants to get his message packaged. They paid P365.00 for the side C. Conduct skills training.
to 2/3 of the population of 48,000 beef. During the packaging, 75 lb of beef D. Raise questions over government’s
in Bulacan. However advertising campaign were discarded as waste. What was the cost commitment to rebuild human rights.
reaches only 3/ 4 of the number per pound for packaged beef 200. What are the next four numbers in
A. P 2.08 per lb this sequence 8,5,4,9,17_____,_____,____
A. 4,3,2,1 D. none of these
B. 5,4,3,2 E. 3,2,1,0
C. 6,3,2,0

13. C 63. A 113. A


163. D
ANSWER KEY
GENERAL EDUCATION 14. D 64. B 114. D
164. C
Secondary
15. C 65. D 115. A
1. C 51. B 101. A 165. A
151. D
16. C 66. A 116. B
2. A 52. A 102. B 166. C
152. C
17. B 67. E 117. C
3. B 53. A 103. B 167. C
153. C
18. C 68 A 118. B
4. D 54. A 104. D 168. B
154. B
19. D 69. A 119. A
5. D 55. C 105. A 169. C
155. A
20. B 70. C 120. A
6. B 56. B 106. C 170. B
156. NO ANSWER
21. C 71. E 121. D
7. C 57. D 107. C 171. A
157. NO ANSWER
22. A 72. C 122. NO
8. B 58. A 108. B ANSWER 172. B
158. NO ANSWER
23. A 73. A 123. D
9. A 59. B 109. A 173. B
159. NO ANSWER
24. C 74. B 124. C
10. C 60. A 110. B 174. A
160. C
25. B 75. B 125. B
11. D 61. C 111. D 175. B
161. A
26. D 76. A 126. C
12. A 62. D 112. D 176. B
162. C
27. D 77. A 127. D 39. A 89. C 139. C
177. A 189. D

28. C 78. A 128. D 40. A 90. C 140. B


178. B 190. B

29. B 79. D 129. C 41. D 91. D 141. C


179. C 191. B

30. A 80. C 130. E 42. C 92. A 142. ?


180. D 192. D

31. D 81. A 131. C 43. A 93. B 143. B


181. A 193. C

32. B 82. A 132. A 44. C 94. B 144. D


182. C 194. D

33. A 83. D 133. B 45. C 95. D 145. A


183. C 195. C

34. B 84. B 134. ? 46. A 96. C 146. A


184. A 196. A

35. A 85. D 135. C 47. C 97. D 147. C


185. B 197. E

36. E 86. A 136. C 48. B 98. D 148. A


186. B 198. ?

37. C 87. D 137. B 49. A 99. D 149. B


187. D 199. D

38. C 88. A 138. C 50. D 100. B 150. B


188. D 200. D
true size and shape of the earth’s land address this problem they want to achieve
masses? equal death rate and birth rate in
A. Azimuthal Mapping population. This government action to limit
C. Equal Area Mapping the population is known as the
B. Conformal Mapping A. Baby Boom C. Population pyramid
D. Equidistant Mapping B. Population distribution
PART II: ANALYZING TEST ITEMS 3. Most scientists believe that El Niño D. Zero population growth
Direction: Encircle the letter of the best phenomenon which was greatly 5. Given the current condition of
answer. experiences in 1982, 1983 occurred when Africa, it is still considered as the most rural
1. The following statements refer to trade winds that normally blow from east to and least urbanized continent in the world.
the characteristics of the Philippines as one west slow down and actually change With these characteristics, which among
of the important territories of Asia, except direction. The winds blow warm water the following sentences is the least
for one toward South America where the normal contributory factor to the current socio-
A. It is a tropical country and weather patterns are changed for a time. political and economic problems of Africa?
geographically located in the northeastern Given this global change, which among the A. Few states are ethnically
part of the world. following conditions is not considered as homogenous and infancy of a strong sense
B. Philippines has one of the highest direct effect of the El Niño phenomenon? of national unity among tribal leaders
literacy rates in Asia with existing policy of A. Occurrences of hurricanes and B. Insufficient capital technology,
free and compulsory six years in heavy rainfall in the mid-Pacific region in a political instability and poorly trained
elementary. normal condition workforce to push development among the
C. The establishment of the first B. Some species of fish like cold-water African nations
Philippine Republic gave birth to the first loving salmon disappeared along the west C. The increasing percentage of school-
constitutional democracy in Asia coast of the United States but tuna and age children who did not attend school low
D. The free port of Manila is one of the shrimp were attracted to the warm waters literacy rate and a grate number of world’s
world’s busiest ports, compared with other C. The gradual warming of the earth is AIDS deaths
Southeast Asian nations known as the Greenhouse effect could D. The replacement of Christianity
2. Globes are essentially important dramatically change climates worldwide among the Islamized society in Africa along
tool in the study of Geography. A map is D. Water temperature in some parts of with the continuous practice of animism
more convenient to use but all map the Pacific Ocean rose to as much as 14 among few villages
projections have some errors in degrees higher than to normal condition 6. The biggest and most populated
presentation of distance and shapes. Given 4. China has over one fifth of the island in the Philippine Archipelago
these conditions, which among the known world’s population and its leaders believe A. Cebu D. Mindanao
mapping projections will we use to give the that its large population hinders the B. Leyte E. Negros
economic progress of the country and to C. Luzon
7. As of 1996, what sector of Philippine America, it could be collectively and C. Karakoram
economy shows an increase near to 50%? properly described as: 18. What religious group suffered
A. Agricultural Sector D. Service Sector A. Latin America D. South America heavily in Europe during the Nazi
B. Energy Sector E. Tourism Sector B. Middle America E. The Holocaust?
C. Mining Sector Americas A. Atheist D. Jewish
8. Which of the following is not C. North America B. Buddhist E. Muslim
included in the main geographical and 13. The name “Oceania” is widely used C. Christian
political-cultural subdivisions of Asia? to refer to the scattered group of islands
A. North Asia D. Southeast Asia A. Central and Southern Pacific Ocean 19. Most of the people in Latin America
B. Northwest Asia E. Southwest Asia D. Micronesia (Little islands) during the colonial period came from what
C. South Asia B. Central and Western Pacific Ocean known place in the world?
9. This continent is the largest and it E. Polynesia (Many islands) A. Amazon Rainforest
covers almost 33% of the earth’s surface. It C. Melanesia (Black Islands) D. Oceania
is consider as the most diverse continent 14. What country is considered as B. Andes Mountain
and home for earliest civilization and major largest producer of Diamond, although few E. Scandinavian
religions of the world of its diamonds are of gem quality? C. Iberian Peninsula
A. Africa D. Europe A. Australia D. New Zealand 20. It is considered as one of the worst
B. Asia E. North America B. Canada E. South eruption of the century that almost affects
C. Austria and Oceania Africa the earth’s atmosphere due to its ash falls
10. Which area of the world has been C. Mexico A. Mt. Hibuk-Hibok D. Mt. Pinatubo
described as the “symbol of worldwide B. Mt. Kanlaon E. Taal Volcano
demographic, environmental and societal 15. The world’s largest island is: C. Mt. Mayon
stress/problems”? A. Australia D. Greenland 21. Which part of the earth where we
A. Africa D. North America B. China E. Madagascar can experience six (6) months of continuous
B. Asia E. South America C. Iceland daylight and six (6) months continuous
C. Austria/Oceania 16. What sea is often included in the darkness each year?
11. Which of the following continent is physical definition of North America? A. Greenland D. North Pole
considered as the world’s most productive A. Baltic Sea D. Read Sea B. Greenwich, England E. Pacific
in agriculture? B. Caribbean Sea E. South China Sea Ocean
A. Africa D. North America C. Mediterranean Sea C. Iceland
B. Australia and Oceania E. South America 17. It is the longest mountain chain in 22. In what part of the world can you
C. Europe the world approximately find the International Date
12. If the Caribbean Islands may be A. Andes D. Kunlun Line (IDL)?
combined with Mexico and Central B. Himalayas E. Pamir
A. Atlantic Ocean D. 27. The first African nation that tried to Direction: Encircle the letter of the best
Pacific Ocean establish industrialization to lessen the answer.
B. Greenwich, England E. Pole worsening unemployment rate of the 1. Map projections have particular
C. North Pole country purposes; specifically what do we call
23. In what part of the world can you A. Bostwana D. South Africa projections that show all areas on Earth’s
find the longest and largest coral reef which B. Egypt E. Zaire surface in proper proportion to visualize
is also known s the “Great Barrier Reef”? C. Kenya patterns of distribution with spatial area?
A. Africa D. North America A. Albers conic projection
B. Australia E. South America 28. It is known to the ancient and D. Lamber azimuthal projection
C. Europe modern world that this is the home of rich B. Equal area projection
24. It is a common knowledge that all and key cities of the world. It is also of the E. Mercator projection
bodies of water are interconnected with major tourist destination and it provides a C. Gnomonic projection
one another but this ocean covers 1/3 of well designed airports and railways across 2. When these imaginary lines are used in
the world from Arctic to Antartica the continent combination, it gives you a unique
A. Arctic Ocean D. Pacific Ocean A. Africa D. Europe destination to every point on Earth?
B. Indian Ocean E. South Atlantic B. Asia E. South America A. International Date Line (IDL)
Ocean C. Australia D. North Pole and South Pole
C. North Atlantic Ocean B. Lines of Longitude and Latitude
29. The opening of the Suez Canal E. Prime Meridian & Equator
25. Which of the following countries is connects the trading route between these C. North, East, West and South
not part of the Association of the South East two bodies of water Hemisphere
Asian Nations (ASEAN)? A. Atlantic Ocean and Pacific Ocean 3. Earthquakes and volcanic activities
A. Malaysia D. Taiwan D. Mediterranean Sea and Red Sea were frequently experienced by the
B. Philippines E. Thailand B. Caribbean Sea and Gulf of Mexico Philippines due to:
C. Singapore E. South China Sea and Philippine Sea A. Climatic change in Asia for a long
C. Indian Ocean and Bay of Bengal period of time
26. Among the South American nations, 30. This is the known Peninsula in B. Erosion caused by heavy winds and
this country is the only Portuguese speaking Europe that occupied by Norway, Sweden, typhoons all year round
country Denmark also included is Iceland and C. High and low humidity brought by
A. Argentina Finland Northeasterly and Southeasterly wind
D. Mexico A. Apennines D. Jutland D. Short, sharp and shift flowing rivers
B. Brazil B. Balkan E. Scandinavian from Luzon to Mindanao
E. Peru C. Iberian E. Unstable Pacific floor under the
C. Chile PART III: ENHANCING TEST TAKING SKILLS Philippine territory
4. Manila has a good medical facilities A. Most African nations had been A. A 100% increase by 2050 in terms of
but there is a great need for medical people subjected to European domination population
specially in the rural areas, the main factor B. Most inhabitants are literally black B. Africa to its greatest annual increase
for this situation is: or dark people of this continent among the other continents
A. Great exodus of medial practitioners C. Most of its economic and political C. Declined life expectancy in
like Nurses, Medical Technician and Medical activities were dependent to industrialized developing countries due to famine and
Doctors going abroad nations diseases
B. Low quality of graduates that cannot D. Most of the African land area is D. One child policy to Asian nations
passed both local and international covered by vast desert and rugged E. Stability in terms of population
standards for medical practices mountains growth rate
C. Political unrest and security of rural E. Most of the African interior was 9. One of the major issues between US
hospital across the nation due to terrorism unexplored and not colonized by European and Mexico which led US government to
and local conflicts and other neighboring nations take action against the growing population
D. Small number of medical graduates 7. The following statement refers to of Mexico.
like medical doctors and nurses to support the general characteristics of African people A. Assist rapid economic growth of
the needs of the country in terms of their cultural traits except for: Mexico to eliminate poverty
E. Traditional medicine is a strong A. Few nations have developed a B. Control the great demand of farmers
competitor for the scientific advancement strong sense of national unity among its for the south flowing Colorado River for dry
of our Medicinal Science in rural areas people region
5. The “Rood of the World” may be B. Imposition of colonial boundaries C. Control the increase of illegal
associated to: among African people further divide the crossing of Mexicans to the US border in
A. Diverse ethnic group of Asia continent search of employment
B. Frequent occurrence of earthquakes C. Indigenous churches brought D. Manifest a virtual veto power of US
and volcanic eruptions common traditions and practices among towards Mexico’s economic policy
C. Numerous islands, arranged in a African people E. Support peasant rebels stage war
series of arcs D. Traditional values prevailed over the against the state
D. Series of high mountains and African Tribes 10. What do you call the new racial type
plateaus E. Various languages further perpetual created due to intermarriages of most
E. Super continent Pangaea and its tribal identities over and above national Caucasian and African slaves brought to
cultural heritage identity Brazil and Colombia?
6. In the 19th century, most Europeans 8. The population statistics as of 2004 A. Aborigine D. Mulatto
considered Africa as “The Dark Continent” shows evidences that population explosion B. Indian E. Native American
because: will bring: C. Mestizo
11. Which of the following is not a D. Hong Kong, Paris, Rome 1 Africa 4 North America
human activity that threats the incredible B. Beijing, Sydney, Washington DC 2 Antartica 5 South America
biological diversity of the Amazon Wildlife? E. London, New York, Tokyo 3 Europe
A. El Niño Phenomenon C. Berlin, Mexico City, Singapore Arrange the continents from biggest to
D. Massive deforestation 15. The leading industry both for value smallest in terms of land area.
B. Development of rural settlement and employment of many people in A. 31254 D. 4 3 2 5 1
E. Road construction Australia. B. 23415 E. 5 2 4 1 3
C. Increased in demand for lumber A. Electronics & information C. 14523
12. The word “Narcotraficantes” is technology 19. Based on the early historical
associated with: B. Food processing accounts of Europe these two nations had
A. Annual occurrences of El Niño C. Forestry and fishing strong ties with the European culture and
Phenomenon D. Production of machinery for the Asian nations.
B. Colombia’s illegal drug trade transportation A. Australia and New Zealand
C. Destruction of the Amazon Forest E. Tourism D. North Korea and South Korea
D. Trading of Black slaves from Africa 16. Chernobyl will be remembered as B. India and Sri Lanka
E. Volcanic and earthquake activities in A. Commonwealth of Independent E. Turkey and Russia
the Andes mountains States C. Iran and Iraq
13. Which of the following reasons why B. Economic bloc of former USSR to 20. Among the 5 continents below
people cannot establish a permanent European Community 1 Africa 4 Australia/Oceania
settlement in Antartica? C. Place of the nuclear reactor 2 Antartica 5 Europe
A. Danger of nuclear weapons testing explosion 3 Asia
over the territory D. Site of the 1992 Olympics Arrange the continents from smallest to
B. Environmental problems like oil E. Space launcher of Russian biggest in terms of its population based on
spills Cosmonauts that blasted the estimated 2004 data
C. It is the coldest place on earth and A. 53124
experiencing the strongest wind 17. The following are general D. 1 3 4 2 5
D. Ozone concentration in the area characteristics of an industrialized country B. 31452
above the continent except for one. E. 4 5 1 3 2
E. The growing controversy over the A. Economic and political stability C. 24513
claims of different nations over the territory B. High literacy rate 21. The following statements refer to
14. The three leading financial centers C. Longer life expectancy the African condition that hinders its
of the world included the following key D. Low infant mortality potential towards economic growth except
cities of: E. One child policy per family for one:
A. Amsterdam, Frankfurt, Seoul 18. Below are listed 5 continents A. Diverse natural resources
B. Insufficient capital technology experiencing across the time except for the better solution to poverty and
C. Political instability one: unemployment problems
D. Poorly trained workforce A. Deforestation A. Cooperation D.
E. Small purchasing power D. Tsunami Privatization
22. Which of the following Asian B. Diastrophism B. Importation E.
countries has historical ties with European E. Volcanic Eruption Urbanization
city and it was considered as the heart of C. El Niño and La Niña C. Industrialization
the Byzantine and Ottoman Empire? 24. This is known super continent that
A. Iraq (Mesopotamia) D. broke up million and million years ago.
Saudi Arabia A. Atlantis D. Pangaea
B. Israel E. Turkey B. Gondwanaland E.
C. Russia Tethys
23. The following occurrences are all C. Laurasia
natural hazards that the world is 25. Among the developing Asian
nations, it is widely promoted to be one of
29. D30. E

PART III: Enhancing Test Taking Skills

1. B2. B

KEY TO CORRECTION 3. E4. A

PART II: Analyzing Test Items 5. D6. E

1. D 2. C 7. C8. B

3. C4. D 9. C10. D

5. D6. C 11. A12. B

7. D8. B 13. C14. E

9. B10. A 15. B16. C

11. D12. B 17. E18. C

13. A14. A 19. E20. C

15. D16. B 21. A22. E

17. A18. D 23. A

19. C20. D 24. D

21. D22. D 25. C

23. B24. D

25. D26. B

27. D28. D
4. Nora Aunor became a legend in the learn and introduce themselves to the class.
showbusiness because of her talent, What process did the teacher emphasize?
determination and hard work. What role
A. enculturation C. indoctrination
does this illustrate?
B. socialization D. acculturation
A. achieve role C. hereditary role
9. Mildred, an accountant, who now
B. ascribed role D. temporary role
lives with a married man was disowned by
5. Pinning veil around the bride and her family and friends. This punishment is a
the groom/exchange of rings, releasing of form of
PRACTICE TEST I
doves, etc. during wedding ceremony show
A. isolation C. ostracism
1. The adoption of scientific techniques that culture is
to control and manipulate environment B. deportation D. character assassination
A. symbolic C. historical
such as modern methods for farming and
10. When a certain culture is acceptable
fishing is called B. continuous D. changeless
to one group and questionable to others, it
A. social change C. cultural change 6. Aling Maria has taught her daughter shows that:
how to prepare good and delicious desserts
B. technological change A. culture is borrowed C. culture is varied
from local fruits and vegetables. What
D. discovery change cultural transmission process is this? B. culture is a shared product

2. Heavily sanctioned folkway which no A. observation C. enculturation D. culture is relative


one dares to question because they
B. acculturation D. indoctrination 11. The teaching and learning of religion
embody moral laws are called
and values mostly make use of
7. Mr. Francisco was teaching
A. beliefs C. mores
something on mores, folkways, customs A. enculturation C. ulturation
B. customs D. tradition and beliefs in his Social Studies class. What
B. indoctrination D. acculturation
was his lesson about?
3. What is that inevitable folkway
12. In order to achieve the goals for
which no one dares to question because A. material culture C. tangible culture
change, all include the following strategies
they embody moral laws are called
B. non-material culture D. hereditary EXCEPT
A. change C. progress culture
A. Change must involve all sectors of
B. development D. evolution 8. It was the first day of school. Miss the society
Dela Rosa prepared activities which will
make her Grade III children sing, play, talk,
B. Change must be centralized where D. Highly urbanized lifestyle D. Responsible parenthood
only a few initiate the change
16. Which of the following statements 19-20. Which among the strategies show
C. Change must be holistic that focuses regarding the Values Conceptual negative concerns for environmental
on the individual and the system Framework is TRUE? protection?
D. Change should come from the A. It is imposing on every student A. Campaign against wrong health and
leaders and the people environmental practices
B. It is prescriptive of many desirable
13. Which change is bought about by traits B. Suggest the use of synthetic
discovery or modernization to increase substances instead of organic ones
C. It is specific for practical usefulness
production?
C. Link with organizations with projects
D. It is flexible that suits to every
A. cultural change C. technological change on waste management
human individual’s needs
B. societal change D. social change D. Cut tall trees and plant ornamentals
17. The goals of Values Education that
to beautify the surroundings
14. Central to cultural renewal and need to be changed include all the following
social change is a revolution of expectations EXCEPT: 21. Filipino’s love for socials, fiestas and
from ourselves as well as from our celebrations is manifestations of which
A. To become aware of the National
institutions. This concept includes the Filipino’s character?
Issues and Problems that beset the country
following resources from Filipinos, EXCEPT:
A. Spirituality C. Superstitious
B. To encourage OFWs to continuous
A. Assertiveness of Filipino families
work abroad in order to help the National B. Family Solidarity D. Joy and Humor
B. A colonized education Economy.
C. A cultural-fair media C. To develop deep sense of
PRACTICE TEST II
responsibility and accountability
D. A religious that upholds justice and
1. An organized and systematized
righteousness D. To initiative action-centered
manner of learning where expectancies are
community organizations
15. Which of the following factors is the controlled by the teacher is termed as:
reason for communication gap in many 18. Which of the following is NOT an
A. teaching C. schooling
homes? example of economic values?
B. education D. motivation
A. Lack of parental guidance A. Hard work and frugality
2. A social agency that makes a child
B. Modern technology C. Self-determination
learn to value himself and eventually others
C. Inadequate education B. Work values is
A. home C. group D. development of attitudes and skills A. The parent expects her child to do
what he tells him; the child obeys so.
B. school D. society
B. The parent wants to his child to be
3. What will surely takes place 7. Filipinism could be taken as having
in the Dean’s list; the child studies hard.
whenever society adapts modernization the same connotation as a sense of
abruptly? C. The child has many friends; the
A. responsibility C. nationhood
parent allows him to spend time with them
A. There is peace. C. There is progress.
B. professionalism D. philosophy during week-ends
B. There is harmony. D. There is confusion.
8. The process of individual and group D. The parent’s priority is for her child
4. When a person fails to adjust to the change caused by contact with various to be an A student, the child’s predominant
fast changing world, he is likely to suffer people interest
from
A. acculturation C. culturation 12. Why can’t the school implement
A. culture shock C. culture change programs for social reconstruction?
B. enculturation D. indoctrination
B. severe headache D. extreme loneliness A. Most of the students are poor and
9. All people have culture and
passive
5. What is enculturation? therefore share a common humanity. This
shows that B. Some teachers do not like teaching
A. Handing down of culture by a
career
teacher A. culture is universal
C. Some teachers have no dedication
B. Handing down of culture without B. culture is organized
to the profession
any question
C. culture is a product of change
D. Most teachers belong to Low Socio
C. Handing of culture from one
D. culture is the sum total of human Economic Status (SES)
generation to the next
experiences
13. Which of the following best
D. Handing down of culture from one
10. Which of the following sciences illustrates a sense of responsibility that a
who knows to somebody who doesn’t know
below is more connected with the study of professional teacher is expected to have?
6. Which of the following functions social traditions?
A. Is humble
cannot be done by the school?
A. theology C. sociology
C. Has pleasant disposition
A. perpetuation of specie
B. psychology D. anthropology
B. Makes good use of time
B. changing cultural practices
11. Which of the following situations
D. Tutors own student for a fee
C. socialization among children presents a value conflict?
14. When an individual or a group A. Interprets the news for the sale of D. Mrs. Vergara is glad that the parents
adapts the culture of others, practice them readability of her Grade III class volunteer to her in
and becomes habitual, this is: certain class activities
B. Analyzes the relevance of the
A. culture lag C. culture shock personal experience with the news material 21. Does valuing process used as a
teaching strategy?
B. culture change D. culture difference C. Explains the message of the editorial
A. Focus on the affective processing of
D. Introduces socialized recitation
value learning
15. The proper sequencing of grade 19. When does the “Kanya-kanya”
B. Emphasize on the effective
levels according to chronological ages of syndrome become positive?
component of learning
learners is called
A. When one can discover what he
C. Emphasize on the processing rather
A. status C. agency likes in life
than on concept
B. organization D. institution B. When the benefit of others is
D. Concentrate on the concept, process
viewed as one loss
16. Divorce is legal in the United States and behavioral components of value
but is not acceptable in the Philippines. C. When one protects the individual
22. When statement can be considered
What does it show? interests
features of our present educational system?
A. culture is illegal D. When one becomes self-reliant and
A. Knowledge is power
can stand on his own.
C. culture is relative
B. Knowledge does not change
20. Which situation shows a desirable
B. culture is uniform
relationship between teachers and other C. Knowledge begins in the senses
D. culture is phenomenon groups of people?
D. Knowledge can be known only by
17. A school operates not only to cater A. Miss Sison issues report cards only if reason
her students but for all the people where it she remembers.
23. When does a teacher of Values
is located can be called a
B. Mr. Fajardo never entertains Education a good facilitator of learning?
A. public school C. societal school parents’ complains; he feels they are only a
A. Monitors action learning plans for
pest
B. private school D. community school the students
C. As a class adviser, Mr. Magalang has
18. What is the responsibility of a B. Provides cognitive inputs such as
not had a single conference with the
newspaper editorial in values processing? lecturette
parents of his students.
C. Clarifies the students values by the B. Values education is a lifelong B. Education is the process of teaching
use of processing questions process and learning
D. Imposes her own beliefs on her C. The home is the primary source of C. Education is an agent of change and
students basic values modernization
D. The school has the sole D. Education is boundary breaking
responsibility to educate the youth between social classes
24. “One cannot give what he does not
have” is a popular statement which
presupposes the following:
25. Which is not an anthropological
A. Facilitators of values must grow in view of education?
their own personal total development
A. Education is a social institution
15. A

16. D

17. B

18. D

19-20. B, D

21. D

PRACTICE TEST II

1. C

KEY TO CORRECTION 2. B

PRACTICE TEST I 3. D

1. B 4. A

2. C 5. C

3. A 6. A

4. A

5. A 7. C

6. C 8. A

7. B 9. A

8. B 10. D

9. A 11. D

10. D 12. A

11. B 13. B

12. D 14. B

13. C 15. B

14. C 16. C
17. D18. C 24. A

19. D 25. B

20. C21. D

22. C23. C

Licensure Examination for Teachers • Globalization of Goods – through


General Education: Social Science importation and exportation Dimensions of Sustainable Development
Focus: Economics (Majorship) • Globalization of Money – through Economic Dimension
Prepared by: Prof. Serafin A. Arviola Jr. official development assistance (ODA), Social Dimension
investments, grants, donations, etc. Institutional Dimension
• Globalization of Information – via Environmental Dimension
ECONOMIC ISSUES AND CONCERNS internet, cable TV, education, etc. Cultural and Spiritual Dimension
• Globalization of People – through
1. Poverty and Income Distribution travel, scholarships, fellowships, job, etc. b. The 8 Millennium Development
• Poverty line – an amount of income Goals
below which a family is considered “poor” 3. Development
• Income Poverty – based from the a. Sustainable Development 1. Eradicate extreme poverty and
United Nations, are those individuals living hunger
on an income of $1 a day or less than Development that meets the needs of the Reduce by half the production of
$1/day. present generation without compromising people living on less than a dollar a day
• Human Development Index – the ability of the future generations to meet Reduce by half the proportion of
measure of human development looking at that own needs. United Nations declared people who suffer from hunger
the following indexes: longevity, income 2005-2014 as the Decade on Education for
and educational attainment Sustainable Development. It pursues the 2. Achieve universal primary education
• Human Poverty Index – a measure following global vision: Ensure that all boys and girls
of poverty looking at the income of complete a full course of primary schooling
individuals The vision of education for sustainable
development is a world where everyone has 3. Promote gender equality
2. Globalization - The movement of the opportunity to benefit from quality Eliminate gender disparity in
money, goods, information and people education and learn the values, behavior primary and secondary education
across nations made possible by the rapid and lifestyles required for a sustainable preferably by 2005, and at all levels by 2015
advances in travel and communication. future and for positive societal
transformation. 4. Reduce child mortality
Reduce by two thirds the mortality governance, development and poverty • It is compulsory level on private
rate among children under five reduction – nationally and internationally individuals and organization by the
Address the least developed government to raise revenue to finance
5. Improve women’s reproductive countries’ special needs. This includes expenditure on public goods and services.
health tariff-and quota-free access for their Purpose of Taxation
Reduce by three quarters the exports; enhanced debt relief for heavily
maternal mortality ratio indebted poor countries; cancellation of • To collect revenue for the
official bilateral debt; and more generous government
6. Combat HIV/AIDS, Malaria and other official development assistance for • To redistribute income
diseases countries committed to poverty reduction • To combat inflation
Halt and begin to reverse the spread Address the special needs of • To correct an adverse balance of
of HIV/AIDS landlocked and small island developing payment
Halt and begin to reverse the States • To check consumption of goods
incidence of malaria and other major which are considered undesirable
diseases c. Philippine Initiatives on Sustainable • To protect local infant industries
Development • To influence population trend
7. Ensure environmental sustainability 1. Philippine Agenda 21 in 1996 • To improve unfavorable terms of
Integrate the principles of 2. Philippine Council for Sustainable trade
sustainable development into country Development in 1997 • To reallocate resources to create a
policies and programmes; reverse loss of 3. Major legislations on Sustainable sense of identity
environmental resources Development
Reduce by half the proportion of Clean Air Act of 1999 Sources and Origin of Taxation
people without sustainable access to safe Ecological Solid Waste Management
drinking water Act of 2001 • The Constitution
Achieve significant improvement in • Statutes or Presidential Degrees
lives of at least 100 million slum-dwellers by TAXATION • Bureau of Internal Revenue
2020 regulations
• It is an inherent power of the state • Judicial Decision
8. Develop a global partnership for to impose and collect revenues to defray • Provincial, Municipal and Barrio
development the necessary expenses of the government. Ordinances
Develop further an open trading and • It is compulsory contribution • Observance of International
financial system, that is rule-based, imposed by a public authority irrespective Agreement
predictable and non-discriminatory. of the amount of services rendered to the • Administrative Ruling and Opinions
Includes a commitment to good payer in return.
Classification of Tax System • Examples: income tax, residence tax,
real state, immigration tax, Republic Act No. 6657 – The
• Progressive Income Tax – the Higher estate/gift/inheritance tax. Comprehensive Agrarian Reform Law of
the income the higher the tax rate. 1988 which was signed into law by Pres.
• Proportional Tax – The tax rate is • Indirect Taxes Corazon Aquino
constant and unaffected by the level of • The tax burden can be shifted to the
income. third party Meaning - The redistribution of lands,
• Regressive Tax – The higher the • Indirect taxes are based on regardless of crops or fruits produced to
income the lower the tax rate. expenditure and consumption farmers and regular farm workers who are
• All indirect taxes are regressive in landless, irrespective of tenurial
Classification of Taxes nature arrangement to include the totality of
• Indirect taxes are optional in the factors and support services designed to lift
• As to the subject matter – Personal sense that they can be avoided their economic status of the beneficiaries
Property, Capitation and Poll Tax; Property • Examples: sales tax, import tax, and all other arrangements alternative to
Tax; Excise Tax • As to who bears the VAT/EVAT physical redistribution of lands, such as
burden – Direct Tax and Indirect Tax production, profit sharing, labor
• As to determination of the amount – Characteristics of a Sound Tax System administration and the distribution of
Specific and Ad valorem • Efficiency – must generated revenue shares of stocks, which will allow
• As to purpose – General and Specific greater than the amount of money the beneficiaries to receive a just share of the
• As to Scope – National and Local government must spend to collect taxes. fruits of the land they work.
• Equity – individual and groups
Types of Taxes belonging to the same income bracket must Principles of Agrarian Reform
be taxed equally while belonging to
• Direct Taxes different income groups must be taxed The policy of the state to pursue a
• The burden cannot be shifted to the differently. comprehensive Agrarian Reform Program
third party • Convenience – to set up measures (CARP) to:
• Direct taxes are based on income and procedures that will make it more • To promote social justice
and wealth convenient for taxpayers to pay. • To move the nation toward sound
• In most cases, direct taxes are • Stability – tax system must not be rural development and industrialization
progressive in nature too often or it will encourage tax payers to • To establish owner-cultivatorship of
• Direct taxes are compulsory in withhold tax payment until a more economic sized farms as basis of Philippine
nature preferred system is put in place agriculture.

AGRARIAN REFORM Coverage of CARP


• All alienable and disposable lands of The policy of the state to pursue a Beneficiaries
the public domain devoted to or suitable for comprehensive Agrarian Reform Program • Agricultural lessees and share
agriculture (CARP) to: tenants
• Stability – tax system must not be • To promote social justice • Regular farm workers
too often or it will encourage tax payers to • To move the nation toward sound • Seasonal farm workers
withhold tax payment until a more rural development and industrialization • Other farm workers
preferred system is put in place • To establish owner-cultivatorship of • Actual tillers or occupants of public
economic sized farms as basis of Philippine lands
AGRARIAN REFORM agriculture. • Collectives or cooperatives
• Other directly working on the land
Republic Act No. 6657 – The Coverage of CARP Salient Features of CARP
Comprehensive Agrarian Reform Law of • All alienable and disposable lands of • CARP covers all agricultural lands
1988 which was signed into law by Pres. the public domain devoted to or suitable for and not only devoted to rice and corn
Corazon Aquino agriculture • CARP covers not only those privately
• All lands of the public domain in owned tenanted lands but also that of
Meaning - The redistribution of lands, excess of the specific limits as determined agricultural land owned by Multinational
regardless of crops or fruits produced to by the Congress Corporations and commercial farms.
farmers and regular farm workers who are • All other lands owned by the • Lower retention limits of three
landless, irrespective of tenurial governments devoted to or suitable for hectares
arrangement to include the totality of agriculture • Rights of indigenous communities,
factors and support services designed to lift • All public lands devoted to or to their ancestral lands are protected to
their economic status of the beneficiaries suitable for agriculture regardless of the ensure their economic, social and cultural
and all other arrangements alternative to agricultural products raised or can be well being
physical redistribution of lands, such as raised. • In determining just compensation,
production, profit sharing, labor the cost of acquisition of the land, the
administration and the distribution of Retention Limits current value of like properties, its nature,
shares of stocks, which will allow • Five hectares for land owners actual use and income, the sworn valuation
beneficiaries to receive a just share of the • Three hectares to be awarded to of the owner, the tax declarations and the
fruits of the land they work. each child of the landowner subject to the assessment made by the government
following qualification: assessors shall be considered.
Principles of Agrarian Reform o At least 15 years old • Lands awarded to beneficiaries shall
o Actually tilling the soil or directly be paid to the Land Bank of the Philippines
managing the farm in 30 annual amortization at six percent
interest per annum.
they must purchase shares of the
COOPERATIVES cooperative, obtain the right to govern the
organization, voting rights of the owner are • Continuing Membership
New Cooperative Laws – Cooperative Code on the basis of one person, one vote. Pre-membership education seminar
of the Philippines (RA 6938), Cooperative • Limited Interest on Capital – Capital as required for entry to the cooperative
Development Authority (RA 6939) and in a cooperative is like a loan because the Special trainings for the cooperative
Executive Order 95 and 96 issued by owners of the capital can expect to received leadership and members
President Fidel Valdes Ramos. a rate of return not exceeding that of the
prevailing market interest rates on • Cooperation Among Cooperative –
Definition - A cooperative is: investing. interlending and pooling of funds
• A free association of persons • Division on Net Surplus – Net
voluntarily joined together surplus should be distributed as follows: Typologies of Cooperative
• With common bond of interest Item %
• Legally constituted allocation 1. According to Level of Cooperatives
• Purpose of conducting an economic General Reserve Fund
enterprise At least 10% • Primary – members of which are
• Owned, controlled and administered Education/Training Fund natural
democratically At Least 10% • Secondary – members of which are
• Making equitable contributions to Optional Fund primaries
the capital required At Least 10% • Tertiary – members of which are
• Accepting a fair share of the risks Dividend/Patronage Refund secondary upward to one or more apex
and benefits Remaining Balance of Savings organization
• Organized in accordance with
generally accepted principles General Reserve Funds – cover 2. According to Services Rendered
losses in operation
Universal Principles of Cooperativism Education/Training Funds – for • Credits – is one, which promotes
members and Management trainings thrift among its members, and creates
• Open and Voluntary Membership - Optional Funds – discretion of funds in order to grant loans for productive
No artificial discrimination against cooperatives for purposes of acquiring land and provident purposes.
individuals because of their race, creed or construction of a building or community • Consumer – is one wherein the
political affiliation, freedom of entry and development primary purpose is to procure and
exit of any member of the cooperative Dividends/Patronage Refunds – the distribute commodities to members and
• Democratic Control – In order for volume of transaction that members have non-members.
members to gain entry to the cooperatives, with the cooperative
• Producers – is one which undertakes transportation, insurance, housing, labor, • Institutional – Members are
joint production whether agricultural or electricity, communications and other employees of a specific institution or
industrial; services. corporation
• Marketing Cooperative – is one • Multipurpose – is one which • Associational – Members are those
which engages in the supply of production combines two or more activities o these who have their own enterprise and belong
inputs to members and in turn market their different types of cooperatives to specific sector or organization.
products. • Community-Level – Members are
• Service – is one engages in medical 3. According to Scope of Membership based on a defined geographical area.
and dental care, hospitalization,
PREBOARD EXAMINATION a. Yes, if nobody can take his place a. I only
b. No, unless she has enrolled in b. I and IV
refresher course of 12 units
PROFESSIONAL EDUCATION c. II and III – I,II,III BEST ANSWER
c. No
d. II only
d. Yes
5. Can Manny Pacquiao be given a
3. Is membership to the accredited special permit to teach boxing in a special
The Teaching Profession, Social Dimensions
professional organization for teachers school?
for Education
mandatory for all LET passers?
a. No, he is not a teacher education
1. To whom does the word teacher
a. No graduate
refer?
b. Yes, when the teacher is already b. No, he has not passed the LET
I. Full time teachers
teaching
c. Yes, he is a graduate of ALS
II. Part time teachers
c. Yes
d. Yes, he has excelled and gained
III. Guidance counselors
d. Only for LET passers who are not international recognition
IV. Librarians repeaters
6. Is it professional for a teacher to
V. Division Superintendent 4. Which is true of the periodic merit receive gifts from the student and parents?
exam for teacher provided for in RA 7836?
a. I, II, and III a. Not at all
I. Consist of oral exam
b. I and III b. No, especially if done in exchange
II. Consist of written exam for requested concessions
c. I, II, III, and IV –I,II,V BEST ANSWER
III. May serve as additional basis for c. Yes, if deserved
d. III and IV
merit promotion in addition to performance
d. Yes, in-season and out-of-season
2. Teacher Kevin has not practiced his rating
gifts
profession for the past five years. Can he go
IV. Taken with fee of P 1000 per
back to teaching immediately?
examinee
7. An Education graduate without a d. Job interviews for teacher applicants b. Student-centered curriculum
license is accepted to teach in a private
10. Among active participation of school c. Mission and core values
school? Is this in violation of RA 7836?
officials and teachers in the community,
d. Student handbook of conduct
a. No provided he has taught for at which of the following is not appropriate
least 3 years due to prevailing religious sentiments? 13. Among rights of the schools, which
is not provided by the law?
b. Yes. No one may teach without a a. Literacy assistance for out of school
license children/youths a. Right for basic education to
determine subjects of the study
c. No b. Household campaign for healthful
practice b. Right to enforce administrative
d. Yes
systems
c. Promoting contraceptives for
8. For relevance to business and
planned parenthood c. Right to provide proper governance
industry, what did the First Biennial
National Education on Education (2008) d. Introducing cooperative thrift d. Right for institutions of higher
impose for updating the Licensure practices learning to determine academic grounds for
Examination for teachers? admission
11. Which of the following is not John
a. Moral or ethical values Dewey’s contribution to the sociological 14. What kind of grassroots model best
foundation of education? advances Education for All as served
b. Technical and scientific
children of slum city dwellers?
competencies a. Facilitating learning along social
conditions of the learner a. Mobile education on Kariton
c. Upgraded laboratory facilities
b. As a social process, education begins b. Leaf flyers for out-of-school children
d. Vocational skills
at birth
c. Radio education modules
9. What does the Teacher Education
c. True education is transmission of
Development Program signify as a d. Educational television
knowledge
prerequisite for employment of teachers in
15. After the implementation of NCBTS,
basic education schools? d. The school is a continuation of
results of LET still reveal low performance
home
a. National Standard Competencies among examinees. What can teacher
among teachers 12. Of the following, which is most education institutions do to upgrade their
fundamental to building up a strong school graduates’ LET performance?
b. Licensure Examination for Teachers
culture of excellence?
a. Review curriculum vis-à-vis TOS
c. Induction of new teachers
a. High standards of performance
b. Intensify Field Study Courses b. Aesthetic sense d. Technology integration in instruction
c. Hire expensive review trainers c. Spiritual values 22. What is known as a self-appraisal for
professional growth that is acceptable and
d. Implement selective admission in d. Personal responsibility
useful for recognizing weakness and
TEIs
19. Which program directly embodies strengths for a new beginning teacher?
16. What is the cultural trait of both the pre-service and in-service
a. master teacher’s evaluation
conflicting values that aims to please programs?
people in different venues and situations b. student’s evaluation
a. BESRA – Basic Education Sector
rather than abide by principles?
Reform Agenda c. principal’s evaluation
a. Crab mentality
b. TEDPA – Technical Education d. self-evaluation
b. Split personality Development Program
23. Among reforms for enhancing
c. Kanya-kanya system c. K-12 teacher professionalism, which has been
implemented by law in order to determine
d. Bahala na mentality d. BEC – Basic Education Curriculum
whether prospective teachers have
17. Among qualities which employers 20. How can the efforts of four agencies acquired professional competencies prior to
look for in the 21st century workplace, (DepEd, CHED, PRC, CSC) be best achieved granting them a permit to teach?
which is the most challenging and for the training and development of
a. accrediting a national organization
demanding? teachers?
for teachers
a. Aptitude for teamwork a. Synchronization
b. setting up centered for excellence in
b. Skills and social behavior b. cost-reduction teacher education centers

c. Readiness to take risks c. streamlining c. licensure examination

d. Specific competencies for work d. sharing of resources d. creation of a professional board for
teachers
18. In educating the whole person as 21. What is the core of the Teacher
demanded by the “Learning to be” pillar of Education Development Program? 24. From global competence as defined
the 21st century education, where does the by international educators, which is the
a. high order thinking skills or HOTS
concept of meaning, purpose and most appropriate characteristic of globally
engagement belong? b. student-centered learning competent individual?

a. Mind and body c. National Competency-Based a. familiarity with new culture


Teaching Standards
b. open-mindedness to new culture Philippines, what language was imposed 30. In the formal education system
under the one-language policy? during Hispanic times in the Philippines,
c. adaptability to new work
what was not implement but which we
environment a. Spanish
enjoyed during the American period?
d. foreign-language policy b. English
a. vocational education
25. For a school, which of the following c. Tagalog
b. private education
is most significant in repairing shorelines
d. Filipino
with depleted coral reefs? c. religious education
28. Of the following, which is the most
a. outreach by depositing rubber tires d. public education
functional intervention in order to achieve a
as artificial coral reefs
basic right of every Filipino Child under the 31. If Dr. Jose Rizal lives in the 21st
b. implement reporting system against Constitution and Magna Carta for Disabled century, what character expression and
dynamite fishermen Persons? commitment would have shown our
generation?
c. legislative lobby to disallow tourism a. Philosophy of education
in endangered shorelines a. inventor of techniques
b. policy for curricular reform
d. outreach by educating the villagers b. citizen and producer
c. home study program
on protection of coral reefs
c. member of family and community
d. structural organization
26. In a tertiary school, the President
d. creative dreamer
organized a Fun Run for students, faculty 29. Of the following interventions,
and personnel to enjoy camaraderie, which is directly aimed at responding to the 32. In the learning to do pillar of new
physical exertion under the sun, sense of transitional gap between academic education, what is the enabling factor that
engagement and achievement. What does achievement and employment? can make the learner fully contribute to a
the activity promote? peaceful and just society?
a. identification of centers of
a. spiritual vigor excellence a. knowledge
b. cultural consciousness b. deregulation of tuition fees b. skills
c. national integrity c. school networking with business and c. insights
industry
d. moral integrity d. values
d. voluntary accreditation of schools
27. In the Education Act of 1901 which 33. Before being able to fully learn to
established a free public education in the live and work together under the pillar of
the 21st century education, what must the 36. Which of the following best defines minds, who have no sensitivity to the
learner attain for himself? a morally mature person? welfare of other people?
a. find peace within oneself a. Cultural values clarification a. Pharisaical
b. attain an altruistic mind b. Unhampered exercise of one’s right b. Strict
c. love his fellowmen c. Transmittal of one’s moral viewpoint c. Lax
d. become self-actualized d. Knowledge and practice of universal d. Callous
moral values
34. The Transparency International’s 40. What was the degree of moral
perception that the Philippines suffers a 37. Educated in a religious school, Sansa certitude when U.S. statement decided to
cultural malaise of corruption, what goes to confession every day to be free of drop the atomic bombing on Hiroshima and
component of our character needs to be any kind of sin. How do you characterize Nagasaki to prevent mass deaths by a land
further developed along the Learning To Be Dona’s moral attitude? invasion of Japan?
Pillar of education in the 21st century?
a. Callous a. Doubtful
a. Familial-social component
b. Pharisaical b. Certain
b. Physical-economic component
c. Scrupulous c. Perplexed
c. Intellectual-emotional component
d. Strict d. Probable
d. Ethical-spiritual component
38. How would you characterize the 41. Teacher Slash is of the thinking that
35. This powerful European country moral attitude of Hispanic friars who taught from the very start students must be made
supplied arms to Afghanistan rebels who religion but were unfaithful to their vow of to realize study is indeed hard work. To
were fighting a terrorist war in the Middle property by amassing the land properties of which philosophy does Teacher Susan
East. What was the principle of moral natives? adhere?
discernment applicable in this case?
a. Scrupulous a. Essentialism
a. Principle of double effect
b. Strict b. Perennialism
b. Principle of lesser evil
c. Lax c. Progressivism
c. Principle of material cooperation
d. Pharisaical d. Reconstructionism
d. Principle of moral cooperation
39. How would you characterize the 42. If your students appear to be more
moral attitude of prisoners with criminal interested in a topic outside your planned
lesson for the day, you set aside your lesson In a faculty meeting, the principal told his 47. If you lean toward a progressivist
plan for that day and grasp the opportunity teachers: We need to improve our school philosophy, with which response do you
to discuss the topic of particular interest to performance in the National Achievement agree?
your students. Strike the iron while it is hot! Test. What should we do? The teachers
a. 4
Which philosophy governs for your action? gave varied answers as follows:
b. 2
a. rationalism 1. Let’s give incentives and rewards to
students who get a rating of 85% c. 1
b. empiricism
2. Let’s teach them to accept complete d. 3
c. existentialism
responsibility for their performance
How a teacher relates to his/her pupils
d. progressivism
3. Let’s make the school environment depends on his/her concepts about
43. Students must be taught self- conducive for learning him/her. In a faculty recollection, the
responsibility is the desire of the teachers were asked to share their thoughts
4. Let’s make use of the experiential
___________ teacher. of the learner, their primary customer.
methods of teaching
What follows are the gists of what were
a. Existentialist
45. On which educational philosophy is shared:
b. Utilitarianist response #1 anchored?

c. Pragmatic a. Behaviorism
Teacher A – The learner is a product of his
d. Constructivist b. Progressivism environment. Sometime he has no choice.
He is determined by his environment.
44. Who asserts that teaching is not just c. Existentialism
depending knowledge into the empty minds Teacher B – The learner can choose what he
d. Essentialism
of the learners? It is helping students create can become despite his environment.
knowledge and meaning of their 46. Which response/s come/s from a
Teacher C – The learner is a social being
experiences? behaviorist?
who learns well though an active interplay
a. Constructivist a. 1 and 3 with others

b. Essentialist b. 2 and 4 Teacher D – The learner is a rational being.


Schools should develop his rational and
c. Existentialist c. 1 and 2
moral powers
d. Pragmatist d. 3 and 4
SITUATIONAL
48. Whose philosophical concept is that I. Parents c. Motivating students
of Teacher A?
II. Teachers d. Changing attitudes and feelings
a. Behaviorist’s
III. the state 55. Of the following effects on learning,
b. Existentialist’s what is the effect of assigning various
IV. the schools
sections of the newspaper, and allowing
c. Progressivist’s
52. Of the three aspects of learning, choice depending on the learner’s choice?
d. Rationalist’s which is not mentioned as needed so that
a. Encouraging participation
the individual learner in the 21st century
49. If you agree with Teacher C, you are
can learn how to learn? b. Reinforcing learning
more of a/an
a. Ability to think c. Allowing different interests
a. Progressivist
b. Mathematical skills d. Changing attitudes and feelings
b. Perrenialist
c. Memory skills 56. A young mother observes her seven
c. Essentialist
year old girl glued to her computer games.
d. Concentration
d. Rationalist What aspect of the family life may suffer
53. Which of the following belongs to due to obsession of the young with
50. Whose response denies man’s
the more sophisticated learning-to-learn technology gadgets?
freewill?
skills for the individual learner?
a. Family social life
a. Teacher A’s
a. To ask and gather data
b. Family economic life
b. Teacher C’s
b. To listen and observe
c. Discipline and obedience
c. Teacher B’s
c. To process and select information
d. Parent-child relationship
d. Teacher D’s
d. To read with understanding
57. Which of the following is not an
54. Of the following effects on learning, advanced process of meta-cognition among
Human Growth and Development, what is the effect of simulations that make learners?
Facilitating Learning, Developmental students feel and sense experience in the
a. Learning how to recognize thoughts
Reading classroom?
b. Acquisition of new knowledge
51. From a broad vantage view of a. Reinforcing learning
human development, who has the primary c. Assessing own thinking
b. Providing experiences that
duty to educate the youths or children?
otherwise might not be had d. Learning how to study
58. Of comprehension or thinking schools. What factor is shown to affect determine champions in identifying people,
strategies, which is relating one or two reading achievement? places, and events. What learning objective
items, such as nouns and verbs? outcome does she aim to achieve?
a. Mobility
a. Basic elaboration strategies a. Knowledge or recall
b. Personality and emotional factors
b. Complex rehearsal strategies b. Perpetual abilities
c. Socio-economic status
c. Complex elaboration strategies c. Application
d. Listening comprehension
d. Affective strategies d. Responding
62. When preacher Xian read the
59. Of skills teacher should understand Genesis story on creation, he explained that 65. In Erikson’s stage theory of
and students need to acquire, which is the God is so powerful he created the universe development questionnaire, which
ability to integrate complex information in only seven days. What level of reading affirmation does not belong to the stage of
into categories through its attributes comprehension did preacher John apply? initiative vs. guilt?
(characteristics, principles or functions)?
a. Evaluative reading on character, plot a. People can be trusted
a. Scanning or style
b. In difficulty, I will not give up
b. Complex cognitive b. Literal reading the lines
c. I feel what happens to me is the
c. Sharpening-leveling c. Applied reading beyond the lines result of what I have done
d. Complexity-simplicity d. Interpretative reading between the d. I am prepared to take a risk
lines
60. Inculcating moral maturity among 66. For cognitive learning, what are sets
students, which of the following relates to 63. What is the main organization and of facts, concepts, and principles that
belief and ideals? orientation of science and social studies describe underlying mechanism that
reading materials? regulate human learning, development and
a. Promoting human equality
behavior?
a. Expository
b. Refraining from prejudiced action
a. Facts
b. Descriptive
c. Avoiding deception and dishonesty
b. Concepts
c. Narrative
d. Respecting freedom of conscience
c. Theories
d. Argumentative
61. Research studies showed that
d. Hypothesis
children in slums generally have lower 64. In his History class, teacher Naomi
reading achievement then children in urban used a current events IQ contest to
67. Literature teacher Kim introduced a. Giving ample opportunity for a pupil d. Autism
figures of speech in poetry to improve to explore rather than simply dish out
73. Of the following, which is most true
ability of her students to interpret verses. information
of adolescents?
What kind of thinking is she developing in
b. Employing principles in multiple
her students? a. Hormonal changes
intelligence in teaching
a. Critical thinking b. Last splurge of dependence
c. Making a left-handed pupil write
b. Metaphoric thinking with her right hand as this is better c. Unruly behavior
c. Convergent thinking d. Allowing open dialogue among d. Defiance of peer group
students of various cultural backgrounds
d. Divergent thinking 74. Research says, “people tend to
71. Of the following which is normally attribute successes to internal causes and
68. Of clusters of meaningful learning
expected of Grade VI pupils? their failures to external causes.” What
activities, which does not belong to spatial
does this imply as a most potent key to
learning activities? a. Getting along with classmates
success?
a. Visualization b. Being independent of parents
a. Reasoning
b. Concept-mapping c. Showing class leadership
b. Imagination
c. Peer tutoring d. Displaying a male or feminine social
c. Application
role
d. Art projects
d. Motivation
72. From categories of exceptionalities
69. From cluster of meaningful learning
in the young child and adolescents what 75. From Kohlberg’s theory of moral
activities, which does not belong to verbal-
involves difficulties in specific cognitive development, what is the moral reasoning
linguistic intelligence learning?
processes like perception, language, or perspective of Mother Teresa who
a. Ecological field trip memory due to mental retardation, pledged her life to serve the sick and very
emotional/behavioral disorder, or sensory old?
b. Debates
impairment?
a. Social contract
c. Journal writing
a. Learning disabilities
b. Universal principles
d. Reading
b. Speech and communication
c. Obedience
70. Which of the following violates the disorders
principle that “each child’s brain is unique d. Law and order
c. Emotional/conduct disorders
and vastly different from one another”?
76. Blind cyclist and teacher Maria d. Reflective 82. On categories of exceptionality in
Bunyan won 8th place in the able-bodied the young, what is difficulty in focusing and
79. In what development stage is the
Sydney 2000 Olympics. Of the following, maintaining attention, and/or recurrent
pre-school child?
which is the central and fundamental hyperactive and impulsive behavior?
quality she displayed by never thinking that a. Early childhood
a. ADHD
blindness is an impediment to becoming a
b. Babyhood
great athlete? b. Emotional/conduct disorders
c. Infancy
a. Perseverance c. Autism
d. Late childhood
b. Passion d. Speech and communication
80. What is mainly addressed by early disorders
c. Dedication
intervention program for children with
83. What kinds of skills are commonly
d. Self-belief disabilities, ages 0 to 3 years old?
dominant in subjects like Computer, PE,
77. How can new information be made a. Ensuring inclusion for special Music, and the like?
more meaningful to students? children
a. Problem-solving skills
a. Relating it to knowledge they b. Early growth development lag
b. Manipulative skills
already know
c. Identifying strengths and
c. Affective skills
b. Valuing new knowledge weaknesses in special children
d. Thinking skills
c. Demonstrating novelty of new d. Preventing labeling of disabled
knowledge children 84. How is the disorderly behavior of
children classified when they tell lies?
d. Increasing retention of new 81. What is the degree of moral
knowledge certitude of Jade Althea who entered into a. Moral
marriage only out of obedience to her
78. Under the domains of learning, to b. Intellectual
parents, but uncertain whether she wanted
what domain do Reflex movements,
marriage at all? c. Social
perceptual abilities, and non-discursive
communication belong? a. Certain d. Psychological
a. Psychomotor b. Lax 85. Which of the following is not among
the major targets of the child-friendly
b. Affective c. Probable
school system (CFSS)?
c. Cognitive d. Doubtful
a. All school children are friendly
b. All children complete their a. Authoritarian values d. Equilibrium
elementary education within six years
b. Universal values 92. In Piaget’s stages of cognitive
c. All children 6-12 years old are development, which is the tendency of the
c. Creedal values
enrolled in elementary schools child to only see his point of view and to
d. Sectarian values assume that everyone has the same point
d. All grade six students pass the
of view?
division, regional, and national tests 89. Among models of reading strategies,
what did student Jk adopt when she reads a. Reversibility
86. Research studies that reading power
back and forth, attending to both what is in
affects college students who have insomnia, b. Egocentrism
her mind and what’s on the page?
conflicts with parents, poor rapport with
c. Symbolic function
other people. What factor(s) is shown to a. Bottoms-up
effect reading achievement? d. Centration
b. Interactive
a. Home conditions 93. Which is the most basic in Maslow’s
c. Down-top
hierarchy of needs?
b. Socio-economic status
d. Top-down
a. Socialization
c. Personality and emotional factors
90. Of the following, how can self-
b. Actualization
d. Perception and comprehension esteem be best developed among learners?
c. Self-esteem
87. Among the following, which is the a. Doing fair share in community work
abstract form of learning, parents teach d. Altruism
b. Fulfilling commitments
their children?
94. Which aspect of multi-intelligence is
c. Through relationships with others
a. Tumulong ka sa paglinis ng bahay enhanced by asking students to work on a
d. Displaying self-control physical model of the atom after a teacher’s
b. Magbasa ka ng libro
discussion on the subject of the atom?
91. Of Piaget’s Cognitive Concepts,
c. Palagi kang magdasal
which refer to the process of fitting a new a. Interpersonal
d. Mapakabuti ka experience to a previously created cognitive
b. Linguistic
structure or schema?
88. What characteristic differentiate
c. Kinesthetical
spiritual intelligence or spiritual quotient as a. Assimilation
developed by Harvard University, from d. Mathematical
b. Schema
sectarian religion (E.g. Christian, Buddhist,
95. Among specialist in reading, who are
Jewish, etc.)? c. Accommodation
mainly concerned about reading as a
thinking process that involves the a. Intellectual d. Brains of boys are bigger and better
recognition of printed or written symbols than those of females
b. Social
which serve as thought stimuli?
Assessment of Learning, Field Study,
c. Moral
a. Semantics Practice Teaching
d. Psychomotor
b. Psychologists 101. Of the types of validity tests, what is
98. How do you describe transfer of concerned with the relation of test scores
c. Linguists
learning across subject matter, e.g value of to performance at some future time, e.g.
d. Sociologists thrift in Economic and Social Science? Freshmen college test can show success in
college?
96. Sequence the following events on a. Horizontal
the historical development of reading: a. Curriculum validity
b. Spiral
I. Greek letters and the Roman b. Criterion validity
c. Vertical
alphabet were developed
c. Content validity
d. Cyclic
II. Through the Semite’s ingenuity,
d. Predictive validity
sounds, and symbols gave rise to the 99. What broad learning is needed for a
Phoenician alphabet learner to desire to learn throughout life? 102. The test questions in Teacher Dae
Dae’s test were confusing and subject to
III. People used pictures and characters a. Four basic Rs
wrong understanding, especially to poorer
to convey messages
b. Basic education students. What was wrong with the test?
IV. Researchers showed the processes
c. General education a. Inappropriate level of difficult of
of reading, comprehension, and
items
interpretation d. Pre-school system
b. Unclear directions
a. I, II, III, and IV 100. What observation attests to the fact
that the sudden student’s motivation vary c. Ambiguity
b. I, II, IV and III
according to socio-cultural background?
d. Test items inappropriate for
c. III, II, I and IV
a. Females mature earlier than boys outcomes being measured
d. IV, II, I and III
b. Children from low-income 103. Of the following, which exemplifies
97. How is the disorderly behavior of household meet more obstacle in learning the best example of cooperation and
children classified when they don’t focus voluntarism in the Parent-Teacher
c. Genetic endowments may show
and lack attention? Associations?
gifted endowments among the young
a. Helping hands after a natural crisis, a. Efficiency a. Promise of high rank and prestige
e.g. devastating storm
b. Usability b. Social service to upcoming
b. Attending regular meetings generations
c. Reliability
c. Fund raising for PT funds c. Economic security and welfare
d. Validity
d. Running the school canteen d. Respected position in society
107. Of the following subjects, which
104. Among standardized tests, which does not belong to performance-based 110. To what process of evaluation does
reveals strengths and weaknesses for subjects in which direct instruction is determining the extent objectives are met
purposes of placement and formulating an effectively used? belong?
appropriate instructional program?
a. Values education a. Authentic
a. Personality tests
b. Music b. Formative
b. Achievement tests
c. Science c. Criterion-referenced
c. Diagnostic tests
d. Mathematics d. Norm-referenced
d. Competency tests
108. Which of these approaches would 111. Which form of the foundation of all
105. Among standardized tests, which reform assessment outcomes? cognitive objects without which the next
can show how students perform in level of higher thinking skills cannot be
a. Apply sanctions on low performing
comparison with each other and to attained?
schools
students in other schools?
a. Knowledge
b. Focus on testing without investing
a. Competency tests
the learner’s needs b. Synthesis
b. Subject exit tests
c. Use understanding as means of c. Application
c. Achievement tests giving feedback on students learning
d. Analysis
d. Diagnostic tests d. Compare results of performance of
112. What primary response factor is
all schools
106. Teacher Bea Bunana makes her tests considered by Essay questions?
easy for students to understand, easy to 109. Using extrinsic motivational
a. Factual information
administer and score and suitable to test assessment, what could be the most noble
conditions, e.g. time. What is she achieving motive in students pursuing a lifetime work b. Wide sampling of ideas
for her tests? and mission for the teaching profession?
c. Originality
d. Less time for construction and d. He got 60% of the items correctly c. It is moderate in difficulty
scoring
116. Which of the following may not be d. It is very easy
113. Among written categories of adequately assessed by a paper and pencil
120. With the mode of answering as a
assessment methods, what did teacher test?
point of reference, which of the following
Maggie Lagid use when she assessed the
a. Sight reading in music does not belong to this test group?
stock knowledge of her students through
questioning in an open class? b. Multiplication skills a. Completion
a. Oral questioning c. Subject-verb agreement b. Essay
b. Performance test d. Vocabulary meaning c. Problem-solving
c. Product rating scale 117. What should be done with test item d. Matching
whose difficulty index is .98?
d. Observation and self-report 121. One half of the class scored very
a. Revise it low. Teacher Janus gave another tests to
114. In the context of the 6 facets of
determine where were the students were
understanding cited by Wiggins and b. Retain it
weakest. What type of test is this?
McTIghe, what is a proof of a student’s
c. Reject it
understanding a principle? a. Aptitude test
d. Reserve it for another group of
a. Stating given examples b. Remedial test
students
b. Repeating it as given by the teacher c. Diagnostic test
118. What is known as the scoring guides
c. Applying it to solve his problem for rating open-ended questions? d. Readiness test
d. Retaining it in memory for a long a. Rubrics 122. On what is normative marking
period of time based?
b. Outcomes
115. What does it mean if student Pete a. High marks of few students
c. Scales
got a 60% percentile rank in class?
b. Failure of some students
d. Outputs
a. He scored better than 60% of the
c. Normal curve of standard
class 119. What does it mean to say that the
distribution
facility index of a test item is .50?
b. He scored less than 60% of the class
d. Student achievement relative to
a. It is reliable
c. He got 40% of the test wrongly other students
b. It is valid
123. What cognitive domain is involved in c. More from the upper answered the 130. Among general categories of
the student’s clarifying information from test correctly assessment methods, what instruments did
conclusion? pre-school teacher Justine use when he
d. The test could not discriminate
rated the handwriting of his students using
a. Synthesis between the upper and lower group
a prototype handwriting model?
b. Evaluation 127. What is the deviation from a
a. Product rating scale
standard or desired level of performance?
c. Analysis
b. Performance test
a. A problem
d. Application
c. Written response instruments
b. A deficit
124. Which of the following indicates a
d. Observation and self-reports
strong negative correlation? c. A defect
131. On what should teacher’s evaluation
a. -75 d. A gap
of a learner’s work be based?
b. -15 128. How does a student’s 80 percentile
i. Attendance
score interpreted?
c. -10
ii. Merit
a. High in all the skills being tested
d. -25
iii. Quality of academic performance
b. Higher than 80% of the members of
125. What is the graphic illustration for
the group iv. Behavior in class
the relationship between two variables?
c. Better relative to the competencies a. I and II
a. Histogram
targeted
b. II, III, and IV
b. Normal curves
d. 80% of the specified content
c. II and III
c. Frequency polygons
129. Of the types of validity for tests,
d. I, II, III, and IV
d. Scatter diagram what is focused on the extent to which a
particular tests correlates with acceptable 132. Self-evaluation can be done in
126. What does a negative discrimination
measure of performance? various ways, but this is not one of them:
index mean?
a. Curricular validity a. Use of an evaluation instrument
a. The test item has low reliability
b. Content validity b. Written reflection
b. More from the lower group
answered the test item correctly c. Criterion validity c. Self-videotape of class performance
d. Predictive validity d. Per feedback session
133. In her test, Teacher Marian R d. [Student’s score + Highest possible 139. What is an alternative assessment
unknowingly gave clues to the answers that score] x 100 tool that consists of a collection of work
reduce usability of the test. What was artifacts or in progress accomplishment by a
136. What can be said of student
wrong with the test? targeted clientele?
performance in a positively skewed score
a. Ambiguity distribution? a. Evaluation instrument
b. Unclear directions a. A few students performed b. Rubric
excellently
c. Poorly constructed test items c. Achievement test
b. Most students performed well
d. Test too short d. Portfolio
c. Almost all students had average
134. In preparing classroom tests, which 140. What computation did teacher
performance
of the following checklists is the LAST Panny use in getting the difference between
among steps in tests preparation? d. Most students performed poorly the highest and lowest scores in each class?
a. How are the objective items to be 137. Which is true when the standard a. Mean
scored? deviation is small?
b. Range
b. How are the test results to be a. Scores are toward both extremes
c. Standard deviation
reported?
b. Scores are spread apart
d. Median
c. How I have prepared a table of
c. Scores are tightly bunched together
specifications? 141. Which measure of central tendency
d. The bell curve is relatively fat is most reliable when scores are extremely
d. How are the test scores to be
high and low?
tabulated? 138. In her tests, Teacher Tomden made
tests that were either too difficult or too a. Cannot be identified unless
135. What formula is used to total and
easy. What was wrong with her tests? individual scores are given
compute test scores at the end of the year?
a. Unclear directions b. Median
a. [Test scores = transmutation table] x
100 b. Inappropriate level of difficulty of c. Mode
the test items
b. [Highest score + Lowest possible d. Mean
score] x 100 c. Ambiguity
142. Which measure of central tendency
c. [Student’s score x 100] d. Identifiable patterns of answers is most reliable to get a picture of the class
performance whose raw scores in a quiz 145. On the test giver’s list of Do’s, which a. The increase in the amount of
are: 97, 95, 85, 86, 77, 75, 50, 10, 5, 2, 1? of the following is not relative to motivating practices does not at all affect the number
students to do their best? of errors
a. Mode
a. Read test directions b. As the amount of practice increases,
b. None. It is best to look at individual
the number of errors decreases
scores b. Reduce test anxiety, e.g. “Take a
deep breath.” c. The decrease in the amount of
c. Mean
practice sometimes affects the number of
c. Explain the purpose of the test
d. Median errors
d. Tell students: “I will be proud of you
143. Self-evaluation has become an d. Decrease in the amount of practice
if you perform well.”
important kind of performance assessment goes with decrease in the number of errors
among teachers, useful as an honest self- 146. What is the range if the score
149. An entering college would like to
criticism and a starting point to removal distribution is: 98, 93, 93, 93, 90, 88, 87, 85,
determine which course is best suited for
evaluation by supervisors, peers, or 85 , 85, 70, 51, 34, 34, 34, 20, 18, 51, 12, 9,
him. Which test is appropriate for this
students. How is self-evaluation described? 8, 6, 3, 1?
purpose?
a. Evidence of teaching performance a. 93
a. Aptitude test
b. Substitute to supervisor’s rating b. 85
b. Intelligence test
c. Guide for self-adjustment c. 97
c. Achievement test
d. Tool for salary adjustment d. Between 51 and 34
d. Diagnostic test
144. What is the common instrument 147. What does the test mean if the
150. Which of the following criteria is the
used in measuring learning in the affective difficulty index is 1?
basis for selecting tests that yield similar
domain?
a. Very difficult results when repeated over a period of
a. Multiple choice time?
b. Missed by everyone
b. Checklist a. Efficiency
c. Very easy
c. Scaling b. Validity
d. A quality item
d. Questionnaire c. Usability
148. What is the meaning of a negative
correlation between amount of practice d. Reliability
and number of errors in tennis?
Principles and Methods of Teaching, b. progressivism has Dante done that is against the
Educational Technology, Curriculum guidelines for using punishment?
c. behaviorism
Development
a. Punishing immediately in an
d. reconstructionism
151. Facilities such as classrooms, emotional state
fixtures, and equipment can often damage 154. Teacher Nancy is directed to pass an
b. Using double standards in punishing
the morale of new teachers and become an undeserving student with a death threat.
obstacle for adapting well to the school Which advise will a utilitarian give? c. Doing the impossible
environment. What should be the policy for
a. Don’t pass him. You surely will not d. Holding a grudge and not starting
assigning said physical facilities?
like someone to give you a death threat in with a clean slate
a. needs of student’s basis order to pass
157. Following the principles for
b. position ranking basis b. Pass the student. That will be off use punishing students, which of the following
to the student, his parents and you. is the LEAST desirable strategy for
c. first-come, first-served basis
classroom management?
c. Pass the student. Why suffer the
d. service seniority basis
threat? a. Punishing while clarifying why
152. There are various functions a fellow punishment is done
d. Don’t pass him. Live by your
teacher or peer coach can help new
principle of justice. You will get reward, if b. Punishing while angry
teachers. What role does a peer coach play
not in this life, in the next!
by being present/available to share ideas, c. Punishing the erring student rather
problems and success with a new teacher? 155. In what setting is differentiated and than the entire class
multi-lingual teaching most effective?
a. a provider of technical feedback d. Give punishment sparingly
a. special children with classes
b. a facilitator of strategies 158. According to the guidelines on
b. multi-grade classes punishment, what does it mean that the
c. an analyzer of teaching job
teacher should give the student the benefit
c. children with diverse cultural
d. a close peer or companion of the doubt?
backgrounds
153. Teacher Princess sees to it that her a. Make sure facts are right before
d. pre-school children
classroom is clean and orderly so her pupils punishing
will less likely disarrange seats and litter on 156. After the embarrassing incident,
b. Doubt the incident really happened
the floor. On which thought is her action Teacher Kevin vowed to himself to flunk the
based? student at the end of the school term. What c. Don’t punish and doubt
effectiveness of punishment
a. existentialism
d. Get the side of the students when c. Internalizing student’s image of d. Flip-flop
punishing themselves
165. Of subcategories of teacher
159. Which of the following guidelines for d. Teacher-student relationship movement behavior, what is happening
punishment may be done? when the teacher is too immersed in a small
162. What is the term for the leap from
group of students or activity, thus ignoring
a. Don’t punish students outside of theory to practice in which the teacher
other students or activity?
school rules on punishment applies theories to effective teaching
methods and theories? a. Truncation
b. Don’t threaten the impossible
a. Integration process b. Flip-flop
c. Don’t use double standards for
punishing b. Informational process c. Stimulus-bounded
d. Don’t assign extra homework c. Conceptualization process d. Thrust
160. For group guidance in classroom d. Construction process 166. From classroom management
management, what element is lacking when strategies applied on erring students, which
163. Of subcategories of movement
there is too much competitiveness and of the following should not be done?
behavior, what is happening when the
exclusiveness with the teacher being
teacher ends an activity abruptly? a. Surprise quiz
punitive and partial to some students?
a. Thrust b. Communicating problems to parents
a. Dissatisfaction with classroom work
b. Truncation c. Parent-principal conference
b. Poor interpersonal relations
c. Stimulus-bounded d. Shaming erring student before the
c. Poor group organization
class
d. Flip-flop
d. Disturbance in group climate
167. Among mistaken goals in the
164. Of subcategories of teacher
161. To demonstrate here authority Acceptance Approach to discipline, what
movement behavior, what is happening
Teacher Kokeyni made an appeal to happens when students defy adult by
when the teacher goes from topic or
undisciplined students. What kind of appeal arguing, contradicting, teasing, temper
activity to other topic or activities, lacking
did she make by saying, “Ladies and tantrums, and low level hostile behavior?
clear direction and sequence of activities?
gentlemen, don’t engage in that kind of
a. Power seeking
behavior, you can do much better?” a. Truncation
b. Withdrawal
a. Invoke peer reaction b. Dangle
c. Revenge seeking
b. Exert authority c. Thrust
d. Attention getting b. Continuity student-centered classroom. What is this
kind of quality content?
168. Teacher Ann Patuan dealt effectively c. Inclusion of elements
with a minor infraction of whispering by a a. Utility
d. Fluency
student to a neighbor during class. Which of
b. Balance
the following did she do? 171. Teacher Aldub makes certain
content interesting to his students. c. Self-sufficiency
a. Reprimand quietly
Focusing on learners, he also uses many
d. Interest
b. Continue to teach and ignore simple examples, metaphors and stories.
infraction What is this quality of lesson content? 174. In the implementation of the
curriculum at the classroom level, effective
c. Reprimand student after class a. Interest
strategies are called “Green”. Which of the
d. Use nonverbal signals (gesture or b. Feasibility following belongs to the Green Flag?
facial expression)
c. Self-sufficiency a. Homogenous students grouping
169. What mistake is teacher Senemin
d. Balance b. Content delivery based on lessons
Basic trying to avoid by never ignoring any
student or group of students in her 172. Teaching English, teacher Krizzy is c. Excess in chalkboard talk
discussions and other activities? careful about her lesson content. What
d. Student interest and teacher
quality of content did he achieve when she
a. Non-direction enthusiasm
made certain her information came with
b. Dangled activity the “information explosion” which she got e. Rigidity if movement
in the Internet, such as how to effectively
c. Divided attention 175. In the implementation of the
teach phonetics?
curriculum at the classroom level,
d. Abrupt end
a. Learnability ineffective strategies are called “Red”.
170. Teacher Dra D Explorer is a great Which of the following belongs to the Red
b. Significance
lecturer and so she is invited to speak and Flag?
represent the school on many occasions. c. Balance
a. Content applied to real-life
What is one quality of her lecturers when
d. Interest situations
she follows a planned sequence, not
diverting so as to lose attention of her 173. Teacher Kevin made certain his b. Overemphasis on drill and practice
listeners? lesson content can be useful to his
c. Available enrichment activities
students, taking care of their needs in a
a. Explicit explanations
d. Integration of problem solving
176. Teacher Maggie explains by spicing 179. In avoiding implying sickness or various pictures, charts, graphs, videos to
her lectures with examples, descriptions suffering, which of the following is the most support her lectures?
and stories. What is this quality in her preferable way to refer to those with
a. Simplified vocabulary
lectures? disabilities like polio?
b. Enrichment through visual aids
a. Planned sequence a. “Is polio-stricken”
c. Causal and logical relationships
b. Elaboration through elements b. “Had polio”
d. Continuing sequence
c. Use of audiovisuals c. “Polio victim”
183. In determining the materials and
d. Simple vocabulary d. “Suffers from polio”
media to use, what consideration did
177. Can technology take the place of the 180. If threat of punishment is necessary Teacher Ina A. Mag adopt when he chose
teacher in the classroom? Select the most on erring students, how should this best be materials that can arouse and sustain in
appropriate answer: done? curiosity?
a. No. It is only an instrument or a tool a. Make the threat and reinforce with a. Satisfaction
warning
b. Yes, when they hire less teachers b. Interest
and acquire more computers b. Make the threat with immediate
c. Expectancy
punishment
c. Yes. When teachers are not
d. Relevance
competent c. Ward and threat at the same time
184. Which of the following is true of a
d. Yes, such as in the case of d. First a warning before the threat
democratic classroom?
Computer-assisted instruction (not teacher-
181. Among cognitive objectives, what is
assisted instruction) a. Teacher acts as firm decision maker
also known as an understanding and is a
178. What kind of tool is technology as step higher than more knowledge of facts? b. Students decide what and how to
evidenced by its use in word processing learn
a. Comprehension
databases, spreadsheets, graphics design
c. Consultation and dialogue
and desktop publishing? b. Analysis
d. Suggestions are sent to higher
a. Analyzing tool c. Synthesis
officials for decisions
b. Encoding tool d. Application
185. This is appropriate use of technology
c. Productivity tool 182. What is the quality of teacher Pining which can unite people of the world rather
Garcia’s lecture when she makes use of than exploit them?
d. Calculating tool
a. For pornography a. Teachers should keep track of I. Provide motivation and draw
learning outcomes commitments
b. For social media
b. Teachers should value information II. Explain rationale and objectives
c. For financial fraud
c. Teachers should document III. Provide feedback
d. For propaganda
information data on students
IV. Practice for mastery
186. From structures in Multifunctional
d. Teachers should teach and test
Cooperative Learning, which involves each a. II, I, IV, and III
learning
student writing in turn one answer as a
b. IV, I, II and III
paper and pencil is passed around the 189. In order to assist new teacher, which
group? is the most effective way to clarify the c. I, IV, III, and II
schools’ goals and responsibilities early in
a. Jigsaw d. I, II, IV and III
the first year?
b. Inside-outside circle 192. Teacher JanJan made certain his
a. Student’s handbook
lesson content is within the capacity of his
c. Roundtable
b. Orientation young forum grade learners. What is the
d. Partners quality of John’s lesson content when he
c. Principals’ memorandum
fits lesson to learner’s capacity to absorb
187. How does the “humaneness” of the
d. School curriculum lesson content?
teacher best described when he/she is full
interest and enthusiasm in the work of 190. Of components of direct instruction, a. Learnability
teaching? which involves teachers and students
b. Balance
working together on a skill or task and
a. Responsiveness
figuring out how to apply the strategy? c. Validity
b. Perceptiveness
a. Consolidation d. Interest
c. Knowledge
b. Guided practice 193. From structures of Multifunctional
d. Sensitivity Cooperative Learning, which makes each
c. Application
group to produce a group product to share
188. Teacher Lester Cruz Valdez gets
d. Modeling with the whole class?
more information about how his students
learn in order to upgrade his pedagogy. 191. In direct/expositive instruction, a. Coop-coop
What principle is he following? what is the logical pattern of procedures in
b. Think-pair-share
a lesson adopted?
c. Team Word-Webbing
d. Partners a. Heuristic learning b. Self-sufficiency
194. This is the more appropriate b. Problem-solving learning c. Feasibility
understanding of technology in education?
c. Discovery learning d. Balance
a. Methods and process
d. Expository learning 199. In determining materials and media
b. Inventions and equipment to use, what consideration did Teacher
197. What is another quality of teacher
Grachie adopt when she gave importance
c. Channels and instruments Lassie Pecson’s lectures when she used
to the level of outcome and the learner’s
words that are within the grasp of her
d. Hardware, designs, and sense of fulfillment in performing the task?
listeners, avoiding technical terms and
environment
jargons? a. Expectancy
195. A teacher introduces herself as
a. Use of specific descriptions and b. Satisfaction
teacher only. What does this imply?
examples
c. Interest
a. She must have been forced to
b. Enriched audiovisuals
pursue a career in teaching. d. Relevance
c. Normal vocabulary
b. The teaching profession is not a very 200. In the inductive approach to
significant one d. Planned sequence learning, what is not among the facilitating
skills needed on the part of the teacher?
c. The teaching profession is the 198. In delivering her lessons, teacher
lowest paid profession Blackie Lou Blanco is careful that no topic is a. Teacher giving generalization of
extensively discussed at the expense of principles
d. She takes no pride in the teaching
other topics. That guiding principle in
profession b. Commenting to pave way for
selection and organization of lesson content
generalizations or principles
196. In the guided exploratory approach is she following?
to learning, which is not the term used for c. Organizing answers
a. Significance
Inquiry learning?
d. Asking the right questions
FINAL REVIEW b. Learning activities that involve problems 7. I drew learners into several content
of classification and ordering. areas and encouraged them to solve a
complex question for Inter-disciplinary
c. Games and other physical activities to
1. A guest speaker in one graduation rites teaching. Which strategy did I use?
develop motor skills.
told his audience: "Reminder, you are what
A. Problem-centered learning
you choose to be." The guest speaker is d. Stimulating environment with ample
more of a/an . objects to play with. B. Unit method
a. Realist b. Pragmatist 5. Study this group of tests which was C. Reading-writing activity
administered with the following results,
c. Idealist d. Essentialist D. Thematic instruction
then answer the question.
2. A teacher who equates authority with 8. The concepts of trust vs. maturity,
Subject Mean SD Ronnels Score
power does NOT . autonomy vs. self-doubt, and initiative vs.
Math 56 10 43 guilt are most closely related with the
a. Shame b. develop self-respect in
works of
every pupil Physics 41 9 31
A. Erikson B. Piaget C. Freud
c. retaliate d. intimidate English 80 16 109
D. Jung
3. Based on Edgar Dale's Cone of In which subject(s) did Ronnel perform best
9. I want to teach concepts, patterns and
Experience, which activity is farthest from in relation to the group's performance?
abstractions. Which method is most
the real thing?
A. Physics and Math b. English c. appropriate?
a. Read b. Hear c. View images d. Physics d. Math
A. Indirect instruction B. Discovery
Attend exhibit
6. Who among the following puts more
C. Direct instruction D. Problem solving
4. Based on Piaget's theory, what should a emphasis on core requirements, longer
teacher provide for children in the concrete school day, longer academic year and more 10. Which behavioral term describes a
operational stage? challenging textbooks lesson outcome in the highest level of
Bloom's cognitive domain?
a. Activities for hypothesis formulation. A. Perennialist B. Essentialist
A. Create B. Evaluate C. Analyze
C. Progressivist D. Existentialist
D. Design
11. A sixth grade twelve-year old boy comes A. LET passers D. Revise lesson objectives
from a dysfunctional family and has been
B. Duly licensed professionals 17. Which is a sound classroom
abused and neglected. He has been to two
management practice?
orphanages and three different elementary C. Possess dignity and reputation
schools. The student can decide on the A. Avoid establishing routines
D. With high-moral values as well as
second grade level, but he can comprehend
technical and professional competence B. Establish routines for all daily needs and
orally material at the fourth or fifth grade
tasks.
level. The most probable cause/s of this 15. Teacher Q does not want Teacher B to
student's reading problem is/are. be promoted and so writes an anonymous C. Apply rules and policies on a case to case
letter against Teacher B accusing her of basis.
A. emotional factors B. poor teaching
fabricated lies Teacher Q mails this
D. Apply reactive approach to discipline.
C. neurological factor D. Immaturity anonymous letter to the Schools Division
Superintendent. What should Teacher 18. What does extreme authoritarianism in
Q do if she has to act the home reinforce in learners?
12. Principal B tells her teachers that professionally?
A. Doing things on their own initiative
training in the humanities is most
A. Submit a signed justifiable criticism
important. To which educational B. Ability to direct themselves.
against Teacher B, if there is any.
philosophy does he adhere? C. Dependence on others for direction.
B. Go straight to the Schools Division
A. Existentialism B. Perennialism Superintendent and gives criticism verbally. D. Creativity in work.
C. Progressivism D. Essentialism C. Hire a group to distribute poison letters 19. In a criterion-referenced testing, what
against Teacher B for information must you do to ensure that your test is fair?
13. Which is the first step in planning an
dissemination.
achievement test? A. Make all of the questions true or false.
D. Instigate student activists to read poison
A. Define the instructional objective. B. Ask each student to contribute one
letters over the microphone.
question.
B. Decide on the length of the test.
16. As a teacher, what do you do when you
C. Make twenty questions but ask the
C. Select the type of test items to use. engage yourself in major task analysis?
students to answer only ten of their choice.
D. Build a table of specification. A. Test if learning reached higher level
D. Use the objectives for the units as guide
thinking skills.
14. In the Preamble of the Code of Ethics of in your test construction.
Professional Teachers, which is NOT said of B. Breakdown a complex task into sub-skills.
teachers?
C. Determine the level of thinking involve
20. Teacher M's pupils are quite weak A. Some pupils are admittedly not capable 26. If you agree with Rizal on how you can
academically and his lesson is already far of learning. contribute to our nation's redemption,
behind his time table. which should you work for?
B. Every pupil has his own native ability and
How should Teacher M proceed with his his learning is limited to this native ability. A. Opening our doors to foreign influence
lesson?
C. Every child is a potential genius. B. Upgrading the quality of the Filipino
A. Experientially B. Inductively through education
D. Pupils can possibly reach a point where
C. Logically D. Deductively they have learned everything. C. Stabilizing the political situation
21. Soc exhibits fear response to freely 24. What should you do if a parent who is D. Gaining economic recovery
roaming dogs but does not show fear when concerned about a grade his child received
27. Teacher A is directed to pass an
a dog is on a leash or confined to a pen. compared to another student's grade,
undeserving student with a death threat.
Which conditioning process is illustrated demands to see both students' grades?
Which advice will a hedonist give?
A. Generalization B. Extinction A. Refuse to show either record.
A. Pass the student. Why suffer the threat?
C. Acquisition D. Discrimination B. Show both records to him.
B. Don't pass him. You surely will not like
22. Teacher P wants to develop the skill of C. Refuse to show any record without someone to give you a death threat in order
synthesizing in her pupils. Which one will expressing permission from principal. to pass.
she do?
D. Show only his child's records. C. Don't pass him. Live by your principle of
A. Ask her students to formulate a justice. You will get reward, if not in this
25. Rights and duties are correlative. This
generalization from the data shown in life, in the next!
means that .
graphs.
D. Pass the student. That will be of use to
A. rights and duties regulate the
B. Ask her students to answer questions the student, his parents and you.
relationship of men in society
beginning with What if ...
28. Are percentile ranks the same as
B. rights and duties arise from natural law
C. Tell her pupils to state data presented in percentage correct?
graphs. C. each right carries with it one or several
A. It cannot be determined unless scores
corresponding duties
D. Directs her students to ask questions on are given.
the parts of the lesson not understood. D. rights and duties ultimately come from
B. It cannot be determined unless the
God
23. Studies in the areas of neurosciences number of examinees is given.
disclosed that the human brain has limitless
C. No D. Yes
capacity. What does this imply?
29. Teacher W wants to review and check C. Programmed instruction D. Inductive B. No. This may exert undue influence ori
on the lesson of the previous day? Which reasoning the members of the promotional staff and
one will be most reliable? so may fail to promote someone on the
32. Which test has broad sampling of topics
basis of merit.
A. Having students identify difficult as strength?
homework problems. C. Yes. The rare invitation will certainly be
A. Objective test B. Short
welcomed by an overworked promotional
B. Having students correct each other's answer test
staff.
work.
C. Essay test D. Problem type
D. Yes. There's nothing wrong with sharing
C. Sampling the understanding of a few
33. Teacher B uses the direct instruction one's blessings.
students.
strategy. Which sequence of steps will she
35. Which illustrates a developmental
D. Explicitly reviewing the task-relevant follow?
approach in guidance and counseling?
information necessary for the day's lesson.
I. Independent practice
A. Spotting on students in need of guidance
II. Feedback and correctiveness
B. Teaching students how to interact in a
30. To elicit more students’ response,
III. Guided student practice positive manner
Teacher G made use of covert responses.
Which one did she NOT do? IV. Presenting and structuring C. Acting as a mediator
A. She had the students write their V. Reviewing the previous day's work D. Making the decision for the confused
response privately. student
A. V-II-IV-III-I B. III-II-IV-I-V
B. She showed the correct answers on 36. The test item "Group the following
C. V-lV-III-II-I D. I-V-II-III-IV
the overhead after the students have items according to shape" is a thought test
written their responses. 34. Teacher H and Teacher I are rivals for item on .
promotion. To gain the favor of the
C. She had the students write their A. creating B. classifying
promotional staff, Teacher I offers her
responses privately then called each of
beach resort for free for members of the C. generalizing D. comparing
them.
promotional staff before the ranking. As
D. She refrained from judging on the one of the contenders for promotion, is this
student's responses. becoming of her to do? 37. Direct instruction is for facts, rules,
and actions as indirect instruction is for
31. Which is a form of direct instruction? A. Yes. This will be professional growth for
, , ,.
the promotional staff.
A. Discovery process B. Problem
solving
A. hypotheses, verified data and 41. Out of 3 distracters in a multiple choice 45. From whom do we owe the theory of
conclusions test item, namely B, C, and D, no pupil deductive interference as illustrated in
chose D as answer. syllogisms?
B. concepts, patterns and abstractions
This implies that D is A. Plato B. Socrates C. Aristotle
C. concepts, processes and generalizations
D. Pythagoras
A. an ineffective distracter B. a vague
D. guesses, data and conclusions
distracter 46. To come closer to the truth we need to
38. Which group of philosophers maintains go back to the things themselves. This is the
C. an effective distracter D. a
that truth exists in an objective order that is advice of the
plausible distracter
independent of the knower?
A. behaviorists B. phenomenologists
42. Which describes norm-referenced
A. Idealist B. Pragmatists
grading? C. idealists D. pragmatists
C. Existentialists D. Realist
A. The performance of the group 47. Which is NOT a sound purpose for
asking questions?
B. What constitutes a perfect score
39. On whose philosophy was A. S. Neil's A. To probe deeper after an answer is given.
C. The students' past performance
Summerhill, one of the most experimental
B. To discipline a bully in class.
schools, based? D. An absolute standard
C. To remind students of a procedure.
A. Rousseau B. Pestalozzi 43. Which Filipino trait works against the
shift in teacher's role from teacher as a D. To encourage self-reflection.
C. Montessori D. John Locke
fountain of information to teacher as
48. Your teacher is of the opinion that the
40. Teacher H strives to draw facilitator?
world and everything in it are ever changing
participation of every student into her
A. Authoritativeness B. Authoritarianism and so teaches you the skill to cope with
classroom discussion. Which student's
change. What is his governing philosophy?
need is she trying to address? The need C. Hiya D. Pakikisama
A. Idealism B. Existentalism
A. to show their oral abilities to the rest of 44. Teacher G's lesson objective has
the class something to do with the skill of C. Experimentalism D. Realism
synthesizing? Which behavioral term is
B. to be creative
most appropriate?
C. to feel significant and be part of a group 49. Research tells that teachers ask mostly
A. Test B. Assess C. Appraise
content questions. Which of the following
D. to get everything out in the open D. Theorize
terms does NOT
refer to content question? D. engage his students in convergent B. must be reactive in our approach to
thinking discipline
A. Closed B. Direct C. Concept
D. Convergent 53. Based on Piaget's theory, what should a C. have to Resolve minor disruptions before
teacher provide for children in the they are out of control
50. Whose teaching is in support of
sensorimotor stage?
Education for All (EFA), he asserted that in D. may apply 9 rules out of 10 consistently
teaching there should be no distinction of A. Games and other physical activities to
56. The attention to the development of a
social classes. develop motor skill.
deep respect and affection for our rich
A. Sun Yat Sen B. Confucius B. Learning activities that involve problems cultural past is an influence of
of classification and ordering.
C. Mencius D. Lao Tsu A. Confucius B. Hegel
C. Activities for hypothesis formulation.
51. As a teacher, you are a C. Teilhard de Chardin D. Dewey
Reconstructionist. Which among these will D. Stimulating environment with ample
57. In what way can teachers uphold the
be your guiding principle? objects to play with.
highest possible standards of quality
A. I must teach the child every knowledge, 54. Teacher H gave her first-grade class a education?
skill, and value that he needs for a better page with a story in which pictures take the
future. place of some words. Which method did
she use? A. By continually improving themselves
B. I must teach the child to develop his
personally and professionally
mental powers to the full. A. The whole language approach
B. By wearing expensive clothes to change
C. I must teach the child so he is assured of B. The Spaulding method
people's poor perception of teachers
heaven.
C. The rebus method
C. By working out undeserved promotions
D. I must teach the child that we can never
D. The language experience approach
have real knowledge of anything. D. By putting down other professions to lift
55. A stitch on time saves nine, so goes the the status of teaching
52. If a teacher plans a constructivist lesson,
adage.. Applied to classroom management,
what will he most likely do? Plan how he 58. Student B claims: I cannot see
this means that we
can perfection but I long for it. So it must be
A. may not occupy ourselves with real.Under which group can he be
A. do evaluate his students' work
disruptions which are worth ignoring classified?
B. do reciprocal teaching because they are minor
A. Idealist B. Empiricist
C. lecture to his students
C. Realist D. Pragmatist
59. In Krathwohl's taxonomy of objectives in B. lt brings together the information that 66. Principal C shares this thought with his
the affective, which is most authentic? has been discussed teachers: Subject matter should help
students understand and appreciate
A. Characterization B. C. it makes provisions for full participation
themselves as unique individuals who
Organization of students.
accept complete responsibility for their
C. Responding D. Valuing D. it clinches the basic ideas or concepts of thoughts, feelings, and actions. From which
the lesson. philosophy is this thought based?
60. NSAT and NEAT results are interpreted
against set mastery level. This means that 63. In which competency do my students A. Perennialism B.
NSAT and NEAT find the greatest difficulty? In the item with Essentialism
a difficulty index of
fall under . C. Existentialism D. Progressivism
A. 0.1 B. 0.9 C. 0.5
A. intelligence test B. aptitude 67. Whose influence is the education
D. 1.0
test program that puts emphasis on self-
64. Which method has been proven to be development. through the classics, music,
C. criterion-referenced test D. norm-
effective in courses that stress acquisition and rituals?
referenced test
of knowledge?
A. Buddha B. Mohammed C.
61. Each teacher is said to be a trustee of
A. Socratic method B. Cooperative Confucius D. Lao Tsu
the cultural and educational heritage of the
learning
nation and is, under obligation to transmit 68. Teacher A is a teacher of English as a
to learners such heritage. Which practice C. Mastery learning D. Indirect Second Language. She uses vocabulary
makes him fulfill such obligation? instruction cards, fill-in-the-blank sentences, dialogues,
dictation and writing excercises in teaching
A. Use the latest instructional technology. 65. Why should a teacher NOT use direct
a lesson about grocery shopping. Based on
instruction all the time?
B. Observe continuing professional this information, which of the following is a
education. A. It requires much time. valid conclusion?

C. Use interactive teaching strategies. B. It requires use of many supplementary A. The teacher is applying Bloom's hierachy
materials. of cognitive learning.
D. Study the life of Filipino heroes.
C. It is generally effective only in the B. The teacher is teaching in a variety of
62. A teacher's summary of a lesson serves
teaching of concepts and abstractions. ways because not all students learn in the
the following functions, EXCEPT
same manner.
D. It reduces student’s engagement in
A. it links the parts of the lesson
learning. C. The teacher wants to make herteachirig
easier by having less talk.
D. The teacher is emphasizing reading and C. Both students are functioning in the 75. Which criterion should guide a teacher
writing skills. average range of intellectual ability in the choice of instructional devices?
69. Which one may support equitable D. Another IQ test should be given to truly A. Attractiveness B. Cost
access but may sacrifice quality? assess their intellectual potential
C. Novelty D. Appropriateness
A. Open admission B. School 72. Helping in the development of
76. What is most likely to happen to our
accreditation graduates who a maka-Diyos is an influence
economy when export continuously
of
C. Deregulated tuition fee hike D. surpasses import is a thought question on
Selective retention A. naturalistic morality B. classical .
Christian morality
70. Teacher A discovered that his pupils are A. creating B. relating cause-and-
very good in dramatizing. Which tool must C. situational morality D. dialectical morality effect
have helped him discover his pupils'
73. Teacher B clears his throat to C. synthesizing D. predicting
strength?
communicate disapproval of a student's
77. The teacher's first task in the selection
behavior. Which specific influence
of media in teaching is to determine the
technique is this?
A. Portfolio assessment .
A. Signal interference B. Direct
B. Performance test A. choice of the students B. availability
appeal
of the media
C. Journal entry
C. Interest boosting D. Proximity
C. objectives of the lesson D. technique
D. Paper-and-pencil test control
to be used
71. Two students are given the WISE II. One 74. In the context on the theory on multiple
78. All men are pretty much alike. It is only
has a full scale IQ of 91, while the other has intelligences, what is one weakness of the
by custom that they are set apart, said one
an IQ of 109. paper-pencil test?
Oriental philosopher. Where can this
Which conclusion can be drawn? A. It is not easy to administer. thought be most inspiring?

A. The second student has significantly B. It puts the non-linguistically intelligent at A. In a multi-cultural group of learners
higher intellectual ability a disadvantage.
B. In multi-cultural and heterogeneous
B. The first student is probably below C. It utilizes so much time. groups of learners and indigenous peoples'
average, while the second has above group
D. It lacks reability.
average potential
C. In a class composed of indigenous
peoples
79. If teacher has to ask more higher-order A. Program for Decentralized Education 86. Which teaching activity is founded on
questions, he has to ask more Bandura's Social Learning Theory?
B. School-Based Management
questions.
A. Lecturing B. Modeling
C. Basic Education Curriculum
A. closed B. fact C. concept D.
C. Questioning D. Inductive
convergent D. Schools First Initiative
Reasoning
80. Student Z does not study at all but when 83. Which technique should a teacher use
87. In mastery learning, the definition of an
the Licensure Examination for Teachers to encourage response if his students do
acceptable standard of performance is
(LET) comes, before he takes the LET, he not respond to his question?
called a
spends one hour or more praying for a
A. Ask a specific student to respond, state
miracle, i.e. to pass the exam. Which A. SMART B. criterion measure
the question, and wait a response.
attitude towards religion or God is
C. behavior D. condition
displayed? B. Tell the class that it will have detention
unless answers are forthcoming. 88. Students' scores on a test were: 72, 72,
A. Religion as fake
73, 74, 76, 78, 81, 83, 85. The score 76 is
C. Ask another question, an easier one.
B. Religion as magic the .
D. Wait for a response.
C. Religion as authentic A. mode B. average C. mean
84. Standard deviation is to variability as D. median
D. Religion as real
mean is to
89. Test norms are established in order to
81. Teacher T taught a lesson denoting
A. coefficient of correlation B. central have a basis for .
ownership by means of possessives. He first
tendency
introduced the rule, then gave examples, A. establishing learning goals
followed by class exercises, then back to the C. discrimination index D. level
B. interpreting test results
rule before he moved into the second rule. of difficulty
Which presenting technique did he use? C. computing grades
85. Quiz is to formative test while periodic
A. Combinatorial B. Comparative is to D. identifying pupils' difficulties
C. Part-whole D. Sequence A. criterion-reference test 90. Which behavior is exhibited by a
student who is strong in interpersonal
82. All subjects in Philippine elementary and B. summative test
intelligence?
secondary schools are expected to be
C. norm-reference test
taught using the integrated approach. This A. Works on his/her own.
came about as a result of the D. diagnostic test
B. Keeps interest to himself/herself.
implementation of .
C. Seeks out a classmate for help when C. treat all learners alike while in the according to Erikson's theory on
problem occurs. classroom psychological development?
D. Spends time meditating. D. prepare modules for slow learners in A. Autonomy B. Trust
class
91. All of the following describe the C. Initiative D. Generativity
development of children aged eleven to 94. Value clarification as a strategy in
97. Which type of report refers toon-the-
thirteen EXCEPT Values Education classes is anchored on
spot description of some incident, episode
which philosophy?
A. they shift from impulsivity to adaptive or occurrence that is being observed and
ability A. Existentialism B. Christian recorded as being of possible significance?
philosophy
B. sex differences in IQ becomes more A. Autobiographical report
evident C. Idealism D. Hedonism
B. Biographical report
C. they exhibit increase objectivity in 95. Teacher E discussed how electricity
C. Value and interest report
thinking flows through wires and what generates the
electric charge. D. Anecdotal report
D. they show abstract thinking and
judgement Then she gave the students wires, bulbs, 98. The main purpose of compulsory study
switches, and dry cells and told the class to of the Constitution is to
92. A student passes a research report
create a circuit that will increase the
poorly written but ornately presented in a A. develop students into responsible,
brightness of each bulb. Which one best
folder to make up for the poor quality of thinking citizens
describes the approach used?
the book report content. Which Filipino
B. acquaint students with the historical
trait does this practice prove? Emphasis on A. It used taxonomy of basic thinking skills
development of the Philippine Constitution
.
B. It was contructivist
C. make constitutional experts of the
A. art over academics B. substance over
C. It helped students understand scientific students
porma
methodology
D. prepare students for law-making
C. art over science D. porma over
D. It used cooperative learning
substance 99. The following are used in writing
96. Rodel is very aloof and cold in his performance objectives, EXCEPT
93. The principle of individual differences
relationships with his classmates. Which
requires teachers to . A. delineate B. diagram
basic goal must haye not been attained by
A. give greater attention to gifted learners Rodel during his developmental years, C. integrate D. comprehend
B. provide for a variety of learning activities
100. Which can effectively measure 104. Teacher F wanted to teach the A. Mean is greater than the median
students' awareness of values? pupils the skill to do cross stitching. Her
B. Median is greater than mean
checkup quiz was a written test on the
A. Projective techniques B. Moral
steps of cross stitching. Which characteristic C. Scores have three modes
dilemma
of a good test does it lack?
D. Scores are normally distributed
C. Likert scales D. Anecdotal record
A. Scorability B. Reliability C. Objectivity
108. Which measure(s) of central tendency
101. The first thing to do in constructing a D. Validity
separate(s) the top half of the group from
periodic test is for a teacher to
105. In self-directed learning, to what the bottom half?
A. decide on the number of items for the extent should a teacher's scaffolding be?
A. Median B. Mean
test
A. To a degree the student needs it.
C. Median and Mean D. Mode
B. go back to her instructional objectives
B. None, to force the student to learn by
109. What was the most prominent
C. study the content himself.
educational issue of the mid 1980s?
D. decide on the type of test to construct C. To the minimum, to speed up
A. Bilingual Education B. Values
development of student's sense of
Education
independence.
102. Teacher F is convinced that C. Accountability D.
D. To the maximum, in order to extend to
whenever a student performs a desired Mainstreaming
the student all the help he needs.
behavior, provided reinforcement and soon
110. How can you exhibit expert power on
the student will learn to perform the 106. What can be said of Peter who
the first day of school?
behavior on his own. On which principle is obtained a score of 75 in a Grammar
Teacher F's conviction based? objective test? A. By making them feel you know what you
are talking about.
A. Cognitivism B. Environmentalism A. He answered 75 items in the test
correctly. B. By making them realize the importance
C. Behaviorism D. Constructivism
of good grades.
B. He answered 75% of the test items
103. Teacher U teaches to his pupils that
correctly. C. By reminding them your students your
pleasure is not the highest good. Teacher's
authority over them again and again.
teaching is against what philosophy? C. His rating is 75.
D. By giving your students a sense of
A. Realism B. Hedonism D. He performed better than 5% of his
belonging and acceptance.
classmates.
C. Epicureanism D. Empiricism
107. Which applies when skewness is zero?
111. Which types of play is most RICKY: WE COULD STUDY DEBRIS FROM THE C. Behaviorist D. Cognitivist
characteristic of a four to six-year old child? METEORITES IF WE CAN FIND ANY.
116. In a study conducted, the pupils were
A. Solitary and onlooker plays TEACHER: THOSE ARE ALL GOOD ANSWERS. asked which nationality they preferred, if
BUT WHAT IF THOSE EXCURSIONS TO THE given a choice.
B. Associative and cooperative plays
MOON, TO THE CENTER OF THE EARTH, OR
Majority of the pupils wanted to be
C. Associative and onlooker plays TO FIND METEORITES WERE TOO COSTLY
Americans. In this case, in which obligation
AND TIME CONSUMING? HOW MIGHT WE
D. Cooperative and solitary plays relative to the state, do schools seem to be
USE THE ELEMENTS WE ALREADY HAVE
failing? In their obligation to
112. For which may you use the direct HERE ON EARTH TO FIND SOME NEW
instruction method? ONES? A. respect for all duly constituted
authorities
Question: The Teacher's questions in the
above exchange are examples of B. promote national pride
A. Become aware of the pollutants around
questions.
us. C. promote obedience to the laws of the
A. fact B. concept C. direct state
B. Appreciate Milton's Paradise Lost.
D. closed
D. instill allegiance to the Constitution
C. Use a microscope properly.
114. The following are sound specific
117. Read this question: How will you
D. Distinguish war from aggression. purposes of questions EXCEPT
present the layers of the earth to your
113. Read the following then answer the A. to call the attention of an inattentive class? This is a question that
question student
A. directs B. leads the student
TEACHER: IN WHAT WAYS OTHER THAN THE B. to teach via student answers to evaluate
PERIODIC TABLE MIGHT WE PREDICT THE
C. to stimulate leamers to ask questions C. assesses cognition D. probes creative
UNDISCOVERED ELEMENTS?
thinking
D. to arouse interestand curiosity
BOBBY: WE COULD GOTO THE MOON AND
118. A mother gives his boy his favorite
SEE IF THERE ARE SOME ELEMENTS THERE 115.Teacher B engages her students with
snack everytime the boy cleans up his
WE DON'T HAVE. BETTY: WE COULD DIG information for thorough understanding for
room. Afterwards, the boy cleaned his room
DOWN INTO THE CENTER OF THE EARTH meaning and for competent application.
every day in anticipation of the snack.
AND SEE IF WE FIND ANY OF THE Which principle governs Teacher B's
Which theory is illustrated?
practice?
MISSING ELEMENTS
A. Associative Learning B. Classical
A. Contructivist B. Gestalt
Conditioning
C. Operant Conditioning D. Pavlonian 121. Shown a picture of children in A. I must teach the child that we can never
Conditioning sweaters inside the classroom, the students have real knowledge of anything.
were asked this question: "In what kind of
119. What should a teacher do for students B. I must teach the child to develop his
climate do these children live?" This is a
in his class who are not on grade level? mental powers to the full.
thought question on
A. Give them materials on their level and let C. I must teach the child so he is assured of
A. inferring B. applying C. creating
them work at a pace that is reasonable for heaven.
D. predicting
them, trying to bring them up to a grade
D. I must teach the child every knowledge,
level. 122. In his second item analysis, Teacher H
skill, and value that he needs for a better
found out that more from the lower group
B. Give them the same work as the other future.
got the test item # 6 correctly. This means
students, because they will absorb as much
that the test item . 125. Which guideline in test construction
as they are
is NOT observed in this test item: Jose
A. has a negative discriminating power
capable of. Rizal wrote.
B. has a lower validity
C. Give them the same work as the other A. The central problem should be packed in
students, not much, so that they won't feel C. has a positive discriminating power the stem
embarrassed.
D. has a high reability B. There must be only one correct answer.
D. Give them work on the level of the other
123. Teacher A knows of the illegal activities C. Alternatives must have grammatical
students and work a little above the
of a neighbor but keeps quiet in order not parallelism.
classmates level to
to be involved in any investigation. Which
D. The alternates must be plausible.
challenge them foundational principle of morality does
Teacher A fail to apply? 126. I combined several subject areas in
120. Which holds true to standardized
order to focus on a single concept for inter-
tests? A. The end does not justify the means.
disciplinary teaching. Which
A. They are used for comparative purposes B. The principle of double-effect strategy/method did I use?
B. They are administered differently C. Always do what is right. A. Problem-entered learning B. Thematic
instruction
C. They are scored according to different D. Between two evils, do the lesser evil.
standards C. Reading-writing activity D. Unit
124. As a teacher, you are a rationalist.
method
D. They are used for assigning grades Which among these will be your guiding
principle? 127. Which is a major advantage of a
curriculum-based assessment?
A. It is informal in nature. B. shaming students. nominator.What kind of objective is the
latter?
B. It connects testing with teaching. C. ridiculing students.
A. Major B. Terminal C. Enabling
C. It tends to focus on anecdotal D. intimidating students.
D. Primary
information on student progress.
131. Which is one role of play in the pre-
135. Which is/are the sources of man's
D. It is based on a norm-referenced school and early childhood years?
intellectual drives, according to Freud?
measurement model.
A. Develops competitive spirit.
A. Id B. Superego C. Id and ego
128. Based on Edgar Dale's Cone of
B. Separates reality from fantasy. D. Ego
Experience, which activity is closest to the
real thing? C. Increases imagination due to expanding 136. Which is an appropriate way to
knowledge and emotional range. manage off-task behavior?

D. Develops the upper and lower limbs. A. Make eye contact.


A. View images B. Attend exhibit
132. During the Spanish period, what B. Stop your class activity to correct a child
C. Watch a demo D. Hear
was/were the medium/media of instruction who is no longer on task.
129. Teacher Y does norm-referenced in schools?
C. Move closer to the child.
interpretation of scores. Which of the
A. The Vernacular B. English
following does she do? D. Redirect a child's attention to task and
C. Spanish D. Spanish and the check his progress to make sure he is
A. She describes group performance in
Vernacular continuing to work.
relation to a level of mastery set.
133. An effective classroom manager uses 137. John Watson said: Men are built not
B. She uses a specified content as its frame
low-profile classroom control. Which is a born. What does this statement point to?
of reference.
low-profile classroom technique?
A. The ineffectiveness of training on a
C. She compares every individual student’s
A. Note to parents B. After-school person's development.
scores with others' scores.
detention
B. The effect of environmental stimulation
D. She describes what should be their
C. Withdrawal of privileges D. Raising the on a person's development.
performance.
pitch of the voice
C. The absence of genetic influence on a
130. The typical autocratic teacher
134. The primary objective of my lesson person's development
consistently does the following EXCEPT
is:To add similar fractions correctly.Before I
D. The effect of heredity.
A. encouraging students. can do this I must first aim at this specific
objective:To distinguish a numerator from a
138. Which does NOT belong to the group Which one can renew a teacher's teachers must be preoccupied only with
of alternative learning systems? enthusiasm? teaching?
A. Multi-grade grouping A. Stick to job A. Yes, if they are given other assignments,
justice demands that they be properly
B. Multi-age grouping B. Initiate changes in jobs
compensated.
C. Graded education C. Judge someone else as wrong
B. Yes, because other community leaders,
D. Non-graded grouping D. Engage in self-pity not teachers, are tasked to leadin
community activities.
139. Which activity should a teacher have 143. With indirect instruction in mind,
more for his students if he wants them to which does NOT belong to the group? C. NO, because every teacher is expected to
develop logical- mathematical thinking? provide leadership and initiative in activities
A. Problem solving B. Lecture-recitation
for betterment of communities.
C. Inductive reasoningD. Discovery
D. Yes, because teaching is enough full time
A. Problem solving B. Choral reading
144. History books used in schools are job.
C. Drama D. Storytelling replete with events portraying defeats and
146. "In the light of the facts presented,
weaknesses of the
140. A goal-oriented instruction culminates what is most likely to happen when ... ?" is
in . Filipino as a people. How should you tackle a sample thought question on
them in the classroom?
A. planning of activities A. inferring B. generalizing
A. Present them and express your feelings
B. evaluation C. synthesizing D. justifying
of shame.
C. identification of topics 147. Which is a true foundation of the social
B. Present facts and use them as means in
order?
D. formulation of objectives inspiring your class to learn from them.
A. Obedient citizenry
141. A teacher/student is held responsible C. Present them and blame those people
for his actions because s/he _. responsible or those who have contributed. B. The reciprocation of rights and duties

A. has instincts B. is mature D. Present them as they are presented, and C. Strong political leadership
tell the class to accept reality.
C. has a choice D. has reason D. Equitable distribution of wealth
145. Teachers often complain of numerous
142. The burnout malady gets worse if a 148. For maximum interaction, a teacher
non-teaching assignments that adversely,
teacher doesn't intervene to change ought to avoid questions.
affect their teaching. Does this mean that
whatever areas he or she can control.
A. informational B. rhetorical
C. leading D. divergent said that the LET possesses A. I and III B. II only C. I only
validity. D. I and II
149. It is not wise to laugh at a two-year old
child when he utters bad word because in A. concurrent B. construct 155. In a social studies class, Teacher I
his stage he is learning to . presents a morally ambiguous situation and
C. content D. predictive
asks his students what they would do. On
A. consider other's views B.
153. Teacher F is newly converted to a whose theory is Teacher I's technique
distinguish sex differences
religion. Deeply convinced of his new found based?
C. socialize D. distinguish right religion, he starts Monday classes by
A. Kohlberg B. Bandura
from wrong attacking one religion and convinces his
pupils to attend their religious services on C. Piaget D. Bruner
150. Which guideline must be observed in
Sundays. Is this in accordance with the Code
the use of prompting to shape the correct 156. You arrive at knowledge by re-thinking
of Ethics of Professional Teachers?
performance of your students? of latent ideas. From whom does this
A. Yes. What he does is values education. thought come?
B. No. A teacher should not use his position A. Experimentalist B. Realist
A. Use the least intrusive prompt first.
to proselyte others.
C. Idealist D. Existentialist
B. Use all prompts available.
C. Yes. In the name of academic freedom, a
157. A child who gets punished for stealing
C. Use the most intrusive prompt first. teacher can decide what to teach.
candy may not steal again immediately. But
D. Refrain from using prompts. D. Yes.What he does strengthens values this does not mean that the child may not
education. steal again. Based on Thorndike's theory on
151. Bruner's theory on intellectual
punishment and learning, this shows that
development moves from enactive to iconic 154. Which does Noam Chomsky, assert
and symbolic stages. about language learning for children? A. punishment strengthens a response

In which stage(s) are diagrams helpful to I. Young children learn and apply B. punishment removes a response
accompany verbal information? grammatical rules and vocabulary as they
C. punishment does not remove a response
are exposed to them.
A. Enactive and iconic B. Symbolic
D. punishment weakens a response
II. Begin formal teaching of grammatical
C. Symbolic and enactive D.
rules to children as early as possible. 158. After giving an input on a good
Iconic
paragraph, Teacher W asks her students
III. Do not require initial formal language
152. If your Licensure Examination Test to rate a given paragraph along the
teaching for children.
(LET) items sample adequately the elements of a good paragraph. The
competencies listed in the syllabi, it can be students' task is in level of
A. application B. analysis D. Technical problem D. Individual teachers giving weights to
factors considered for rating
C. evaluation D. synthesis 162. Ruben is very attached to his mother
and Ruth to her father. In what 165. Based on Freud's psychoanalytic
159. Which is most implied by a negatively
developmental stage are they according to theory which component(s) of personality is
skewed score distribution?
Freudian psychological theory? (are) concerned with a sense of right and
A. The scores are evenly distributed from wrong?
A. Oedipal stage B. Latent
left to the right
stage A. Super-ego B. Super-ego and Ego
B. Most pupils are achievers C. Id D. Ego
C. Anal stage D. Pre-genital stage
C. Most of the scores are low 166. Which assumption underlies the
163. Behavior followed by pleasant
teacher's use of performance objectives?
D. Most of the scores are high consequences will be strengthened and will
be more likely to occur in the future. A. Not every form of learning is observable.
160. How can you exhibit referent power on
Behavior followed by unpleasant
the first day of school? B. Performance objectives assure the
consequences will be weakened and will be
learrier of learning.
less likely to be repeated in the future.
Which one is explained? C. Learning is defined as a change in the
A. By making them feel you know what you
learner's observable performance.
are talking about. A. Freud's Psychoanalytic Theory
D. The success of learner is based on
B. By telling them the importance of good B. Thorndike's Law of Effect
teacher performance.
grades.
C. B. F. Skinner's Operant Conditioning
167. Who among the following needs
C. By reminding your students your Theory
less verbal counseling but needs more
authority over them again and again.
D. Bandura's Social Learning Theory concrete and operational forms of
D. By giving your students a sense of assistance? The child who .
164. Which one can enhance the
belonging and acceptance.
comparability of grades? A. has mental retardation
161. If teacher wants to test students'
A. Using common conversion table for B. has attention-deficit disorder
ability to organize ideas, which type of
translating test scores in to ratings
test should she formulate? C. has learning disability
B. Formulating tests that vary from one
A. Multiple-choice type D. has conduct disorder
teacher to another
B. Short answer
C. Allowing individual teachers to determine
C. Essay factors for rating
168. We encounter people whose prayer D. take place over a long period of time A. Have hands that write fast.
goes like this: "O God, if there is a God; save
171. In Krathwohl's affective domain of B. Have eyes on the back of your heads.
my soul, if I
objectives, which of the following is the
C. Have a mouth ready to speak.
have a soul" From whom is this prayer? lowest level of affective behavior?
D. Have minds packed with knowledge.
A. Stoic B. Empiricist A. Valuing B. Characterization
C. Agnostic D. Skeptic C. Responding D. Organization
176. Teacher B is a teacher of English as a
169. The best way for a guidance 172. With specific details in mind, which
Second Language. She uses vocabulary
counselor to begin to develop study one has (have) a stronger diagnostic value?
cards, fill-in-the- blank sentences, dictation
skills and habits in underachieving student
A. Multiple choice test and writing exercises in teaching a lesson
would be to .
about grocery shopping. Based on this
B. Non-restricted essay test
information, which of the following is a
C. Restricted essay test valid conclusion?
A. has these underachieving students
observe the study habits of excelling D. Restricted and non-restricted essay tests A. The teacher is reinforcing learning by
students giving the same information in, a variety of
173. What is the mean of this score
methods.
B. encourage students to talk about study distribution: 4, 5, 6, 7, 8, 9, 10?
habits from their own experiences B. The teacher is applying Bloom's hierarchy
A. 7 B. 6 C. 8.5 D. 7.5
of cognitive learning.
C. has them view film strips about various
174. Availment of the Philippine Education
study approaches C. The teacher wants to do less talk.
Placement Test (PEPT) for adults and out-
D. gives out a list of effective study of-school youths is in support of the D. The teacher is emphasizing listening and
approaches government’s educational program towards speaking skills.
.
170. To promote effective practice, which 177. With synthesizing skills in mind, which
guideline should you bear in mind? Practice A. equitable access B. quality has the highest diagnostic value?
should be
C. quality and relevance D. A. Essay test B. Performance test
A. done in an evaluative atmosphere relevance
C. Completion test D. Multiple choice
B. difficult for students to learn a lesson 175. With-it-ness, according to Kounin, is tests
one of the characteristics of an effective
C. arranged to allow students to receive 178. How can you exhibit legitimate power
classroom manager. Which phrase goes
feedback on the first day of school?
with it?
A. By making your students feel they are 181. Which of the following propositions is TEACHER: THOSE ARE ALL GOOD ANSWERS
accepted for who they are. attributed to Plato? BUT WHAT IF THOSE, EXCURSIONS TO THE
MOON, TO THE CENTER OF THE EARTH, OR
B. By informing them you are allowed to act A. Truth is relative to a particular time and
TO FIND METEORITES WERE TOO COSTLY
in loco parentis. place.
AND TIME CONSUMING? HOW MIGHT WE
C. By making them realize the importance B. Human beings create their own truths. USE THE ELEMENTS WE ALREADY HAVE
of good grades. HERE ON EARTH TO FIND SOME NEW
C. Learning is the discovery of truth as
ONES?
D. By making them feel you have mastery of Latent ideas are brought to consciousness.
subject matter. Question: Which questioning strategy/ies
D. Sense perception is the most accurate
does/do the exchange of thoughts above
guide to knowledge.
illustrate?
179. Which questioning practice promotes 182. In the parlance of test construction
A. Funneling B. Sowing and
more class interaction? what does TOS mean?
reaping
A. Asking the question before calling on a A. Table of Specifics B. Table of
C. Nose-dive D. Extending and
student. Specifications
lifting
B. Focusing on divergent question C. Table of Specific Test Items D.
184. Referring to Teacher S, Nicolle
Team of Specifications
C. Focusing on convergent questions. describes her teacher asfair, caring and
183. Read the following then answer the someone you can talk to.Which power or
D. Asking rhetorical questions.
question: leadership does Teacher S have?
180. Theft of school equipment like tv,
TEACHER: IN WHAT WAYS OTHER THAN THE A. Referent power B. Legitimate power
computer, etc. By teenagers in the
PERIODIC TABLE MIGHT WE PREDICT THE
community itself is becoming a common C. Reward power D. Expert
UNDISCOVERED ELEMENTS?
phenomenon. What does this incident power
signify? BOBBY: WE COULD GO TOTHE MOON AND
185. By what name is Indirect instruction
SEE IF THERE ARE SOME ELEMENTS THERE
A. Prevalence of poverty in the community. the Socratic method also known?
WE DON'T HAVE.
B. Inability of school to hire security guards. A. Mastery learning B. Indirect
BETTY: WE COULD DIG DOWN TO THE
Method
C. Deprivation of Filipino schools. CENTER OF THE EARTH AND SEE IF WE FIND
ANY OF THE MISSING ELEMENTS. C. Morrison method D.
D. Community's lack of sense of co-
Questioning method
ownership. RICKY: WE COULD STUDY DEBRIS FROM THE
METEORITES IF WE CAN FIND ANY.
186. In a treatment for alcoholism, Ramil A. Personality tests B. prophecies. What is this phenomenon
was made to drink an alcoholic beverage Performance tests called?
and then made to ingest a drug that
C. Paper-and-pencil tests A. Halo effect B. Pygmalion effect
produces nausea. Eventually, he was
nauseated at the sight and smell of alcohol D. Standardized test C. Ripple effect D. Hawthorne effect
and stopped drinking alcohoL Which theory
190. In instructional planning it is necessary 194. Under which program were students
explains this?
that the parts of the plan from the first to who were not accommodated in public
the last have . elementary and secondary schools because
of lack of classroom, teachers, and
A. Operant conditioning
instructional materials, were enrolled in
B. Social Learning Theory A. clarity B. symmetry C. coherence private schools in their respective
D. conciseness communities at the government's expense?
C. Associative Learning
191. The cultivation of reflective and A. Government Assistance Program
D. Attribution Theory
meditative skills in teaching is an influence
B. Study Now-Pay Later
187. The search for related literature by of .
accessing several databases by the use of a C. Educational Service Contract System
A. Shintoism B. Zen Buddhism
telephone line to connect a computer
D. National Scholarship Program
library with other computers that have C. Confucianism D. Taoism
database is termed 195. How would you select the most fit in
192. With which goals of educational
government positions? Applying Confucius
A. compact disc search institutions as provided for by the
teachings, which would be the answer?
Constitution is the development of work
B. manual search
skills aligned? A. By course accreditation of an accrediting
C. on-line search body
A. To develop moral character
D. computer search B. By merit system and course accreditation
B. To teach the duties of citizenship
188. Which one can best evaluate students' C. By merit system
C. To inculcate love of country
attitudinal development?
D. By government examinations
D. To develop vocational efficiency
A. Essay test B. Portfolio
196. How can a teacher enhance his/her
193. Researchers conducted show that
C. Observation D. Short answer test questioning technique for an effective
teacher's expectations of students become.
teacher-student interaction?
189. Which are direct measures of Do not require initial formal language
competence? teaching for children self-fulfilling A. Immediately
B. You may answer your own question if no A. SMART C. To the minimum, to speed up
one can answer development of student's sense of
B. Condition
independence.
C. Allow sufficient “think time” at least 7 to
C. Criterion Measure
10 seconds D. To the maximum, in order to extend to
D. Behavior the student all the help he needs
D. Extend wait time until the students
respond 200. Teacher Luke clears his throat to 203. Which is the first step in planning an
communicate disapproval of a student’s achievement test?
197. A teacher who advocates the
behavior. Which specific influence
pragmatic philosophy of education believes A. Select the type of test items to use.
technique is this?
that experience should follow learning thus
B. Decide on the length of the test.
she has to? A. Proximity control
C. Define the instructional objective
A. Equip her students with basic skills and B. Interest boosting
abilities D. Build a table of specification
C. Signal Inference
B. Encourage her students to memorize 204. Which may NOT be a benefit derived
D. Direct appeal
facts from the use of graphic organizers?
201. The main purpose of compulsory study
C. Provide her students opportunities to A. Make relationships among detail clear
of the Constitution is to __________
apply their skills and abilities
B. Enable students to identify important
A. develop students into responsible,
D. require her students mastery of the ideas and details
thinking citizens
lesson
C. Strengthen team work
B. acquaint students with the historical
198. Respect for honest differences of
development of the Philippine Constitution D. Represent stated information in concrete
opinions is one objective
form
C. makes constitutional experts of the
A. Human relationship
students 205. Rights which cannot be renounced or
B. self-actualization transferred because they are necessary for
D. prepare students for law-making
the fulfillment of man’s primordial
C. Civic responsibility and conscience
202. In self-directed learning, to what obligations are called
D. Economic self sufficiency extent should a teacher's scaffolding be?
A. Perfect rights B. Alienable
199. In mastery learning, the definition of A. To a degree the student needs it. rights
an acceptable standard of performance is
B. None, to force the student to learn by C. Acquired rights D. Inalienable rights
called
himself.
206. Principal Miguel shares this thought D. Promotion and improvement of social B. Numerous studies show how individual
with his teachers: "Subject matter should and economic status of public school develop
help students understand and appreciate teachers
C. Early development is more critical than
themselves as unique individuals who
209. the 19th century saw the development later development
accept complete responsibility for their
of state controlled and state supported
thoughts, feelings, and actions." From D. Social expectations influence
public school system. Existing ideology was
which philosophy is this thought based? development in early stage
the concept of national sovereignty. Under
A. Perennialism a democracy, education equips citizens with 212. Which questioning technique would be
the principles and duties of citizenship to appropriate for inductive lessons?
B. Essentialism
guarantee national stability. Such is the
A. Use questions requiring only memory
C. Existentialism ideology of __________ .
responses
D. Progressivism A. Citizenship B. Progressivism
B. Expect participation only among the
207. How can a teacher establish the C. Nationalism D. Education more motivated students
reliability of a test?
210. The following are the characteristics of C. Involve students actively in the
1) Repeat the same test a good teaching/learning practice EXCEPT? questioning process
2) Administer a parallel test A. Students are governed by fixed and rigid D. as teacher, you ask no questions
standards
3) Split the test 213. What principle is reflected when a
C. Life-like situations introduced as learning teacher always provides for the
4) Vary the number of items
experience in the classroom development of all essential knowledge
A. 1, 2 and 3 B. 2,3 and 4 manipulative skills and attitudes?
C. Students are given more opportunity to
C. 1 and 4 only D. 2 and 4 only act or experience learning. A. Principle of Needs B. Principle of
Balance
208. On which policy is R.A. 4670 known as D. Evaluation is made as an integral part of
the Magna Carta for Public Teachers teaching procedures. C. Principle of Unity D. Principle of
focused? organization
211. Which of the following is NOT a
A. Right to establish or join an organization principle of development? 214. A teaching method proceeds from the
details of a lesson towards generalization is
B. Code of ethics for professional teachers A. Development follows an orderly,
called:
predictable sequence
C. Recruitment shall take place after
A. Deductive B. Inductive
training
C. Debate D. Problem solving 218. The sociogram reveals that four 221. The _________ method is used when
students is your class formed a “clique”. the learners are made to observe things in a
215. To accomplish many good activities in
This means that the teacher should: certain place like the market.
a particular class period, the teacher must
A. Allow them to be together all the time A. Participatory B. Case study
B. Encourage them to join the groups C. Simulation D. Field trip
A. Add curricular activities B. Provide
unique teaching aids C. Discourage them from joining other 222. The recommended method to use if
groups the teachers wishes each learner to
C. Institute a systematic plan D. Use
concentrate in learning a topic to his skills
expensive material D. disband the group
are properly assessed is.
216. one way to encourage students who
A. Semantic webbing
lack in reading to love reading is to:
219. Parents negatively react to the
B. Independent study
A. Reprimand him B. Ask the students to inclusion of play in children’s curriculum. A
buy books teacher who knows the importance of play C. Role playing
would tell the parents that __________?
C. Provide reading materials D. Isolate Him D. Field trip
A. Playing is important in the child’s
217. Which of the following style or 223. The ________ method is observed if
cognitive development
technique would make the teacher a good we wish the pupil learn from real life
classroom manager– teacher with eyes at B. playing is a good form of exercise situation dilemmas.
the back of the head?
C. Playing gives children pleasure A. Situation B. Lecture
A. Give your students the “head to toe”
D. Playing is a stage of child’s development C. Textbooks D. Observation
look when angry
220. The ability to perceive how objects are 224. The ________ method is used to make
B. Uses eye contact technique with your
related in order to mentally perceive what the learners study in detail a specific thing,
students
is seen, thus creating concrete visual images person or place not known to them
C. Fear and fight dominates the classroom from memory refers to
A. Case study B. Participation
atmosphere
A. Visual spatial intelligence
C. Project D. Field trip
D. Being aware of all the actions and
B. Musical Intelligence
activities in the classroom 225. Which of the following method will
C. Linguistic intelligence you used to verify a certain findings and to
make the learners handle apparatus
D. Logical reasoning intelligence
properly?
A. Textbook method B. Laboratory 230. The ________ approach is utilized 235. To enable the learners to learn by their
method when the learners are trained to ask own pace of growth, _________ approach is
intelligent question. used.
C. Field trip method D. Project
method A. Process B. Discovery A. Inquiry B. Discovery C. Mastery
D. Modular
C. Inquiry D. Value certification
236. The ______ approach is observed
226. The ________ method is utilized if the 231. If you wish to relate a subject matter
when the learners want to meet the
learners are trained to do creative products. to one of the four principles of learning, the
criterion level of success act set by the
________ approach should be used.
A. Project B. Case study C. Field trip teachers.
D. Simulation A. Multidisciplinary B. Interdisciplinary
A. Mastery learning B. Interdisciplinary
227. If the material is dangerous for the C. Conceptual D. Integration
C. Conceptual D. Modular
learners to handle, which of the following
232. The ________ approach is used if the
method will you use? 237. Which approach is used to emphasize
teacher wishes to solve a problem being
the skills in informing conclusions?
A. Textbook B. Group met in the school.
discussions A. Multidisciplinary B. Interdisciplinary
A. Mastery B. Integration
C. Lecture-demonstration D. Eclectic C. Conceptual D. Inquiry
C. Action learning D. Value clarification
228. Which method is used to develop 238. Which of the following approaches is
233. You wish to make the learners learn or
scientific inquiry among the learners? used to include issues confronting the
internalized fully a subject matter to be
societies?
A. Project B. Case study taught to them. Which of the following will
you need? A. Integration B. Interdisciplinary
C. Problem solving D. Simulation
A. Integrated B. Multi-media C. Conceptual D. Inquiry
229. The ________ method is used to find
out the learner’s knowledge about a certain C. Master D. Multi-disciplinary 239. The _______ approach is used to make
topic assigned to them. the learners enunciate their feelings or
234. Which of the following approaches will
attitudes about certain issues.
A. Independent study B. Textbook be used if you wish to relate a particular
subject to all disciplines of learning? A. Value clarification B. Mastery
C. Lecture D. Question and
answer A. Multidisciplinary B. Interdisciplinary C. Integration D.
Interdisciplinary
C. Value clarification D. Integration
240. The _________ method is used if the 244. Young children have a short attention B. Daily attendance record of each student.
learners are to use their senses effectively. and interest span. What kind of task should
C. Grade obtained by each student in all
the teacher give them?
A. Textbook B. Lecture his/her subjects.
demonstration A. Challenging and interesting activities
D. Personal data of the students in the
C. Observation D. Independent study B. Long but interesting activities registrar.
241. The most important criterion in test C. Easy and difficult activities 248. Which type of test is used to discover
construction is further attitudes about self and
D. Short, varied, interesting activities
A. Preparation of table of specifications
245. What is possessed by the learner when
B. Congruency of the items with objectives he can use language with ease and fluency A. Personality test B. Intelligence test
in any given situation?
C. Stem should contain the central problem C. Achievement test D. Diagnostic test
A. Bilingual ability B. High cognitive
D. Options should be almost the same
skills
length
249. Which behavior is exhibited by a
C. A photographic memory D.
242. Which of the following is the most student who is strong in interpersonal
Communicative competence
important contribution of Gestalt intelligence?
psychology to the theories of learning? 246. What is the main concern of spiraling a
A. Works by his/her own.
curriculum?
A. Cognitive insight
B. Spends time meditating.
A. curriculum renewal and revision
B. The use of multimedia approaches
C. Keeps interest to himself/herself.
B. Horizontal articulation among the
C. The concept of readiness in learning
students in a grade level D. Seeks out a classmate for help when
D. The use of reinforcement problem occurs.
C. Vertical articulation of a given subject
243. What psychological principle is used across a grade level 250. A parent visited you regarding his son’s
when teacher links the new information to low grades. He showed you his quizzes, unit
D. Incorporating government thrusts and
the previous one to enable the students to tests and projects. You discovered that his
societal concerns
gain a holistic view of the topic? name is Mel and you wrongfully put his
247. Which of this information is not name on the girl’s list. What will you do?
A. Stimulation B. Accommodation
entered in Form 1 or the School Registrar?
A. Recognize your mistake and promise to
C. Assimilation D. conceptualization
A. Alphabetical list of students, boys correct the grade
separated from girls.
B. Insists: that you are right in grading him. B. Refer them to a doctor. 256. Social adjustment means the ability to
behave in accordance with?
C. Refer the matter to the principal C. Give them remedial class D. Suspend
them from classes A. Universal truths
D. Ignore the complain
253. Children learn what they live by. Treat B. Self-concept
them with respect and they will respect
C. Stereotyped behaviors
251. What should a cooperating teacher do others. Shout at them and they will be
to help the student teacher who has been shouting at others, too. How would you D. Social expectations
assigned to him/her? explain this behavior?
257. How can a teacher avoid “breakdown
A. Provide opportunities for the student A. They are easily impressed B. They are and interruptions” I daily class procedures?
teacher to acquire the skills and imitative
competencies to be an effective teacher.
C. they cannot tell right from wrong D.
A. Punish the misbehaving students
B. Show your lesson plan and let him/her They are observant
follow what is in your lesson plan. B. Establish routine for daily tasks
254. What should be done with a student in
C. Write or prepare activities to be done the upper grades who is a non-reader? C. Assign a leader to assist everyone
and let him/her execute these in class
A. Encourage him to join a reading club D. Allow students to make their own
D. Don’t allow him/her to make her own regulations
B. Give him comic books
decisions as to how the lesson is to be
258. Which is made after certain norms has
introduced. C. Have him attend remedial reading class
been established?
252. Which of these techniques is BEST D. Transfer him to lower section
A. Departmental test B. Local-city
suited to developing skill in asking and
255. In which situation is learning most wide test
answering questions?
likely to happen?
C. Teacher-made test D.
A. Interviews B. Song analysis
A. When students work by themselves Standardized test
C. Pictorial review D. Riddles
B. When students are quiet and well 259. A high school principal would like to
252. Most delinquents are found to have behaved know the causes of drop-outs in his school
low IQ. They can hardly read and so he could find solutions to this problem.
C. When all the needed materials are
comprehend. How can a teacher help What type of researched is used?
available
them?
A. Applied B. Action
D. When students know the importance of
A. Call for their parents.
the task at hand C. Pure D. Experimental
260. If we aim to produce globally B. Consistently follow schedule for D. Intelligences hinges in physical structure
competitive graduates, the Philippine classroom routine
267. Which of the following activities is
education should give major emphasis to
C. Rush if you are getting late for the right stressed by humanistic education?
_______________.
time
A. English, Science and Mathematics
D. Have a big clock installed in the
A. Enjoy the great works of man such as
B. Technology and citizenship education classroom for everyone's guidance
classics
C. Humanities and work education 264. One learns by association and also by
B. Learn the philosophy “know thyself”
insight. This shows that the association and
D. Bilingual education and values education
cognitive theories of learning are: C. Make the distinctly civilized, educated
261. Which of the following can be and refined
A. Diametrically opposed B.
considered a form of civic engagement?
Complementary D. Develop man into a thinking individual
A. Patronizing Filipino products
C. Partly wrong D. 268. For a teacher to be competent, he/she
B. Critical of government officials Partly correct is required to specialize in certain area. This
pillar of learning is
C. Bribing government officials 265. Mothers who demand their 3 to 5 year
old child to suspend their time in serious A. Learning to do B. Learning to
D. Electoral participation
academic study, forget that early childhood live together
262. A teacher wants to make a rubric for is in the
C. Learning to be D. Learning to
scoring students' output. Which format will
A. Gang age B. Questioning age know
use differential weighs for the qualities of a
product or a performance? C. Initiative age D. Toy age 269. What pillar of learning is concerned on
material development rather than of
A. Performance based B. Rating scales 266. Identical twins are more than alike
human development?
than fraternal twins. Which of the following
C. Holistic rubric D. Analytic
statements/principles is supported by this? A. Learning to do B. Learning to live
rubric
longer
A. Heredity has a part in determining
263. How can a teacher develop the value
physical appearance C. Learning to be D. Learning to
of time such as punctuality and maximal
know
utilization of time? B. Intelligence is determined partly by pre-
natal nutrition 270. What is the advantage of using
A. Avoid disruptions due to improper
computers in processing test results?
behavior C. Environment affects both fraternal and
identical twins
A. Its processing takes a shorter period of B. Tell the parents about the condition B. Support of living models
time.
C. Stop schooling till after she gave birth C. Pressure of work
B. Test results can easily be assessed.
D. Direct her to an abortion clinic D. High salary
C. Its statistical computation is accurate.
274. Which describes an inappropriate 278. I cannot help but recall the sister’s
D. All of the above. practice in the education of young children? convent which served as my boarding house
in high school now that I am in a noisy
271. Multiple choice test is considered the A. Individual differences are expected and
boarding house. Which principle of
BEST type of test because accepted
association explains this?
_______________
B. Integrated teaching-learning
A. Similarity B. Contiguity
A. It is easy to conduct
C. Isolated skills development
C. Frequency D. Contrast
B. It contains many responses
D. positive guidance techniques
C. measure several competencies in one
275. Honesty remains a value even if
test 279. Which principle of association ass
nobody in a organization values it. This
applied to memory is this? The recall of an
D. It possess the qualities of other types of pronouncement comes from the mouth of
object or idea triggers recall of other
tests a(an)
objects like it
272. While serving during the elections, A. Pragmatist B. Idealist
A. Contrast B. Contiguity C. Similarity
some trouble makers enter your precinct.
C. Reconstructionist D. Progressivist D. Frequency
What do you think is the most appropriate
thing to do? 276. I cannot forget my friend’s birthday for 280. Whose thought is this: “Although there
it comes one day after my birthday. Which is an external world from which human
A. Challenge them to a duel
principle of association as applied to beings acquire sensory information, ideas
B. Close the precinct and go home memory explains this? originate from the workings of the mind.”

C. Ignore them and look for a safe place A. Contiguity B. Similarity A. Idealist B. Realist C. Empiricist
D. pragmatist
D. Seek the assistance of law enforcers C. Frequency D. Content
281. The process by which certain
273. Which of the following will you 277. A fresh teacher graduate is usually
potentials are inherited from the parents
recommend to a senior high school scholar idealistic. Which one will most likely inspire
for his development
who is impregnated by a fellow student? her to cling to her idealism?
A. Life B. Birth C. Heredity D.
A. Force her boyfriend to marry her A. Introduction of educational innovations
Character
282. This theory states that there are 8 286. The ability of a child to conceptualize D. assumes that human activities are based
basic development stages that the the retention and preservation of the same on stimulus and response
individual has to pass through his life quantity under various changes.
290. These statements imply that children
A. Learning Theory at the early learning stage consider parents
and teachers as authorities and models.
B. Psychoanalytic Theory A. Recognition B. Reversibility
A. Parents and teachers should always
C. Psychosocial Theory C. Assimilat8ion D. Conservation
coordinate children’s activities
D. Cognitive Development 287. Refers to the idea that no individual
B. Parents should enforce strict discipline at
are exactly the same or alike.
283. Transition age from childhood to home and teachers in school
adulthood where rapid physical changes A. Cognitive theory B. Exclusivity
C. Parents and teachers should be the role
and sex maturity occur resulting in changes theory
models at all times
in ways of feelings, thinking and acting.
C. Individual Differences D.
D. Parents and teachers should always
A. Puberty B. Adolescence Emotional Quotient
consult each other with regards the child’s
C. Early Adulthood D. Stage V 288. He is known as the Father of Modern intellectual development
I.Q. Test
284. Modifying an existing scheme after an 291. Any change in the behavior of an
individual’s interaction with the A. Lewis Terman B. Erick individual
environment, resulting in the creation of a Erickson
A. Learning B. Response
new scheme.
C. Laurence Kohlberg D. Martin
C. Change D. Development
A. Assimilation B. Interaction Lesley
292. Which of the following principles IS
C. Recognition D. Accommodation 289.“Intellectual appreciative experience” is
NOT considered under Classical

285. Theory stating that a person’s behavior Conditioning by Ivan Pavlov?
can be motivated by urges towards self- A. Base on the premise that all learning has
A. Excitation B. Adhesive
satisfaction. emotional correlates
Principle
A. Psychoanalytic Theory B. Obtained in the field of music, art and
C. Stimulus Generalization D. None of the
literature
B. Cognitive Development Theory above
C. the acquisition and retention of facts and
C. Psychosocial Theory 293. This stimulation of action best explains
information
the behavior of an individual to take what
D. Moral development theory
he perceives to be the shortest route to his A. “A place where children have fun and 299. How are institutions of learning
goals. enjoyment” encouraged to set higher standards over
and above the minimum requirement for
B. “Garden where children could grow and
state recognition?
develop”
A. Recognition B. Assimilation
A. Scholastic achievement B. Faculty
C. “The learning center of life”
C. Response D. Motivation development
D. “A place where new beginnings begin”
294. The process by which an individual C. Academic freedom D. Voluntary
acquires the social and cultural heritage of 297. Which of the following statements is accreditation
the society where he belongs. given emphasis by “humanistic education?”
300. The period of physical, especially
A. Socialization A. The great works of man such as the sexual, and mental maturation which is
classics should be enjoyed. characterized by rapid somatic growth is
B. Internalization
known as
B. Man should learn the different
C. Integration
philosophies of education A. infancy B. early childhood
D. Acquisition
C. Build a man who is distinctly civilized, C. Puberty D. Adulthood
295. What is the main function of the educated and refined
301. Claustrophobia is an irrational fear of
philosophies of education?
D. Develop man into a thinking individual
A. Darkness B. Strangers
A. To aid the leaner to build his/her own
298. A teacher who advocates the
personal philosophy. C. Closed Space D. Height
pragmatic philosophy of education believes
B. Define goals and set directions from that experience should follow learning, 302. An eye defect characterized by clear
which educational efforts should be thus, she has to? vision in one dimension but unfocused
exerted. vision on the other is called
A. require her student mastery of the
C. Provide guidelines in the foundation of lessons A. Myopia B. Astigmatism
educational policies and programs
B. encourage her students to memorize C. Hyperopia D. Presbyopia
D. Provide norms and standards for facts
303. Which of the following statements
evaluation purposes.
C. equip her students with basic skills and does not apply to adolescents?
296. According to Froebel, kindergarten is abilities
A. They desire the approval of their peers
also known as
D. provide her student with opportunities
B. They seek dependence on their parents
to apply their skills and abilities
C. They have a marked sex development D. there is no marked differences in their C. it is affected by attitude D. it is never
time of maturity constant
D. None of the above
308. Rationalization is used by student who 312. Transfer of training easily takes place if
the activities involved
304. As young people mature, society
A. Are different
expects them to develop competencies and A. Always give explanation or reason for
assume social roles in a conventional their failures rather than own their faults B. Have identical element
manner.
B. Like to take the blame for their faults C. Occur in the same place
A. expectation of parents B.
C. bribe their elders with promises D. Vary in difficulty
influence of peers groups
D. substitute words for deeds 313. When the learner is well-motivated, he
C. influence of formal education D.
performs his task
cultural demands 309. Which of the following is true of
Abnormal Psychology? A. with indifference
305. The founder of the theory of
psychology called psychoanalysis was A. It studies the cause of personality defects B. with disinterest
A. Lock B. Hume C. Freud D. B. It measures the accomplishments of the C. with arrogance
Leibnitz individual
D. with enthusiasm
306. When the learner reaches a point C. it concentrates on the scholastic
314. A six-year-old child who has a mental
where no further improvement can be performance of the individual
age of eight years has an IQ of
expected, he is in a so-called
D. it investigates the educational
A. 120 B. 130 C. 132 D. 133
A. Development crisis B. Learning plateau background of the individual
315. The ratio obtained by dividing mental
C. Regression D. Depression 310. Which of the following is a continuous
age by chronological age times 100 is called
variable?
307. Regarding the sexual maturation of
A. derived quotient B. deviation
boys and girls, teachers should bear in mind A. Weight B. Sex C. Nationality
that: D. Race C. Intelligence quotient or IQ D. Intelligence
ratio
A. Girls mature at a late stage than boys 311. Which of the following is true about
one’s IQ? 316. Which of the following was written by
B. Girls mature at an earlier stage than boys
Plato?
A. it remains fairly constant B. it is highly
C. Boys and girls mature at the same time
changeable
A. Sic et Non B. The School and 321. Hardship allowance is given to a 324. If a student asks a question which the
Society teacher when teacher does not have a ready answer, the
latter should:
C. The Republic D. Emile A. s/he’s assigned in a depressed area
A. Dismiss the question as irrelevant
B. s/he’s given additional teaching load
B. Offer a bluff
317. Who among those below asserted that C. s/he’s assigned in a barangay high school
“Education is for complete living?” C. Admit the fact that he doesn’t know the
D. s/he’s assigned in a hazardous area
answer
A. Dewey B. Spencer C. Kant
322. Which of the following principles of
D. Froebel D. Ask volunteers to answer the question
human growth refers to the situation that
and do research on it later.
318. The right of an educational institution girls mature earlier than boys?
and its faculty to prescribe the 325. The heredity traits acquired by a
A. Growth follows a sequential pattern
methods/strategies of teaching refers to: person in his lifetime
B. Development rates vary
A. Building style A. Are transmissible to his offspring
C. Each stage of development has
B. Choice of curriculum B. Reappears in his future grandparent
characteristics traits
C. Academic freedom C. Have no influence on the offspring
D. Maturation should precede certain type
D. Co and extra-curricular program of learning D. Become recessive traits
319. The 1987 Constitution provides that 323. A teacher who gives a uniform 326. When student are given a chance to
religious instruction can be given assignment to be worked out by all learners settle differences of opinion by discussion,
in Arithmetic is not observing a they develop:
A. with the students’ consent
characteristic of a good assignment. Which
A. Fair play B. Tolerance
B. with the parent/guardian approval characteristic is overlooked?
C. Irritants D. sociability
C. with mayor’s permit A. It should be definite
327. The school’s responsibility towards
D. with the school’s support B. It should be stimulating
teenagers “gang age” is:
320. Public schools in the Philippines are C. It should emphasize the essential
A. Provide the gang all the freedom it needs
the contribution of which colonizer?
D. It should provide for individual
B. Gives classroom activities to give
A. American B. British differences
direction to out-of-school youth activities
C. Japanese D. Spanish
C. Supervise gang activities
D. set up norms of conduct or the member 331. The need to recognize and develop C. Language D. Logical
of the gang special sensitivity to language, thus helping reasoning
the learners to use the right word, phrase
335. The capacity to analyze one’s feelings
and/or graph to grasp new meaning refers
and thus be able to understand and be able
328. In an intelligence test, a 13-year old girl to
to know the motives of other people’s
got a score equivalent to that of a 15-year
A. Visual intelligence B. Linguistic actions.
old. This means:
Intelligence
A. Spatial B. Personal
A. That the girl must be accelerated
C. Feelings sensitivity D. Jargon
C. Logical D. Diametric
B. That the girl 12-years older mentally
332. The sensitivity to tone and pitch,
C. That the girl has a chronological age of 15 allowing one to produce musical scoring is
intelligence in? 336. The type of intelligence which enables
D. That she has a mental age of 13
a person to understand other person’s
A. Musical B. Verbal
329. Which statement is not necessary to feelings, behavior and motivation.
Ability
achieve the learner’s interest in a learning
A. Emotional B. Spatial
activity? C. Quantitative exercises D.
Qualitative analysis C. Social intelligence D. Quantitative and
A. The activity must lead to a practical end
Qualitative
333. One’s ability to do abstract reasoning
B. The activity must be within the ability of
and manipulate symbols refers to what type 337. The type of intelligence which
the learner
of intelligence? characterizes actress, actors, mimes,
C. The activity must fill a need recognized dancers and people of the Arts?
A. Musical B. Personality
by the learner
identification A. Bodily-kinesthetic B. Scientific
D. The learner must have the experience
C. Mental ability C. Research D. Emotions
that will furnish the background for the
activity D. Mathematical-logical 338. An emerging thrust in determining
one’s personality, whether pleasant or
330. He is responsible for the theory which 334. The ability to perceive how objects are
unwholesome, this type of personality
recognizes the importance of developing related in order to mentally perceive what
measurement is the wholesomeness of
multiple intelligence? is seen, thus creating concrete visual images
one’s virtues, i.e., values, relationships with
from memory refers to?
A. Jean Piaget B. Howard Gardner other, adjustments to varying situations,
A. Visual-spatial Intelligence B. Musical behavior an motivations
C. Frederick Froebel D. Sigmund Freud
A. Emotional Quotient (E.Q.)
B. Intelligence Quotient (I.Q.) 341. Heredity has a part in determining A. Aggressive tendencies towards
intelligence. Which of the following classmates
C. Maladjustment personality
statements support this principle?
B. Poor habits in organizing work materials
D. Anticipated behavior
A. Environment affects both fraternal and
C. Interrupting a speaker
339. It is a measurement of personality identical twins
which is the result by dividing the mental D. Abandoning a project before it is finished
B. Intelligence hinges in physical structures
age by the chronological age.
345. Learning-disabled children most
C. Intelligence is determined partly by pre-
A. Emotional quotient (E.Q.) characteristically have:
natal nutrition
B. Intelligence Quotient (I.Q.) A. low IQ
D. Identical twins are more alike than
C. Multiple intelligences fraternal twins B. poor socio-economic backgrounds
D. Forecasted behavior quotient 342. Educators who contributed to the C. An average level of intelligence
“open education” movement include:
340. The teacher must be aware that both D. minimal brain damage
heredity and environment represent A. Neill and piaget B. Kohl and
346. Which of the following is true about
complex factors, exerting many specific kozol
educable mentally retarded children?
influences on an individual’s growth. Which
C. Bruner and Silberman D. All
of the following statements best represents A. Their IQ range between 50 and 70
of the above
the influence of heredity and environment?
B. They have short attention spans and
343. A child’s social skills can be measured
A. Heredity counts; environment is less experience difficulty in generalizing
by:
important.
C. Their reading, writing, and arithmetic
A. direct observation and parent-teacher
B. If the environment is changed, heredity skills cannot be improved
conferences
becomes less important.
D. A and B above
B. psychological test
C. The relative influences of heredity and
347. Which of the following are
environment can vary widely in an C. adaptive behavior scales
characteristics of a dyslexic child?
individual’s growth
D. A and C above
A. Mirror writing B. listlessness
D. In the long run, both tend to cancel each
344. A teacher uses behavioral modification
other’s influences C. Below-average intelligenceD.
techniques in his classes. Which of the
Hyperactivity
following student behaviors would he find
most difficult to change?
348. Students with secondary reading 351. During the learning process the C. A social-problem-solving meeting to
problems have capacity to read, but are teacher has most control over: resolve teacher or student problems
non-readers because of: relating to the school, the class, or any
A. The learners
individual member.
B. The learning environment
A. auditory problems
C. The Learning process
D. A sensitivity-training meeting for the
B. congenital defects
D. The behavior of the learners purpose of helping students ace their
C. visual-acuity impairment school-related problems and learn how
352. Which of the following conditions does
their actions can affect others.
D. Environmental or Emotional factor NOT contribute to a climate psychologically
suited to learning?
349. If a teacher accepts Maslow’s theory
on the hierarchy of needs, he or she will A. The teacher acts like a “real person.”
probably structure objectives to:
B. The teacher makes all of the decisions
A. Meet both physiological and intellectual about students learning activities.
needs of students
C. The teacher accepts students as they are
B. Eliminate testing
D. The teacher shows trust in students’
C. Eliminate extrinsic motivations decisions
D. maintain a certain anxiety level for 353. William Glasser advocates the frequent
increased competition use of classroom meetings, with teacher
and students sitting in a small circle. Which
350. The knowledge explosion has led to
one of the following types of discussion
crowding more and more information into
would NOT be appropriate in such a
curriculum courses. A likely result is that:
setting?
A. the textbook will no longer be the main
A. An educational-diagnostic conference on
instructional medium in many classes
the learning weaknesses of individual
B. the child may spend more time in school students
C. the teacher may have to rely more on B. .An open.-ended meeting for the purpose
these multimedia materials of exploring and discussing student’s ideas
about the curriculum.
D. all of the above
III. The three branches of government are 5. In the decade of the 70’s, one clamor of
separate and independent of one another. the activists who staged street
demonstration on
IV. The three branches of government have
a check and balance over one another. the streets was “Down with the oligarchs!”
What did they accuse government of as
a. I only
suggested by the underlined word?
b. II only
a. A rule of the few rich
c. II, III, and IV
b. A form of dictatorship
d. I, III, and IV
LET REVIEWER c. Anti-poor
3. If our present government is a
GENERAL EDUCATION- SOCIAL STUDIES democracy, where does power reside? d. Elitist

1. Which statement is TRUE of the pre- a. In the Filipino people 6. With the Batasang Pambansa performing
Spanish Filipino government? legislative and executive powers in the
b. In Congress
Marcos
a. The datu exercised all the powers of
c. In the President
government. regime, which form of government was
d. In the Supreme Court implemented?
b. Laws were formulated by a law making
body elected by the datu. 4. In President Quezon’s time, the country a. Parliamentary
had the Philippine Commonwealth. What is
c. Laws were formulated by a law making b. Dictatorial
TRUE
body elected by the community.
c. Monarchial
about the Philippine Commonwealth? The
d. There was a court created by the datu to
Filipino people _____________________. d. Presidential
hear complaints.
a. Were fully independent. 7. What form of government is
2. What characteristic/s of the government
characterized by the separation of powers?
is established by the 1987 Constitution? b. Were partially independent.
a. Parliamentary
I. Presidential system of government with c. Were granted total freedom to course
three branches. their destiny only in matters of education. b. Presidential
II. Parliamentary system of government. d. Were citizens of the United States. c. Aristocracy
d. Monarchical
8. To which type of political system do we judiciary. Which among the following c. Democracy
belong? doctrines best describes the model of
d. Totalitarianism
governance in
a. Colonialism
14.In England, Queen Elizabeth acts as a
the Philippines?
b. Totalitarianism head of state. This country also has a
a. Tricameralism parliament
c. Democracy
b. Separation of powers and a Prime Minister. Which among the
d. Capitalism
following describes the form of
c. Emancipation of state
9. In which form of government does power government?
or authority reside in a few persons who d. Division of labor
a. Totalitarianism
govern
12.The Philippine legislature is divided into
b. Constitutional monarchy
for their own interest? two major bodies, the Senate and the
House of c. Oligarchy
a. Oligarchy
Representatives. Which among the d. Aristocracy
b. Monarchy
following describes this division of the
15.What right is violated when one opens a
c. Democracy legislative body
letter without permission from the
d. Parliamentary of the country? addressee?

10.Which of the following represents the a. Bipartisanism a. Right to privacy of communication and
smallest subunit of government in the correspondence.
b. Co- legislative power
Philippines?
b. Right to read a letter.
c. Unicameralism
a. Purok
c. Right to open the envelope without
d. Bicameralism
b. Barangay permission.
13.Which type of governance is
c. Sitio d. Right to private affairs.
characterized by a union of partially self-
d. Zone governing states or 16.When an individual is imprisoned
without proper investigation, what right is
11. The Philippine government is divided regions united by a central government?
violated?
into three branches: executive, legislative
a. Federalism
and a. Right to due process of law
b. Socialism
b. Right to secure persons
c. Right to process paper d. Right to search for private information ____________________.
d. Right to protection 20. Which principle states that no man in a. Political rights
his country is above the law and that laws
17.What law was passed by the Philippine b. Civil rights
must be
Congress in 1995 which affirms the total
c. Social rights
obeyed by all and applied to everyone – rich
integration of persons with disabilities into
or poor, lowly or powerful, without fear or d. Economic rights
the mainstream of society?
favor? 23. Which right is violated by wiretapping?
a. Rule of law a. Freedom of conversation
a. Republic Act No. 7277
b. Rule of the majority b. The right to private property
b. Republic Act No. 7784
c. Separation of church and state c. The right to privacy of communication
c. Republic Act No. 7722
d. Social justice d. Right to information on matters of public
d. Republic Act No. 7776
concern
21.In connection with government
18. An individual has the right to file writ of
transactions involving public interest which 24.In Philippine history, who was known as
amparo before the investigation of an
policy is “The Great Dissenter”?
administrative case filed against him/ her.
adopted in the Constitution to assure public a. Camilo Osias
What fundamental right is invoked by the
of accountability and transparency?
b. Manuel Roxas
individual?
a. Full public disclosure
c. Claro M. Recto
a. Right to life, liberty and security
b. Balanced and healthful ecology
d. Eulogio Rodriguez
b. Right to due process
c. Private enterprise and incentives to
25.Who first introduced the Islamic religion
c. Right to be defended by a public attorney needed investments
to the Philippines?
d. Right to self- defense d. Rural development and agrarian reform
a. Rajah Baguinda
19.What is the writ of habeas corpus? 22. Filipino citizens have the power to
b. Idi Amin
participate in the establishment or
a. Right to information privacy
administration of c. Mukdum
b. Right to accumulate data
government such as right to vote and be d. Abu Bakr
c. Right to transmit data voted upon as an exercise of
26. The Japanese successful invasion was b. Inflation c. Gender
climaxed by the surrender of the joint
c. Economic stability d. Cultural context
Filipino
d. Equilibrium 33. Which does one portray when he thinks
American forces on May 6, 1942. Where did
that what is foreign is best and that what is
this happen? 30. Tomatoes as off season and so price for
local
tomatoes are up. Which law/principle in
a. Bataan
is inferior?
Economics explains it?
b. Corregidor
a. Xenocentrism
a. Substitution effect
c. Capas
b. Relativism
b. Law of cause and effect
d. Manila
c. Ethnocentrism
c. Principle of marginal utility
27.Who is the President who is known for
d. Favoritism
his “Filipino First Policy” and Austerity d. Law of supply and demand
Program? 34. The statement that success often comes
31. Which explains GNP?
to those with humble beginnings would
a. Diosdado Macapagal
a. The total value in pesos of goods and apply
b. Carlos P. Garcia services produced during the year.
best to which of the following figures?
c. Manuel Roxas b. The total value in pesos of goods
a. Ramon Magsaysay
produced during the year.
d. Ramon Magsaysay
b. Gloria M. Arroyo
c. The total value in pesos of services
28. With whom is the “Strong Republic”
produced during the year. c. Corazon C. Aquino
associated?
d. The total value in pesos of goods and d. Joseph E. Estrada
a. Fidel V. Ramos
services produced for 2 quarters.
35.Which of the following represent
b. Joseph Estrada
32. Which term refers to social aspects of ethnocentric behavior?
c. Gloria Macapagal- Arroyo sex or to socially defined roles and
a. A tourist who lectures his foreign hosts
expectations
d. Corazon Aquino on the "uncivilized" nature of
that are associated with sex?
29.When one company controls the supply their marriage customs
of sugar, which term applies? a. Social differentiation
b. A student who tutors an immigrant in
a. Monopoly b. Social class English
c. A Hispanic community group demands c. be independent from Spain with certain 42.Which location should have most nearly
that public aid forms be published in English conditions twelve hours of daylight and twelve hours
of
and Spanish d. be represented in the Spanish Cortes
darkness during December? A town that is
d. A peace Corps volunteer who helps dig 39.How is the so-called colonial mentality
located __________.
wells in Central Africa manifested?
a. Halfway between the Equator and the
36.With the promotion of social justice in a. Cultural relativism
South Pole.
mind, which does NOT belong to the group?
b. Cultural diversity
b. Close to the Equator.
a. Equitable access to education
c. Xenocentrism
c. Close to the North Pole.
b. Profit sharing
d. Ethnocentrism
d. Close to the South Pole.
c. Diffusion of wealth
40. The maximum price that can be legally
43.Society denotes
d. Absolute right over property changed for a good or service is called
______________________.
_____.
37.Which part of the Visayas receives the
a. the totality of social organization and its
least precipitation? a. minimum wage
complex of social relationships
a. Northern b. price floor
b. a group of people who share a common
b. Eastern c. legal wage culture

c. Western d. price ceiling c. a geographical aggregate who live a


common life and interact with one another
d. Central 41.The following are legitimate children,
EXCEPT ___________. d. all of the above
38.When the Filipino reformists asked for
the assimilation of the Philippines by Spain, a. Those born by artificial insemination. e. none of the above
what
b. Those legitimated. 44.Sociologists define culture as:
did they ask for? For the Philippines to
c. Those born during a valid marriage of a. a state of refinement
________.
parents.
b. the plastic and graphics arts
a. become independent from Spain
d. Those born out of a valid marriage of the
c. activities such as TV sitcoms, soap operas
b. become a province of Spain parents.
or rock music
d. the totality of meanings, values, customs, 48.The movement of a person or group of a. Right to information privacy
norms, ideas and symbols persons to another place more or less for
b. Right to accumulate data
relative to a society permanent residence is –
c. Right to transmit data
e. none of the above a. migration
d. Right to search for private information
45.The obligation to repay a person for b. population growth
53.When a teacher is charged with an
whom one has received a favor with
c. population change administrative case committed in the lawful
undefined
discharge
d. ecological movement
quantification is ___.
of professional duties, what right may the
e. none of these
a. Pakikisama d. utang na loob teacher invoke for her defense?
49.____ is the material wealth given to the
b. kusang loob e. none of these a. Right to receive compensation for the
bride and her family before marriage.
duration of the case.
c. magandang loob
a. bride wealthd. fetish
b. Right to be given the due process of law.
46.The occurrence of graft and corruption,
b. fosterage all of these
nepotism and favoritism in the Philippines if c. Right to be defended by the organization
c. dowry of teachers to which he/ she is a member.
often attributed to the value of:
50.The division of society into layers in d. Right to be provided with free legal
a. Personalism d. hospitality
termed as – service by the appropriate office.
b. Impersonalism e. none of these
a. stratification d. life chance matrix 54.Can you be arrested without a warrant
c. utang na loob of arrest?
b. status inconsistency e. none of these
47.The type of residence which permits the a. No, if you are a minor.
c. status defects
newly married couple to reside
b. No, if you are more than 60 years old.
independently of 51.The most preferred form of marriage in
human societies is – c. Yes, if you were reported to have
the parents of the bride and the groom is –
committed a crime.
a. monogamy d. polyandry
a. patrilocal d. neolocal residence
d. Yes, if you are in the act of committing a
b. polygamy e. all of these
b. matrilocal e. none of these crime.
c. bigamy
c. bilocal 55.Can a person be imprisoned for debt?
52. What is writ of habeas data?
a. No, if he can’t pay the interest of his b. 40 62.Which is TRUE of the historical
debt. development of the Philippines?
c. 20
b. Yes, if he has no property with which to a. It has never been an independent nation.
d. 30
pay his debt.
b. It has been an independent nation ever
59.Who has the power to declare the
c. No. since.
existence of a state of war?
d. Yes, if he stubbornly refuses to pay. c. It has evolved from a colony to a fully
a. Chief justice
independent nation.
56.What does “presumption of innocence”
b. President
mean in so far as human rights are d. It has not achieved full independence
concerned? c. Senate President from the very beginning.
a. A suspect is considered guilty until d. Congress 63.Basically, the Philippine agrarian reform
proven otherwise. program is a question of _______________.
60.How is the crime of rape classified?
b. A suspect has the right to remain silent. a. Land distribution
a. Heinous
c. A suspect has the right to a legal counsel. b. Nepotism and corruption
b. Homicide
d. A suspect remains innocent until proven c. Graft and corruption
c. Slander
guilty.
d. Environmental degeneration
d. Malicious Mischief
57.No person may be elected as President
64.Which is TRUE of the Philippines?
of the Philippines unless he/ she is a 61.All Filipino citizens have the right to vote
____________. and to be voted upon as a government a. It has been colonized by two European
official. colonial powers.
a. Natural born citizen
What is this constitutional right called? b. It has never been united as a nation.
b. Resident of the Philippines for at least 2
years a. Passive right to vote c. There have been attempts to change its
government to a parliamentary
c. At least 21 years old b. Political franchise
form.
d. Professional c. Suffrage
d. It is the only Christian country in Asia.
58.A bill becomes a law even if not signed d. Electoral right
by the President after ___ days. 65.Which is TRUE of former president
Corazon Aquino?
a. 60
I. The first female president of the 68.During the Spanish era, who was the c. Goods are distributed among the
Philippines. revolutionary leader who waged a long war members.
against
II. The first female president in Asia. d. Goods are distributed among the
the government because it did not allow members but elected and appointed
III.Described as the “icon of democracy”.
proper burial for his brother? officials get
a. I, II, and III
a. Macario Sakay more to comprehensive their service.
b. I and II
b. Andres Bonifacio 71.I rent out for Php 10, 000 per month my
c. I and III only building and lot to the Jesus is Lord
c. Diego Silang
Group
d. I only
d. Francisco Dagohoy
which uses it exclusively for religious
66.Which of the following was first to
69.What was the first book published in the purposes. I am exempt from payment of
happen?
Philippines?
________________.
a. Aguinaldo was captured.
a. Del Superior Govierno
a. Property tax
b. Aguinaldo declared Philippine
b. Doctrina Christiana
independence. b. Professional tax
c. Pasiong Mahal
c. Guerilla warfare against the US was c. Income tax
initiated. d. Barlaame Josaphat
d. Community tax
d. The Philippines was ceded to the US by 70.While in the capitalist system, “to the
72.Which is an unlawful act of escaping
the Treaty of Paris. strongest goes the spoil”, what happens in
from payment of taxes?
the
67.In 1565, Legaspi concluded a blood
a. Evasion
compact with the chief of Bohol. Who is cooperative system?
referred to? b. Avoidance
a. All benefits are shared to the members
a. Raja Sulayman proportionate to the shared capital. c. Exemption
b. Sikatuna b. Goods are distributed among the d. Shifting
members but benefiting mostly the
c. Lakandula 73.What does “capital” in economics refer
policymaking
to?
d. Rajah Tupas
body.
a. Investment and loss computed
b. Outcome of business transaction 77.How do you describe a typical social
structure which describes a pattern through
c. Money/ machines invested to transact
which
business
relationships at work are ordered?
d. Profit and labor spent spent for business
a. Technological
74.Which term refers to duties payable on
goods, whether imported or exported? b. Communistic
a. Assessment c. Capitalistic
b. Tariff d. Bureaucratic
c. Subsidiary 78.When parents decide to send their
children to a private school and pay their
d. Revenue
tuition fee
75.The expression promdi connotes
instead of sending them to public school,
______________________.
they don’t get their share in their benefit
a. Ethnocentrism
derived from the taxes they pay. This is a
b. Xenocentrism case of ____________________.

c. Favoritism a. Social injustice

d. Racial discrimination b. Double jeopardy

76.The primary purpose of taxation is to c. Double taxation


_____________________.
d. Social inequity
a. Raise revenue for the support of the
10
government.
b. Reduce inequalities in wealth and
incomes.
c. Fortify the government against invaders.
d. Make the country a leading industrialized
country in the world.

You might also like